Recreatii Matematice [PDF]

  • 0 0 0
  • Gefällt Ihnen dieses papier und der download? Sie können Ihre eigene PDF-Datei in wenigen Minuten kostenlos online veröffentlichen! Anmelden
Datei wird geladen, bitte warten...
Zitiervorschau

Anul XI, Nr. 2

Iulie – Decembrie 2009

RECREAŢII MATEMATICE REVISTĂ DE MATEMATICĂ PENTRU ELEVI ŞI PROFESORI

10 ani de la apariţia revistei “Recreaţii Matematice”

e i   1 Asociaţia “Recreaţii Matematice” IAŞI - 2009

Semnificaţia formulei de pe copertă: i Într-o formă concisă, formula e  1 leagă cele patru ramuri fundamentale ale matematicii: ARITMETICA GEOMETRIA ALGEBRA ANALIZA MATEMATICĂ

reprezentată reprezentată reprezentată reprezentată

de de de de

1



i e

Redacţia revistei : Petru ASAFTEI, Dumitru BĂTINEŢU-GIURGIU (Bucureşti), Temistocle BÎRSAN, Dan BRÂNZEI, Alexandru CĂRĂUŞU, Constantin CHIRILĂ, Eugenia COHAL, Adrian CORDUNEANU, Mihai CRĂCIUN (Paşcani), Paraschiva GALIA, Paul GEORGESCU, Mihai HAIVAS, Gheorghe IUREA, Lucian-Georges Lăduncă, Mircea LUPAN, Gabriel MÎRŞANU, Alexandru NEGRESCU (student, Iaşi), Gabriel POPA, Dan POPESCU (Suceava), Florin POPOVICI (Braşov), Maria RACU, Neculai ROMAN (Mirceşti), Ioan SĂCĂLEANU (Hârlău), Ioan ŞERDEAN (Orăştie), Dan TIBA (Bucureşti), Marian TETIVA (Bârlad), Lucian TUŢESCU (Craiova), Adrian ZANOSCHI, Titu ZVONARU (Comăneşti).

Materialele vor fi trimise la una dintre adresele e-mail : [email protected] [email protected] [email protected]

COPYRIGHT © 2008, ASOCIAŢIA “RECREAŢII MATEMATICE”. Toate drepturile aparţin Asociaţiei “Recreaţii Matematice”. Reproducerea integrală sau parţială a textului sau a ilustraţiilor din această revistă este posibilă numai cu acordul prealabil scris al acesteia. TIPĂRITĂ LA SL&F IMPEX IAŞI Bd. Carol I, nr. 3-5 Tel. 0788 498933 E-mail: [email protected] ISSN 1582 - 1765

C˘ atre cititori dup˘ a zece ani de aparit¸ie a revistei Cascada cˆ ant˘ a: ˆ Imi rev˘ ars cu bucurie apele, dac˘ a un strop din ele ajunge ˆınsetatului Rabindranath Tagore

ˆIn toamna anului 1999 apare la Ia¸si primul num˘ar al revistei Recreat¸ii Matematice. Continuˆandu-¸si aparit¸ia ˆın mod constant cu dou˘a numere anual, revista a atins zece ani de existent¸˘ a ˆın prim˘avara acestui an. Revista este gˆandit˘a de init¸iatorii ei s˘a fie o continuatoare a vechii reviste Recreat¸ii S ¸ tiint¸ifice (1883–1888), de la care a preluat ¸si numele, put¸in modificat, noua revist˘a avˆand un cont¸inut strict matematic. Revista Recreat¸ii S ¸ tiint¸ifice este prima revist˘a ¸stiint¸ific˘a din ¸tar˘a care s-a adresat tineretului cu chestiuni din toate ramurile ¸stiint¸ei, dar cu un cont¸inut predominant matematic; ˆın cuvˆantul de ˆınceput al celui de-al ¸saselea an de aparit¸ie se spune, cu modestie, dar ¸si cu mˆandrie: Credem c˘ a noi am tras cea ˆınt˘ ai brazd˘ a [...] Brazda′ i mic˘ a ¸si ˆıngust˘ a, dar exist˘ a! Este o mare ˆındr˘ azneal˘ a s˘a-t¸i propui a fi continuator al unei reviste care a f˘acut oper˘a de pionerat ˆın cultura romˆaneasc˘a ¸si a avut o contribut¸ie important˘a ˆın educat¸ia matematic˘ a a tineretului. Preluˆand scopurile generoase ale creatorilor Recreat¸iilor S ¸ tiint¸ifice, actuale ¸si ast˘azi, dorint¸a noastr˘a suprem˘a este de a asigura noii reviste un nivel de calitate ¸si de utilitate la ˆın˘ alt¸imea celui avut de ”str˘abuna” sa. Acum, la ˆımplinirea a zece ani de aparit¸ie, Recreat¸iile Matematice este o revist˘a considerat˘ a printre primele publicat¸ii de gen din ˆıntreaga ¸tar˘a. Fondatorii revistei Recreat¸ii Matematice sunt (ˆın ordine alfabetic˘a): T. Bˆırsan, C. Calistru, Al. C˘ ar˘ au¸su, C. Cocea, A. Corduneanu, Gh. Iurea, care au format primul comitet de redact¸ie. ˆIn timp, li s-au al˘aturat alt¸i profesori ¸si elevi. De Alin Spum˘ a ¸si Mihai Gˆ artan, disp˘arut¸i din rˆandurile noastre, ne vom aminti cu pietate, dar ¸si cu recuno¸stint¸˘ a pentru aportul lor entuziast la aparit¸ia revistei ˆın perioada de ˆınceput a ei. Cu nr. 2/2001 intr˘ a ˆın redact¸ie Gabriel Popa, care devine ˆın scurt timp unul dintre principalii realizatori ai revistei. Revista este distribuit˘a ¸si are colaboratori, elevi ¸si profesori ˆın toat˘a ¸tara: Bˆarlad, Bra¸sov, Bucure¸sti, Craiova, Galat¸i, Hˆarl˘au, Or˘a¸stie, Pa¸scani, Pite¸sti, Rm. Vˆalcea, Satu Mare, Sf. Gheorghe (Tulcea), Suceava, Timi¸soara, Vaslui etc. Ment¸ion˘am cˆa¸tiva colaboratori din ¸tar˘ a ata¸sat¸i Recreat¸iilor Matematice: D. Popescu, F. Popovici, I. S ¸ erdean, M. Tetiva, D. Tiba, L. Tut¸escu, T. Zvonaru ¸s.a. Structura revistei comport˘a trei p˘art¸i mari: I) evenimente din lumea matematicii (conferint¸e, anivers˘ ari, comemor˘ari etc.), II) partea ”teoretic˘a” cu rubricile: articole ¸si note, nota elevului, chestiuni metodice, istoria matematicii, corespondent¸e ¸s.a., III) partea aplicativ˘a cu rubricile: concursuri ¸si examene, probleme ¸si solut¸ii, pagina rezolvitorilor. Mai sunt pres˘arate ˆın cuprinsul numerelor de revist˘a amuzamente matematice, diverse informat¸ii, premii ¸si elevi premiat¸i. Se g˘asesc prezentate ˆın paginile revistei figuri de mari matematicieni: P. Fermat, N.H. Abel, H. Poincar´e, L. Euler, A.N. Kolmogorov, W.R. Hamilton ¸s.a., ilustre nume ale matematicii romˆane¸sti: Spiru Haret, Gh. Vr˘ anceanu, Al. Myller, Gr. C. Moisil, 85

M. Haimovici ¸s.a., probleme celebre ale matematicii din toate timpurile: postulatul V al lui Euclid, construct¸ii cu rigla ¸si compasul, problema celor patru culori, conjectura lui Poincar´e ¸s.a., institut¸ii ¸si reviste care au contribuit la progresul ˆınv˘a¸t˘amˆantului ¸si cercet˘arii matematice romˆane¸sti: Seminarul Matematic ”Al. Myller” din Ia¸si, Observatorul astronomic din Ia¸si, revista Recreat¸ii S ¸ tiint¸ifice, Revista S ¸ tiint¸ific˘ a ”V. Adamachi” ¸s.a., evenimente importante: Congresele internat¸ionale ale matematicienilor romˆ ani din 2003 ¸si 2007, cˆat ¸si reflectarea altor aspecte ale viet¸ii matematice. Revista Recreat¸ii S ¸ tiint¸ifice, pe care ˆıncerc˘am s˘a o continu˘am, a fost s˘arb˘atorit˘a ˆın 2003, la 120 de ani de la aparit¸ia sa, prin publicarea unor materiale omagiale, dar ¸si a unor articole selectate din numerele acestei vechi reviste. A fost s˘arb˘atorit˘a mai amplu ˆın 2008, la 125 de ani de la aparit¸ie, ˆın cadrul unui simpozion desf˘a¸surat ˆın aula filialei Ia¸si a Academiei Romˆane. O important˘a act¸iune a fost, reeditarea celor ¸sase tomuri de Recreat¸ii S¸tiint¸ifice prin redarea textului originar cu mijloace moderne ¸si prin producerea unei variante electronice a lui de c˘atre firma Kepler Syst`emes d′ Information (director Marinela Ghigea). ˆIn rubricile p˘art¸ii ”teoretice” a revistei au fost publicate ˆın ace¸sti 10 ani, un num˘ar de 216 articole: 121 note matematice, 26 note ale elevilor, 34 chestiuni metodice, 10 ”corespondent¸e” (articole primite din afara ¸t˘arii), 9 articole adresate claselor primare ¸si 16 articole repartizate ˆın alte rubrici. Datorit˘ a str˘adaniilor unor membri ai redact¸iei revistei, au fost atra¸si, ca rezolvitori sau colaboratori cu note matematice, elevi talentat¸i, olimpici, care, f˘ar˘a ˆındoial˘a, se vor afirma ˆın scurt timp ca valoro¸si cercet˘atori ˆın domeniul matematicii: Gabriel Dospinescu, Adrian Zahariuc, Marius Pachit¸ariu, Irina Mustat¸˘ a, Vlad Emanuel, Cezar Lupu, Iurie Boreico (Chi¸sin˘au), Marius Tiba, recompensat¸i cu premii ˆın bani ¸si diplome. La rubrica Corespondent¸e articolele au ap˘arut ˆın limba ˆın care au fost redactate (romˆan˘ a, francez˘a ¸si englez˘a) ¸si provin de la corespondent¸i din ¸t˘arile: Republica Moldova, Germania, Kazahstan, Frant¸a ¸si China. Date statistice sumare dau o imagine asupra p˘art¸ii de probleme ¸si solut¸ii (toate problemele au primit solut¸ii dup˘a un an de la publicare): ciclul primar–173 probleme, gimnaziu–432, liceu–400, de tip G (avansate pentru gimnaziu)–165, de tip L (avansate pentru liceu)–165. La aceastea trebuie ad˘augate ¸si problemele date la diferite concursuri nat¸ionale care sunt prezentate ˆın revist˘a edit¸ie de edit¸ie: Concursul ”Al. Myller”, Concursul ”Florica T. Cˆ ampan”, Concursul ”Recreat¸ii Matematice” ¸s.a. Pentru trei aparit¸ii la rubrica Pagina rezolvitorilor, elevii harnici sunt premiat¸i cu cˆate o diplom˘a ¸si c˘art¸i; ˆın acest interval de 10 ani au fost premiat¸i 234 de elevi rezolvitori, cei mai mult¸i fiind din ciclul primar. Eforturile colectivului de redact¸ie au fost u¸surate datorit˘a colabor˘arii unui num˘ar mare de profesori din ˆıntreaga ¸tar˘a, care prin materialele oferite au contribuit la ridicarea nivelului revistei. Revista Recreat¸ii Matematice este recenzat˘a ˆın G.M.-A ¸si ˆın revista de referate Zentralblatt f¨ ur Didaktik der Mathematik (ZDM). Pagina web a revistei poate fi vizitat˘a la adresa: http://www.recreatiimatematice.ro

Redact¸ia revistei 86

NECULAI GHEORGHIU (1930-2009) ˆIn luna ianuarie 2009, Profesorul nostru, Neculai Gheorghiu a plecat dintre noi pentru totdeauna. Va sta de acum ˆıncolo la masa umbrelor. Scriu aceste rˆanduri, nu numai ˆın calitate de decan al Facult˘ a¸tii de matematic˘a din Ia¸si, facultate pe care Profesorul Gheorghiu a slujit-o cu d˘aruire ¸si pasiune o viat¸˘ a, ci mai ales ˆın calitate de discipol al s˘au. Am urmat dou˘a cursuri ¸tinute de domnia sa, cel de analiz˘a matematic˘a din semestrul I al anului I ¸si cel de analiz˘a funct¸ional˘ a din anul al III-lea. ˆIn plus, Profesorul Gheorghiu a fost ˆındrum˘atorul ¸stiint¸ific al lucr˘arii mele de licent¸˘ a. Ca asistent universitar, am f˘acut mult¸i ani la rˆand seminarul la cursul de analiz˘a funct¸ional˘ a din anul al III-lea. Perioada student¸iei mele a reprezentat o etap˘a de vˆarf a carierei Profesorului. Astfel, ˆın anul 1973 a ap˘arut, ˆın colaborare cu prof. Teodor Precupanu, cursul litografiat Analiz˘ a matematic˘ a, publicat ˆın 1976 la Editura didactic˘ a ¸si pedagogic˘a, iar ˆın 1974 a ap˘arut, la Editura Academiei Romˆane, cartea Introducere ˆın analiz˘ a funct¸ional˘ a. ˆIn aceea¸si perioad˘a, 1972-1976, a fost prodecan al Facult˘a¸tii de matematic˘a a Universit˘ a¸tii ”Al.I. Cuza” Ia¸si, punˆand ˆın slujba acesteia calit˘a¸tile sale de bun organizator ¸si om cu gˆandire limpede ¸si de perspectiv˘a. Anterior, fusese decanul Facult˘ a¸tii de matematic˘a a Institutului Pedagogic de 3 ani din Ia¸si, unde a fost colaborator apropiat al prof. Ilie Popa, rectorul institutului, ¸si a militat neobosit pentru o calitate superioar˘a a ˆınv˘a¸t˘amˆantului. A f˘acut parte din comitetul de conducere al Seminarului matematic ”Al. Myller” din Ia¸si. Profesorul Neculai Gheorghiu a fost ˆınzestrat cu un talent didactic de except¸ie. Lect¸iile sale erau de o claritate, de o rigoare ¸si de o naturalet¸e deosebite. Nu am predat decˆ at lucruri simple, obi¸snuia s˘a spun˘a Profesorul; ˆın realitate, concepte cu un ˆınalt grad de abstractizare ¸si teorii complexe deveneau simple ¸si transparente datorit˘a harului s˘au pedagogic. Ele au contribuit decisiv la orientarea mea spre domeniul analizei matematice. Nu put¸ini au fost ¸si mai sunt aceia care, la rˆandul lor, au afirmat ¸si afirm˘a cu recuno¸stint¸˘a c˘a, precump˘anitor, ¸si-au modelat gˆandirea matematic˘ a tocmai datorit˘a acestor lect¸ii. Avea ¸si un mod original de a se comporta cu student¸ii ¸si cu cei din jur. Era deschis, direct, sever ¸si uneori dur, dar aceste asperit˘a¸ti ale firii sale erau completate ¸si compensate de bog˘a¸tia sufletului s˘au ¸si omenia sa. Fat¸˘a de student¸i dovedea largi disponibilit˘a¸ti de a-i asculta ¸si ˆınt¸elege, dar ¸si de a-i ajuta, ˆın m˘asura ˆın care o 87

putea face. Era ˆıntotdeauna corect ¸si cinstit. Toate acestea au f˘acut ca Profesorul Neculai Gheorghiu s˘a fie foarte mult apreciat ¸si iubit de multe generat¸ii de student¸i. Aproape de fiecare dat˘a cˆand eram undeva unde se g˘aseau fo¸sti student¸i ai s˘ai, ap˘area ˆıntrebarea Ce mai face Profesorul Gheorghiu? sau, mai direct ¸si mai sugestiv, Ce mai face Nae?, ca ¸si cum Nae era un fost coleg sau un prieten comun. Avea nostalgia locului natal; vorbea cu afect¸iune de Mileanca (fostul judet¸ Dorohoi), satul ˆın care s-a n˘ascut ¸si a urmat clasele primare ¸si nu uita s˘a adauge cu mˆandrie c˘a la Mileanca se afl˘a cel mai bun cernoziom. ˆI¸si amintea cu pl˘acere de ¸scolile pe care le-a urmat ¸si de faptul c˘a a fost propun˘ator de probleme la Gazeta Matematic˘ a (toate de geometrie, ¸tinea s˘a precizeze). O dovad˘ a a posibilit˘a¸tilor ˆıntinse de comunicare a Profesorului Neculai Gheorghiu cu semenii s˘ai din domenii diverse de activitate este implicarea sa ˆın viat¸a sportiv˘a a Ia¸sului, la un moment dat avˆand funct¸ii de conducere ˆın baschetul ¸si fotbalul ie¸sean. S-a preocupat ¸si de ˆınv˘ a¸t˘amˆantul matematic preuniversitar. ˆIn repetate rˆanduri a onorat ca pre¸sedinte comisia judet¸ean˘a a Olimpiadei nat¸ionale de matematic˘ a, a ¸tinut prelegeri pentru elevi, a propus probleme la concursuri ¸si olimpiade ¸si este coautor la o frumoas˘a culegere de probleme pentru elevi intitulat˘a Matematici elementare; probleme de sintez˘ a (Editura Junimea, Ia¸si, 1983). Evident, ca orice culegere de probleme bine scris˘a ¸si aceasta cont¸ine rezolv˘ari integrale ¸si multe comentarii utile. Exist˘a acolo o superb˘a problem˘a propus˘a de Profesorul Gheorghiu la etapa final˘a a Olimpiadei de matematic˘a din 1963. Profesorul Neculai Gheorghiu avea o inteligent¸˘a sclipitoare. A avut rezultate notabile ¸si ˆın activitatea de cercetare ¸stiint¸ific˘a, domeniul s˘au predilect de preocup˘ari fiind acela al ecuat¸iilor diferent¸iale ordinare. Teza sa de doctorat, sub conducerea prof. dr. doc. Ilie Popa, cu titlul Despre comportarea asimptotic˘ a a solut¸iilor ecuat¸iilor diferent¸iale de ordinul doi a fost foarte bine apreciat˘a. Rezultatele cu totul deosebite din activitatea didactic˘ a ˆımpreun˘a cu rezultatele din activitatea de cercetare au f˘acut ca Profesorul Gheorghiu s˘a avanseze rapid ˆın cariera universitar˘a, astfel c˘a la 40 de ani era deja profesor universitar. Profesorul Neculai Gheorghiu continu˘a s˘a fie prezent printre noi ¸si dup˘a disparit¸ia sa fizic˘a; este de g˘asit ˆın rafturile de c˘art¸i ale Seminarului Matematic ”Al. Myller”, dar ¸si p˘astrat cu recuno¸stint¸˘a ¸si iubire ˆın inimile numeroaselor promot¸ii de student¸i care l-au avut ca dasc˘al. Va r˘amˆ ane mereu ˆın amintirea noastr˘a ca un profesor reprezentativ al Facult˘a¸tii de matematic˘a a universit˘ a¸tii ie¸sene, cu o contribut¸ie important˘a ˆın progresul ˆınv˘a¸t˘ amˆ antului matematic romˆanesc!

ˆ ˘ Prof. dr. Ovidiu CARJ A Decan al Facult˘ a¸tii de Matematic˘a Universitatea ”Al.I. Cuza” Ia¸si

88

Conjectura Beal pentru polinoame 1 ˆ Temistocle BIRSAN , Gabriel DOSPINESCU 2

Abstract. This paper deals with the solution of equation (1) in the set of polynomials with integer coefficients and it has an informative aim. As well as in the case of Fermat′ s Conjecture, the solution to Beal′ s Conjecture (i.e., the claim that Eq. (1) with p, q, r- integer numbers greater than 2 has no solution with x, y, z positive and mutually prime) in the set C [X]is elementary (Proposition 1). The triples (p, q, r) with p, q, r in N∗ such that equation (1) has solutions in C [X] are: (1, q, r), (2, 2, r), (2, 3, 3), (2, 3, 4), (2, 3, 5). Keywords: polynomials, Beal′ s Conjecture, Fermat′ s Conjecture, regular polyhedra. MSC 2000: 11D41.

ˆIn 1966, Andrew Beal instituie un premiu pentru demonstrarea sau infirmarea a¸sa-numitei acum Conjecturi Beal [1]: Ecuat¸ia xp + y q = z r ,

(1)

unde p, q, r sunt numere ˆıntregi mai mari ca 2, nu are nici o solut¸ie cu x, y, z ˆıntregi pozitivi ¸si relativ primi. Pentru p = q = r (= n) ecuat¸ia devine xn + y n = z n ,

(2)

ecuat¸ie care a fost subiect de preocup˘ari pentru lumea matematic˘a ˆınc˘a din antichitate ¸si mai este ¸si acum, de¸si a fost realizat˘a rezolvarea ei complet˘a. S ¸ coala lui Pitagora a demonstrat c˘a ecuat¸ia x2 + y 2 = z 2 admite o infinitate de triplete (x, y, z) formate cu numere ˆıntregi, pozitive ¸si prime ˆıntre ele ce o verific˘a. Pierre Fermat a afirmat c˘a ecuat¸ia (2) pentru n ≥ 3 nu admite nici o solut¸ie cu numere x, y, z ˆıntregi ¸si nenule ¸si a notat pe marginea unei pagini a Aritmeticii lui Diofant c˘a posed˘a o demonstrat¸ie minunat˘ a a acestui fapt. Generat¸ii de matematicieni, aflat¸i pe urmele acestei demonstrat¸ii, n-au reu¸sit s˘a o g˘aseasc˘a sau s˘a l˘amureasc˘a care ar fi putut s˘a fie aceasta, ceea ce a f˘acut pe unii s˘a presupun˘a c˘a Fermat a gre¸sit pe parcursul demonstrat¸iei sale. ˆIn sfˆar¸sit, ˆın 1995, Andrew Wiles demonstreaz˘a Marea teorem˘ a a lui Fermat, a¸sa cum istoria matematicii ret¸ine afirmat¸ia lui Pierre Fermat [6]; demonstrat¸ia lui Wiles este, ˆıns˘ a, accesibil˘a unui cerc foarte restrˆans de speciali¸sti. ˆIn acest context, apare cu atˆat mai uimitor ¸si remarcabil faptul c˘a rezolvarea ecuat¸iei (2) ˆın mult¸imea C [X] a polinoamelor cu coeficient¸i complec¸si este elementar˘a, 1 Prof.

dr., Catedra de matematic˘ a, Univ. Tehnic˘ a ”Gh. Asachi”, Ia¸si ´ Ecole Normale Sup´ erieure, Paris

2 Student,

89

accesibil˘a unui elev de liceu. Mai precis, cu ajutorul teoremei Mason - Stothers, enunt¸at˘ a mai jos, se poate dovedi afirmat¸ia urm˘atoare: Dac˘ a n ≥ 3, atunci ecuat¸ia (2) nu are solut¸ii ˆın C [X] cu polinoame neconstante ¸si relativ prime. O prezentare a acestor fapte este f˘acut˘ a ˆın [4]; cititorii revistei sunt informat¸i asupra acestui subiect ˆın [2]. ˆIn aceast˘a ordine de idei, se impune de la sine ˆınlocuirea ecuat¸iei (2) cu (1) ¸si rezolvarea acesteia din urm˘a ˆın mult¸imea C [X]. Este tocmai ceea ce ne propunem. L˘as˘ am, ˆıns˘ a, cititorilor pl˘acerea de a rezolva Conjectura lui Beal propriu-zis˘a (ˆın Z) (informat¸ii asupra premiului oferit de A. Beal sunt date ˆın [1]). Vom apela ¸si ˆın acest caz la Teorema Mason - Stothers ([4], [2]). Fie f, g, h ∈ C [X] neconstante ¸si relativ prime. Dac˘ a are loc egalitatea f + g = h, atunci max {deg f, deg g, deg h} ≤ n0 (f gh) − 1

(3)

(pentru f ∈ C [X] cu descompunerea f (X) = α

k Q

mi

(X − ai )

, se noteaz˘a deg f =

i=1

m1 + m2 + · · · + mk – gradul lui f ¸si n0 (f ) = k – num˘arul r˘ad˘acinilor sale distincte). Demonstrat¸ia este elementar˘a ¸si poate fi g˘asit˘a ˆın [2]. Propozit¸ia 1 (Conjectura Beal pentru polinoame). Dac˘ a p, q, r ∈ Z ¸si p ≥ 3, q ≥ 3, r ≥ 3, atunci ecuat¸ia (1) nu are solut¸ii ˆın C [X] cu polinoame neconstante ¸si relativ prime. Demonstrat¸ie. Proced˘am ca ¸si ˆın cazul ecuat¸iei (2). Presupunem c˘a ecuat¸ia (1) cu exponent¸ii p, q, r satisf˘acˆand condit¸iile din enunt¸ admite o solut¸ie (f, g, h) cu polinoame neconstante relativ prime. Ca urmare, putem aplica Teorema Mason – p q r Stothers polinoamelor (f (X)) , (g (X)) , (h (X)) ¸si scrie p

p

q

r

deg (f (X)) ≤ n0 (f (X)) (g (X)) (h (X)) − 1, de unde p deg f (X) ≤ n0 (f (X) g (X) h (X)) − 1 ¸si, deci, p deg f (X) ≤ n0 (f (X)) + n0 (g (X)) + n0 (h (X)) − 1. Scriind ¸si inegalit˘a¸tile analoage acesteia relativ la polinoamele g ¸si h ¸si adunˆandu-le membru cu membru, obt¸inem relat¸ia (p − 3) deg f (X) + (q − 3) deg g (X) + (r − 3) deg h (X) ≤ −3, care este fals˘a, c˘aci p ≥ 3, q ≥ 3, r ≥ 3, ¸si demonstrat¸ia este ˆıncheiat˘a. ˆInainte de a vedea ce se ˆıntˆampl˘a atunci cˆand unul (cel put¸in) dintre exponent¸ii ecuat¸iei (1) este mai mic ca 3, vom face cˆateva 90

Observat¸ii. 1) ˆIn prezent¸a egalit˘a¸tii f + g = h, faptul c˘a polinoamele f, g, h sunt prime ˆıntre ele este echivalent cu condit¸ia ca f, g, h s˘a fie relativ prime dou˘a cˆate dou˘a. 2) Dac˘a εp1 = 1, εq2 = 1 ¸si εr3 = 1 ¸si (x, y, z) este o solut¸ie ˆın C [X] a ecuat¸iei (1), atunci sunt solut¸ii ale acestei ecuat¸ii ¸si tripletele: (ε1 x, ε2 y, ε3 z), (x, ε2 y, ε3 z), (x, y, ε3 z) etc. Vom face abstract¸ie de aceste solut¸ii derivate ale unei solut¸ii g˘asite. 3) F˘ar˘ a a restrˆange generalitatea, putem considera c˘a ˆın tripleta (p, q, r) a exponen¸tilor ecuat¸iei (1) avem p ≤ q ≤ r.

(4)

ˆIntr-adev˘ ar, putem lua p ≤ q schimbˆand, eventual, x ¸si y ˆıntre ele ˆın ecuat¸ia (1). p Dac˘a r ≤ p, scriem (1) sub forma z r + (ε1 x) = y q , unde εp1 = −1. Dac˘a p ≤ r ≤ q, r q punem (1) sub forma xp + (ε2 z) = (ε3 y) , unde εr2 = εq3 = −1. Conform Propozit¸iei 1 ¸si ¸tinˆ and seama de (4), rezult˘a c˘a p ∈ {1, 2}. I Cazul (1,q,r ) este banal: solut¸ia general˘a a ecuat¸iei x + yq = zr

(5) este dat˘a de (6)

x = hr − g q ,

y = g,

z = h,

∀g, h ∈ C [X] ,

pentru orice exponent¸i q, r ∈ N∗ . II Cazul (2,q,r ). Cu teorema Mason - Stothers vom obt¸ine limit˘ari importante ˆın privint¸a exponent¸ilor q ¸si r. Fie (x, y, z) o solut¸ie ˆın C [X] a ecuat¸iei x2 + y q = z r (cu x, y, z neconstante ¸si relativ prime) ¸si fie a = deg x, b = deg y, c = deg z. Teorema amintit˘a ne spune c˘a €

Š

max {2a, qb, rc} ≤ n0 x2 y q z r − 1, de unde max {2a, qb, rc} ≤ a + b + c − 1, deci avem (7)

a < b + c − 1 ¸si

max {qb, rc} ≤ 2 (b + c − 1) .

1 q Cum max {qb, rc} ≥ (qb + rc) ≥ (b + c), combinˆand cu a doua relat¸ie din (7) vom 2 2 q obt¸ine (b + c) ≤ 2 (b + c − 1), ceea ce conduce la q ∈ {2, 3}. 2 II.1 Subcazul (2,2,r ), cu r ≥ 2. Ecuat¸ia (8)

x2 + y 2 = z r

se mai scrie (x + iy) (x − iy) = z r 91

¸si cum x + iy ¸si x − iy sunt polinoame prime ˆıntre ele rezult˘a c˘a fiecare este puterea de exponent r a unui polinom: x + iy = f r ¸si x − iy = g r . ˆIn final, ecuat¸ia x2 + y 2 = z r are solut¸ia dat˘a de (9)

x=

1 r (f + g r ) , 2

y=

1 (f r − g r ) , 2i

z = f g,

∀f, g ∈ C [X] .

Verificarea faptului c˘a (9) este o solut¸ie a ecuat¸iei (8) este imediat˘a. II.2 Subcazul (2,3,r ), cu r ≥ 3. Din (7), avem max {3b, rc} ≤ 2 (b + c − 1), din care rezult˘a c˘a b < 2 (c − 1) ¸si rc < 6 (c − 1) , deci r ∈ {3, 4, 5}. A¸sadar, au r˘amas trei situat¸ii de analizat: (2, 3, 3), (2, 3, 4) ¸si (2, 3, 5). II.2.1 Ecuat¸ia x2 + y 3 = z 3 . Vom vedea c˘a rezolvarea acestei ecuat¸ii se reduce la un caz anterior studiat. ˆIntr-adev˘ar, scriem ecuat¸ia ˆın forma (10)

€

Š

(z − y) (z − εy) z − ε2 y = x2 ,

unde ε2 + ε + 1 = 0. ˆIntrucˆat z − y, z − εy ¸si z − ε2 y sunt relativ prime dou˘a cˆate dou˘a, din egalitatea precedent˘a deducem c˘a fiecare dintre acestea este p˘atratul unui polinom: z − y = f 2 , z − εy = g 2 , z − ε2 y = h2 . Pentru ca acest sistem liniar ˆın z ¸si y s˘a fie compatibil, impunem polinoamelor f , g, h condit¸ia −εf 2 + (1 + ε) g 2 = h2 . Aceasta, ˆıns˘ a, se reduce dup˘a substitut¸ii evidente la ecuat¸ia x2 + y 2 = z 2 , care se rezolv˘a conform cazului II.1. II.2.2 Ecuat¸ia x2 + y 3 = z 4 . Proced˘am ca ˆın cazul precedent. Scriem ecuat¸ia ˆın discut¸ie sub forma (11)

€

z2 − x

Š€

Š

z2 + x = y3 .

Cum z 2 − x ¸si z 2 + x sunt relativ prime, urmeaz˘a c˘a ele sunt cuburi de polinoame: z2 − x = f 3,

z2 + x = g3 .

Astfel, suntem condu¸si la ecuat¸ia 2z 2 = f 3 + g 3 care se reduce la rˆandul ei la ecuat¸ia x2 + y 3 = z 3 ˆıntˆ alnit˘ a ˆın subcazul II.2.1. Practic, solut¸iile ecuat¸iei x2 + y 3 = z 4 vor fi obt¸inute parametric ¸si se vor exprima ˆın funct¸ie de solut¸iile cazului (2, 2, r). II.2.3 Ecuat¸ia x2 + y 3 = z 5 . Rezolvarea acestei ecuat¸ii pare s˘a fie deosebit de dificil˘a. ˆIn lipsa unei descompuneri de tipul (10) sau (11), nu putem proceda ca mai sus. 92

Stabilim doar faptul c˘a ecuat¸ia are solut¸ii. Se poate verifica direct c˘a tripleta (x, y, z) dat˘a de x = X 30 + 522X 25 − 10005X 20 − 10005X 10 − 522X 5 + 1, (12)

y = −X 20 + 228X 15 − 494X 10 − 228X 5 − 1, € Š √ 5 z = 1728 X 11 + 11X 6 − X

este o solut¸ie a acestei ecuat¸ii ([3] , [5]). Observat¸ie. ˆIn [3], Felix Klein pune ˆın evident¸˘a leg˘atura strˆans˘a care exist˘a ˆıntre poliedrele regulate ¸si solut¸iile ˆın C(X) ale ecuat¸iei (1). Astfel, cazul (2, 3, 3) este legat de tetraedrul regulat, cazul (2, 3, 4) de cub ¸si octogonul regulat, iar cazul (2, 3, 5), cu solut¸ia (12), de dodecaedrul ¸si de icosaedrul regulat. Aceast˘a leg˘atur˘ a este ment¸ionat˘ a ¸si discutat˘a ˆın [5]. ˆIn concluzie, tripletele de exponent¸i (p, q, r), p, q, r ∈ N∗ , pentru care ecuat¸ia (1) are solut¸ii ˆın C [X] sunt: (1, q, r), (2, 2, r), (2, 3, 3), (2, 3, 4), (2, 3, 5) ¸si toate permut˘arile acestora. Bibliografie 1. *** - Beals Conjecture, The New Zeland Math. Mag., 35(1998), no.2, 38. 2. T. Bˆırsan - Marea teorem˘ a a lui Fermat pentru polinoame, RecMat - 1/2004, 5-9. 3. F. Klein - Vorlesungen u ¨ber das Ikosaeder und die Aufl¨ osung der Gleichungen vom f¨ unften Graden, Teubner, Leipsig, 1884. 4. S. Lang - Math Talks for Undergraduates, Springer, 1999. 5. V. V. Prasolov - Essays on Numbers and Figures, Amer. Math. Soc., Mathematical World, v. 16, 2000. 6. A. Wiles - Modular elliptic curves and Fermat ′ s Last Theorem, Annals of Math., 142(1995), 443-551.

ˆIn 1962 Bachet de M´ eziriac public˘a o versiune latin˘a a Aritmeticii ce includea peste o sut˘a de probleme ¸si avea margini largi ale textului. Pe paginile unui astfel de exemplar, Pierre Fermat ˆı¸si nota solut¸iile, comentariile ¸si rezultatele proprii. Pe marginea C˘art¸ii a II-a Fermat not˘a afirmat¸ia : ecuat¸ia xn + y n = z n nu are solut¸ii ˆın numere ˆıntregi ¸si nenule pentru n ≥ 3. cunoscut˘a acum ca Marea Teorem˘ a a lui Fermat. Apoi a scris comentariul: (continuare la pagina 20) 93

Teorema lui Brouwer - un caz particular elementar Cornelia-Livia BEJAN 1 Abstract. By following the ideas developed by T. Traynor in [2], an elementary proof of Brouwer′ s fixed point theorem is presented for the restricted case of continuously differentiable functions. Keywords: closed ball, ball, fixed point, continuously differentiable function. MSC 2000: 54H25.

Teorema de punct fix a lui Brouwer are o istorie lung˘a. Ideile ce conduc la demonstrat¸ia acesteia se g˘asesc la Henri Poincar´e ˆınainte de 1886. L.E.J. Brouwer a demonstrat teorema pentru n = 3 ˆın 1909. ˆIn 1910 J. Hadamard d˘a prima demonstrat¸ie pentru n arbitrar, iar Brouwer d˘a o alta ˆın 1912. ˆIn [2], autorul a dat o prezentare elementar˘a a teoremei de punct fix a lui Brouwer. ˆIn aceast˘a not˘a, adaptˆand demonstrat¸ia elementar˘a dat˘a ˆın [2], ne propunem s˘a prezent˘ am un caz particular al teoremei lui Brouwer, ˆıntr-o form˘a ¸si mai accesibil˘a profesorilor ¸si student¸ilor. Peste tot vom nota cu D discul unitate din plan ¸si cu C frontiera sa, adic˘a cercul. Teorema de punct fix a lui Brouwer afirm˘a c˘a orice aplicat¸ie continu˘ a f : D → D admite m˘ acar un punct fix, adic˘ a exist˘ a m˘ acar un element x ∈ D astfel ˆıncˆ at f (x) = x. Altfel spus, dac˘a deform˘am ˆın mod continuu discul unitate ˆın el ˆınsu¸si, atunci exist˘a m˘acar un punct care nu-¸si schimb˘a pozit¸ia. Vom prezenta un caz particular al acestei teoreme (ipoteze ˆınt˘arite) ¸si anume: Teorem˘ a. Dac˘ a f : D → D este o funct¸ie cu derivatele de ordinul ˆıntˆ ai continue, atunci ea admite m˘ acar un punct fix. Lem˘ a. Nu exist˘ a nici o aplicat¸ie f : D → C cu derivatele de ordinul ˆıntˆ ai continue care s˘ a lase fixe toate punctele cercului, adic˘ a f (x) = x, ∀x ∈ C. Demonstrat¸ie. Presupunem c˘a ar exista o astfel de aplicat¸ie f ¸si not˘am g(x) = f (x) − x, ∀x ∈ D. Constat˘am c˘a g este o funct¸ie cu derivatele de ordin ˆıntˆai continue ¸si, ˆın consecint¸˘ a, majorate (ˆın norm˘a) de o constant˘a k. Din teorema valorii medii avem ∥g(x) − g(y)∥ ≤ k∥x − y∥, ∀x, y ∈ D, adic˘a distant¸a ˆıntre g(x) ¸si g(y) este majorat˘a de distant¸a dintre punctele x ¸si y, multiplicat˘a cu k. Definim aplicat¸ia ft (x) = x + t · g(x) = (1 − t)x + tf (x), ∀t ∈ [0, 1]. Se verific˘a faptul c˘a ft las˘ a fixe punctele cercului, ∀t ∈ [0, 1]. Ar˘at˘am c˘a ft este injectiv˘a pentru 0 ≤ t < k1 . ˆIntr-adev˘ ar, dac˘a ft (x) = ft (y), atunci ∥x − y∥ = t∥g(x) − g(y)∥ ≤ tk∥x − y∥, de unde obt¸inem x = y ˆıntrucˆat tk < 1. ˆIn plus, pentru fiecare t ∈ [0, 1] fixat, avem ft′ (x) = I +tg ′ (x), unde I este aplicat¸ia identic˘ a a planului. Pentru 0 ≤ t < k1 se vede c˘a aplicat¸ia ft duce discul D ˆın el ˆınsu¸si, adic˘a ft (D) = D. ˆIn continuare s˘a scriem: Z

π = Aria(D) = Aria(ft (D)) =

detft′ (x)dx, 0 ≤ t
0. Evident, aplicat¸ia f are derivatele de ordinul ˆıntˆai continue, iar h(x) = x, ∀x ∈ C. Pe de alt˘a parte, avem: Caz 1. Dac˘ a (x · f (x))x = (x · x)f (x), atunci h(x) =

x − (x · f (x))x − f (x) + (x · x)f (x) x − f (x) = ̸= 0, 1 − x · f (x) 1 − x · f (x)

deoarece am presupus c˘a f nu are puncte fixe. Caz 2. Dac˘a (x·f (x))x ̸= (x·x)f (x) rezult˘a c˘a x ¸si f (x) nu sunt liniar dependente. Dup˘a modul cum este definit˘a aplicat¸ia h, deducem c˘a h(x) ̸= 0 c˘aci ˆın caz contrar x ¸si f (x) ar fi liniar dependente. ˆIn concluzie, constat˘am c˘a h nu se anuleaz˘a ¸si deci putem defini o aplicat¸ie H(x) = h(x) , ∀x ∈ D. Se vede c˘a H : D → C are derivatele de ordin ˆıntˆai continue ¸si ∥h(x)∥ fixeaz˘a punctele cercului, adic˘a H(x) = x, ∀x ∈ C. ˆIn baza Lemei, ˆıns˘a, am ajuns la o absurditate, ˆıntrucˆ at o astfel de aplicat¸ie H nu exist˘a. Demonstrat¸ia este ˆıncheiat˘a. Observat¸ie. O generalizare de la plan la spat¸iul cu n dimensiuni se face urmˆand pas cu pas calea parcurs˘a mai sus. ˆIn liceu, elevii ˆıntˆalnesc teorema lui Brouwer ˆın cazul unidimensional: o funct¸ie f : [0, 1] → [0, 1] continu˘ a are cel put¸in un punct fix. Bibliografie ¨ 1. L.E.J. Brouwer - Uber Abbildungen von Mannigfaltigkeiten, Math. Ann., 71(1912), 97-115. 2. T. Traynor - An Easy Analitic Proof of Brouwer′ s Fixed Point Theorem, Atti Sem. Mat. Fis. Univ. Modena, XLIV (1996), 479-483. 95

Exponentul de triangularitate al unui triunghi 1 ˘ Dorin MARGHIDANU Abstract. The following problem is investigated: if a, b, c denote the lengths of the sides of a triangle, it is required to determine the values of the real and positive exponents α such that the powers aα , bα , cα can still be the side lengths of a triangle. It is introduced the notion of triangularity exponent t and it is proved that t = 2 for the right-angled triangles, t ∈(1, 2) for the obtuse-angled triangles and t > 2 for the acute-angled triangles (Proposition 2). Keywords: triangularity exponent. 2000 MSC: 51M15.

ˆIn aceast˘a not˘a d˘am un r˘aspuns la urm˘atoarea ˆıntrebare: Dac˘ a a, b, c sunt lungimile laturilor unui triunghi, pentru care numere α reale ¸si pozitive puterile aα , bα , ca pot forma de asemenea un triunghi? ˆIn prima parte a notei prezent˘am instrumentul algebric de lucru – o extindere a inegalit˘a¸tii 11.19 din [1], p.99, iar ˆın cea de-a doua introducem exponentul de triangularitate al unui triunghi, not¸iune necesar˘a rezolv˘arii problemei propuse. 1. La fel cum a fost demonstrat˘a inegalitatea 11.19 din [1], putem stabili ¸si Propozt¸ia 1. Dac˘ a pentru numerele reale strict pozitive a, a1 , a2 , . . . , an exist˘ a α α + . . . + a , atunci au loc: + a un num˘ ar real α > 0 astfel ˆıncˆ at aα = aα n 2 1 (1) 1) aβ > aβ1 + aβ2 + . . . + aβn , ∀β ∈ R, β > α; (2) 2) aβ < aβ1 + aβ2 + . . . + aβn , ∀β ∈ R, 0 < β < α. Demonstrat¸ie. Din relat¸ia de condit¸ie rezult˘a c˘a a > a1 , a > a2 , . . . , a > an . , . . . , aβ−α > aβ−α ¸si avem 1) β > α implic˘ a aβ−α > aβ−α n 1 β−α α β−α · aan · aα aβ = aβ−α · aα = aβ−α (aα 1 + ... + a 1 + . . . + an ) = a β β−α β · aα > aβ−α · aα n = a1 + . . . + an . 1 + . . . + an 1

¸si, inversˆand semnul de inegali2) β < α implic˘ a aβ−α < aβ−α , . . . , aβ−α < aβ−α n 1 tate ˆın calculul de la punctul 1), obt¸inem (2). α α Observat¸ii. 1. Relat¸ia (1) r˘amˆane valabil˘a ¸si ˆın condit¸ia aα > aα 1 + a2 + . . . + an . 2. Pentru n = 2, α = 2 ¸si β ∈ N, β > 2, punctul 1) revine la afirmat¸ia 11.19 [1].

Corolar. Dac˘ a a, b, c sunt lungimile laturilor unui triunghi ¸si este verificat˘ a condit¸ia aα = bα +cα , atunci (aβ , bβ , cβ ) formeaz˘ a un triunghi dac˘ a ¸si numai dac˘ a β < α. Vom indica dou˘a aplicat¸ii directe, ˆın geometrie, ale rezultatelor precedente. Aplicat¸ia 1. ˆ Intr-un paralelipiped dreptunghic de muchii a, b, c ¸si diagonal˘ a mare D, avem Dp > ap + bp + cp , ∀p ∈ R, p > 2. Demonstrat¸ie. Fie d diagonala fet¸ei de laturi a, b. Conform punctului 1) al Propozit¸iei 1 (cu n = 2, α = 2), aplicat de dou˘a ˆın cazul particular al triunghiului dreptunghic avem Dp > dp + cp > ap + bp + cp . 1 Prof.

dr., Liceul Teoretic din Corabia, [email protected]

96

Aplicat¸ia 2. Fie OABC un tetraedru tridreptunghic cu vˆ arful O ¸si OA = a, OB = b, OC = c. Atunci AB p + BC p + CAp > 2(ap + bp + cp ), ∀p ∈ R, p > 2. Demonstrat¸ie. Cu Propozit¸ia 1, aplicat˘a la triunghiurile dreptunghice OAB, OBC ¸si OCA, obt¸inem relat¸iile AB p > ap + bp , BC p > bp + cp ¸si CAp > cp + ap care, adunate, conduc la inegalitatea din enunt¸. 2. Vom introduce o not¸iune, cu rol decisiv, prin urm˘atoarea Definit¸ie. Num˘ arul real pozitiv t se nume¸ste exponent de triangularitate al triunghului ABC cu laturi de lungimi a, b, c, dac˘a tripleta (as , bs , cs ) formeaz˘a un triunghi pentru orice s ∈ R∗+ , s < t, ¸si nu formeaz˘a un triunghi pentru s ≥ t. Rezolvarea problemei propuse este dat˘a de Propozit¸ia 2. Sunt adev˘ arate urm˘ atoarele afirmat¸ii: 1) t = 2, dac˘ a triunghiul este dreptunghic; 2) t ∈ (1, 2), dac˘ a este obtuzunghic; 3) t > 2, dac˘ a este ascut¸iunghic; ˆın acest caz, t este num˘ ar real (finit), dac˘ a triunghiul nu-i isoscel sau dac˘ a este isoscel ¸si unghiul opus bazei sale este strict mai mare de 60◦ ¸si t = +∞, dac˘ a triunghiul este isoscel ¸si are unghiul opus bazei mai mic sau egal ca 60◦ . Demonstrat¸ie. Fie ABC cu a = max{a, b, c}. Consider˘am funct¸ia continu˘a f (x) = ax − bx − cx , x > 0. 1) Triunghiul fiind dreptunghic, avem a2 = b2 + c2 . Conform Corolarului de mai sus, deducem c˘a t = 2. 2) ˆIn acest caz, f (1) = a − b − c < 0 ¸si f (2) = a2 − b2 − c2 > 0 (triunghiul ABC fiind obtuzunghic). Rezult˘a c˘a exist˘a o valoare t∈(1, 2) (unic˘a, conform Propozit¸iei 1) astfel ˆıncˆ at f (t) = 0, adic˘a at = bt + ct . Din nou apelˆand la Corolar, deducem c˘a t este exponentul de triangularitate al triunghiului. 3) Avem f (2) = a2 − b2 − c2 < 0 (triunghi ascut¸iunghic) ¸si f (∞) = lim f (x) = •

 ‹x

˜

x→∞

 c x b − = +∞, dac˘a triunghiul nu-i isoscel sau dac˘a este isoscel lim a 1 − x→∞ a a ¸si baza sa este mai mare ca laturile sale (echivalent, m˘asura unghiului din vˆarf este strict cuprins˘a ˆıntre 60◦ ¸si 90◦ ). Deci, exist˘a t ∈ (2, ∞) unic astfel ˆıncˆat at = bt + ct . Conchidem c˘a t astfel g˘asit este exponentul de triangularitate ˆın subcazul considerat. ˆIn sfˆar¸sit, dac˘a triunghiul este isoscel cu baza mai mic˘a sau cel mult egal˘a cu laturile sale (echivalent, unghiul din vˆarf are m˘asura mai mic˘a sau cel mult 60◦ ), atunci se constat˘a direct c˘a puterile de exponent α ale laturilor acestuia formeaz˘a un triunghi de acela¸si tip, oricare ar fi α ∈ R. A¸sadar, ˆın acest subcaz avem t = +∞. x

Considerat¸iile precedente sugereaz˘ √ √ √ a examinarea triunghiurilor care verific˘a condit¸ii ca a3 = b3 + c3 (sau a = b + c etc.), a¸sa cum s-a f˘acut ˆın cazul triunghiurilor dreptunghice (a2 = b2 + c2 ) sau altor triunghuiuri speciale (triunghiuri mediane, triunghiuri cu laturi ˆın progresie aritmetic˘a etc.). Bibliografie 1. O. Bottema, R.Z. Djordjevi´ c, R.R. Jani´ c, D.S. Mitrinovi´ c, P.M. Vasi´ c – Geometric inequalities, Wolters-Noordhoff, Groningen, 1969. 97

Asupra unui ¸sir de integrale Riemann Dan POPESCU 1 Abstract. This Note is an extension of a former paper–[3] in the reference list. The author remarks that many problems proposed as topics to various contests are direct consequences of Propositions 1 and 2 in the note. Keywords: continuous function, periodic function, Rimann integral. MSC 2000: 26A42.

ˆIn cele ce urmeaz˘a, sunt prezentate dou˘a condit¸ii suficiente de convergent¸˘a a ¸sirului real de integrale   Z

b

f (x) g (nx) dx

,

a

n∈N

unde f ¸si g sunt funct¸ii integrabile Riemann care asigur˘a corectitudinea definirii ¸sirului. Un prim rezultat este prezentat ˆın [3]: Propozit¸ia 1. Fie f : [0, T ] → R o funct¸ie integrabil˘ a Riemann ¸si g : [0, ∞) → R o funct¸ie cu perioada T >0, astfel ˆıncˆ at restrict¸ia g|[0,T ] este integrabil˘ a Riemann. Atunci Z

(1)

T

lim

n→∞

0

1 f (x) g (nx) dx = T

Z

 Z

T



T

f (x) dx

g (x) dx .

0

0

O aplicat¸ie direct˘a a acestui rezultat este urm˘atoarea: Fie f : R → (0, ∞) o funct¸ie continu˘ a cu perioada 1. Atunci Z

lim

n→∞

1

Z

f (x) · f (nx) dx =

1

2

f (x)

.

0

0

(Cristinel Mortici, etapa judet¸ean˘ a, 2003) ˆIn lucrarea [2], se propune tot o consecint¸˘a direct˘a a rezultatului (1): Fie funct¸ia f : R → R continu˘ a ¸si periodic˘ a cu perioada T > 0. S˘ a se demonstreze c˘ a, pentru a, b ∈ R, a < b, avem Z b Z b−a T lim f (nx) dx = f (x) dx. x→∞ a T 0 Tot ca o consecint¸˘ a a rezultatului (1), este prezentat˘a urm˘atoarea problem˘a: Fie funct¸ia continu˘ a f : [0, 1] → R. S˘ a se demonstreze c˘ a ¸sirul (an )n≥1 , definit Z

1

de relat¸ia an =

2

{nx} f (x) dx, este convergent ¸si s˘ a i se afle limita, ˆın funct¸ia f ,

0

unde {x} = x − max {k ∈ Z; k ≤ x} ,∀x ∈ R. (Octavian Purcaru, Lista scurt˘ a, O.N.M., 2003) 1 Profesor,

Colegiul Nat¸ional ”S ¸ tefan cel Mare”, Suceava

98

2

Demonstrat¸ie. Cum funct¸ia g : R → Rg (x) = {x} este periodic˘a cu perioada principal˘ a 1, au loc: Z

Z

1

an = lim

n→∞

Z

1

deoarece

0

 Z

1

f (x) g (nx) dx = 0

Z

1

f (x) dx

 2

{x} dx

0

=

1 3

Z

1

f (x) dx, 0

1

2

{x} dx =

x2 dx, funct¸iile de integrat fiind egale pe [0, 1) . 0

0

ˆIn lucrarea [1], apare urm˘atoarea problem˘a, semnat˘a de Mihail Bencze: Z

1

S˘ a se calculeze lim

n→∞

t2 {nt} dt, unde {x}are semnificat¸ia de mai sus.

0

Un al doilea rezultat util: Propozit¸ia 2. Fie funct¸iile continue g : [0, ∞) → R ¸si f : [0, a] → R, unde a >0. Dac˘ a lim g (x) = L ∈ R, atunci x→∞

Z

(2)

lim

n→∞

Z

a

a

f (x) dx.

g (nx) f (x) dx = L 0

0

Demonstrat¸ie. ˆIntr-adev˘ ar, dac˘a h (x) = g (x) − L, ∀x ∈ [0, ∞), atunci Z

Z

a

Z

a

g (nx) f (x) dx = 0

Z

a

(h (nx) + L) f (x) dx =

h (nx) f (x) dx + L

0

0

Z

a

0

1 h (nx) f (x) dx = n

Z

 ‹

na

h (t) f 0

t dt. n

Cum ∃b > 0 astfel ˆıncˆ at |f (x)| ≤ b, ∀x ∈ [0, a] , Z

a

0

Z Z ab na b na |h (t)| dt = |h (t)| dt. h (nx) f (x) dx ≤ n 0 na 0

Dac˘a H este o primitiv˘a pentru |h| pe intervalul [0, ∞) , Rx 0

lim

x→∞

|h (t)| dt H (x) − H (0) = lim = lim |h (x)| = 0, x→∞ x→∞ x x Z

a

ceea ce asigur˘a c˘a lim

x→∞

h (nx) f (x) dx = 0. 0

Observat¸ie. Relat¸ia (2) se poate scrie ¸si ′

(2 )

Z

lim

n→∞

0

a



1 g (nx) f (x) dx = lim a→∞ a 99

Z

‹Z

a

a

g (x) dx 0

f (x) dx. 0

Notˆand nx = t, se obt¸ine:

f (x) dx, 0

a

ceea ce justific˘a prezentarea celor dou˘a ¸siruri de integrale Riemann ˆımpreun˘a . O aplicat¸ie a acestui rezultat: Fie funct¸iile f : [0, ∞) → R ¸si g : [0, 1] → R, astfel ca lim f (x) = L ∈ R. S˘ a se x→∞ demonstreze c˘ a Z Z 1 x 1 n lim f (x) g g (x) dx. dx = L n→∞ n 0 n 0 (Laurent¸iu Panaitopol, etapa judet¸ean˘ a, 2003) Demonstrat¸ie. Cu substitut¸ia x = nt, limita devine Z

Z

1

lim

1

f (nt) g (t) dt = L

n→∞

0

g (x) dx 0

¸si finalizarea este clar˘a. Propunem cititorului urm˘atoarele exercit¸ii: 1. S˘a se demonstreze c˘a Z

π

lim

n→∞

Z n→∞

0 1

2. S˘a se calculeze lim

e−x cos nx dx =

2  π. 1 − e−π

{nx} [x] dx, justificˆandu-se ˆıntˆai existent¸a limitei, unde

0

[x] = max {k ∈ Z|k ≤ x} ¸si {x} = x − [x] , ∀x ∈ R. 3. S˘a se demonstreze c˘a, pentru orice funct¸ie continu˘a f : [0, a] → R, are loc relat¸ia Z a

lim

n→∞

e−nx f (x) dx = 0.

0

Bibliografie 1. D.M. B˘ atinet¸u - Giurgiu ¸s.a. – Analiz˘ a matematic˘ a. Probleme pentru clasa a XII-a, Editura Matrix, 2004. 2. R. Miculescu ¸s.a. – Probleme de calcul integral, Editura GIL, 2005. 3. D. Popescu, F. Popovici – O generalizare a lemei lui Riemann, Recreat¸ii Matematice, Ia¸si, 4(2002), nr.1, 12-13.

100

Dualitatea unor sume combinatoriale Andrei VERNESCU 1 Abstract. The main result consist in the combinatorial identity (6), where Ωn is the notation in (1). Some connections of this identity with other combinatorial identities of similar kind, that are known or are expected to be established, are formulated. Keywords: combinatorial identity. MSC 2000: 05A19.

1. ˆIn lucrarea [1], din num˘ arul 2/2007 al acestei reviste, s-a aplicat o ideee foarte (2k − 1)!! elegant˘a pentru calculul unei sume care cont¸ine expresiile , k = 1, 2, 3, . . . , n. (2k)!! ˆIn nota de fat¸˘ a venim ˆın continuarea lucr˘arii citate, prezentˆand calculul bazat pe aceea¸si idee al altei sume, care ar putea fi considerat˘a, din punctul de vedere al mijloacelor folosite, ca fiind duala celei precedente. Pentru facilitarea calculelor care vor urma, vom utiliza notat¸ia (1)

Ωn =

(2n − 1)!! (2n)!!

(n = 1, 2, 3, . . .).

(Am introdus aceast˘a notat¸ie ˆın manuscrisul din 1987 al c˘art¸ii [11], am publicat-o pentru prima dat˘a ˆın articolul [6], am folosit-o ˆın toate edit¸iile culegerii de probleme [7], ca ¸si ˆıntr-o serie de lucr˘ari: [8], [9], [11] ¸si [13].) Tot ˆın scopul scrierii mai simple a formulelor ¸si calculelor care vor urma, este necesar s˘a definim Ωn ¸si pentru n = 0. Pentru aceasta, ˆın cazul n ≥ 1, s˘a amplific˘am (2n)! fract¸ia din partea dreapt˘a a egalit˘a¸tii (1) cu (2n)!! = 2n n!. Obt¸inem Ωn = n 2 . (2 n!) Ca aceast˘a formul˘ a s˘a poat˘a fi prelungit˘a ¸si pentru n = 0, convenim s˘a punem prin definit¸ie (2)

Ω0 = 1.

Cu aceast˘a notat¸ie, identitatea care formeaz˘a rezultatul principal al articolului citat [1] se scrie concentrat astfel: n X

(3)

(−1)k Cnk Ωk = Ωn .

k=0

Ea poate fi considerat˘a din dou˘a puncte de vedere: ca formul˘a de ˆınsumare combinatorial˘ a, precum ¸si ca identitate referitoare la expresiile Ωk . 2. Obt¸inerea identit˘ a¸tii (3) se efectueaz˘a ˆın [1] utilizˆand integralele: Z

(4)

Z

π/2

In =

cos x dx = 0

1 Conf.

π/2

sinn x dx,

n

0

dr., Departamentul de S ¸ tiint¸e, Univ. ”Valahia”, Tˆ argovi¸ste

101

π pentru care I0 = , I1 = 1 ¸si pe care, folosind notat¸ia (1) ˆımpreun˘a cu definit¸ia (2), 2 putem s˘a le mai scriem sub forma: 8 π > < Ωk ,

dac˘a n = 2k 2 In = 1 1 > : , dac˘a n = 2k + 1. · Ωk 2k + 1

(5)

k−1 Ik−2 , k ≥ 2, a expresiilor integralei k In – scrise ˆıns˘ a f˘ar˘ a a folosi expresia Ωk – este reamintit˘a ˆın [1].) P Astfel, cu utilizarea notat¸iei ment¸ionate pentru Ωn ¸si a simbolului de ˆınsumare , putem sintetiza obt¸inerea rezultatului principal din [1] astfel: (G˘asirea, pe baza formulei de recurent¸˘a Ik =

π Ωn = I2n = 2 Z

π/2

= 0

n X

k

(−1)

Z

Z

π/2

π/2

cos2n x dx = 0

sin

x

Šn

1 − sin2 x

dx =

k=0

n X

‚Z

(−1)

k

n X

(−1)k Cnk I2k =

k=0

Œ

π/2

Cnk

sin

2k

x dx

=

0

k=0

=

dx =

0

! 2k

Cnk

€

n πX (−1)n Cnk Ωk , 2 k=0

adic˘a

n πX π Ωn = (−1)k Cnk Ωk , 2 2 k=0

de unde (3). Putem aplica acum exact aceea¸si idee de calcul pornind de la integrala (4) de ordin impar, I2n+1 . Vom obt¸ine: 1 1 · = I2n+1 = Ωn 2n + 1 t=sin x

Z

1

= = =

Z

Z

π/2 0

€

=

€

1 − sin2 x

0

1−t

2

Šn

Z

n X

1

dt =

0 n X

π/2

cos2n+1 x dx =

0

Z k

(−1)



1 2k

Cnk

t

dt

0

k=0

Šn

t=sin x

cos x dx ===

!

(−1)

k

Cnk t2k

dt =

k=0

=

n X

(−1)k

k=0

Cnk , 2k + 1

adic˘a (6)

n X

(−1)k

k=0

Cnk 1 1 = · . 2k + 1 Ωn 2n + 1

Aceast˘ a identitate, care constituie rezultatul principal al lucr˘arii noastre, nu se mai refer˘a la expresiile Ωk ˆın interiorul sumei, ci cont¸ine pe Ωn doar la rezultat, fiind 102

(spre deosebire de (3), care admite dou˘a interpret˘ari) doar o identitate combinatorial˘a. Dar, din punct de vedere al procedeului de deducere folosit, ea constituie o identitate dual˘a celei din [1], adic˘a (3). ˆIns˘a, ca de multe ori ˆın domeniul identit˘a¸tilor combinatoriale, identitatea g˘asit˘a nu este nou˘a! ˆIntr-adev˘ ar, ˆın cartea lui H. W. Gould [2], la pag. 6, este prezentat˘a identitatea   n X 1 x k n = x+n (−1) (x ∈ R) k x+k n k=0

 

α , pentru α ∈ R ¸si k ∈ N∗ , are semnificat¸ia: k

ˆın care notat¸ia consacrat˘a  

α k

 

cu

α 0

=

α(α − 1)(α − 2) · . . . · (α − k + 1) , 1 · 2 · 3 · ... · k  

def

== 1, iar dac˘a α = n ∈ N ¸si k ≤ n se obt¸in combin˘arile,

n k

= Cnk .

1 , se obt¸ine (6). 2 3. Prin prisma identit˘ a¸tilor combinatoriale, egalitatea (6) ocup˘a pozit¸ia a patra din urm˘atoarea succesiune de formule: Introducˆand ˆın identitatea lui Gould x =

def

n X Cnk

def

n X

Sn ===

k=0

Tn == =

k=0

k+1

X Ck 2n+1 − 1 1 def = (−1)k n = ; Sn(a) == ; n+1 k+1 n+1 n

=

Cnk = f (n); 2k + 1

def = Tn(a) ==

k=0 n X

k=0

(−1)k

Cnk 1 1 = · , 2k + 1 Ωn 2n + 1

unde a desemneaz˘ a o sum˘a alternat˘a, iar f (n) este o expresie neprecizat˘a. (a) G˘asirea sumelor Sn ¸si Sn este binecunoscut˘a (o cale fiind integrarea identit˘a¸tilor (1+t)n =

n X

Cnk tk , respectiv (1−t)n =

k=0 (a)

n X

(−1)k Cnk tk , pe intervalul [0, 1]). Obt¸inerea

k=0

sumei Tn a format obiectul prezentei note, expus ˆın sect¸iunea 2 ¸si astfel, se mai pune problema calculului sumei Tn . O alt˘a problem˘a de studiu ar putea-o constitui calculul unei sume asem˘an˘ atoare cu cea din (3), dar nealternat˘a, anume

n X

Cnk Ωk .

k=0

Bibliografie 1. A. Corduneanu, Gh. Costovici – Un ¸sir strˆ ans legat de ¸sirul lui Wallis, Recreat¸ii Matematice, Anul IX, Nr. 2 iulie-decembrie 2007, 95-96. 2. H. W. Gould – Combinatorial identities, Morgantown Printing and Binding, Co., 1972. 3. R. L. Graham, D. E. Knuth, O. Patashnik – Concrete Mathematics, Addison Wesley Longman, Reading MA, 1994. 103

4. D. E. Knuth – The art of Computer Programming, Vol. 1, Addison Wesley Longman, Reading, MA 1977 (traducerea la Ed. Teora, Bucure¸sti). 5. J. Riordan – Combinatorial Identities, J. Wiley & Sons, New York, 1968. 6. L. T´ oth, A. Vernescu – Dezvoltarea asimptotic˘ a a ¸sirului lui Wallis, G. M.-A, 11 (1990), Nr. 1, 26-29. 7. A. Vernescu – Analiz˘ a matematic˘ a. Probleme. Calcul diferent¸ial, Editura Pantheon, Bucure¸sti, 1991. 8. A. Vernescu – Ordinul de convergent¸˘ a al ¸sirului lui Wallis, G. M.-A, 12 (1991), 7-8. 9. A. Vernescu – Asupra unui tip de relat¸ie de recurent¸˘ a, Revista Matematic˘a din Timi¸soara (seria a IV-a), nr. 4/2003, 8-14. 10. A. Vernescu – S ¸ iruri de Numere Reale, Ed. Univ. Bucure¸sti, 2004. 11. A. Vernescu – Num˘ arul e ¸si matematica exponent¸ialei, Ed. Univ. Bucure¸sti, 2004. 12. A. Vernescu – The natural proof of the inequalities of Wallis type, Libertas Mathematica, 24 (2004), 183-190. 13. A. Vernescu, C. Mortici – New results in discrete asymptotic analysis, General Mathematics, to appear.

(continuare de la pagina 9) M˘ a aflu ˆın posesia unei demonstrat¸ii minunate a acestei afirmat¸ii, dar marginea paginii este prea strˆ amt˘ a pentru a o cuprinde. Care avea s˘a fie o provocare timp de peste 350 de ani pentru multe generat¸ii de matematicieni. Abia ˆın 1995 Andrew Wiles a demonstrat afirmat¸ia, punˆand cap˘at provoc˘arii lui Fermat. Drumul parcurs de lumea matematic˘a pˆan˘a la acest final a fost pres˘arat de tentative de demonstrat¸ie, e¸securi, descoperiri epocale, drame ¸si tragedii individuale, dezv˘aluiri senzat¸ionale ˆın mass-media etc. N-au lipsit nici glumele pe marginea provoc˘arii lui Fermat: ˆIn stat¸ia de metrou de pe Eighth Street, New York a ap˘arut inscript¸ia: xn + y n = z n : nu exist˘ a solut¸ie. Am descoperit o demonstrat¸ie cu adev˘ arat remarcabil˘ a a acestui fapt dar nu pot s-o scriu fiindc˘ a-mi vine trenul. Sau catrenul: ”Pe untul meu sunt multe litere scrise”, Sup˘ arat un client la o mas˘ a r˘ acnise; ”N-am avut loc, r˘ aspunse piccolo-ul Pierre, Nici pe margine, nici pe rafturile din frigider.” (Simon Singh–Marea Teorem˘ a a lui Fermat, Humanitas, Bucure¸sti, 2005.) 104

Asupra problemei C.O: 5004 din G.M. Dan Mihai MOCANU

1

Abstract. In this Note, the problem C.O.: 5004 of the journal G.M. – 2/2009, p.103 is generalized and a couple of simple results in a triangle are established; they deal with isogonal lines and angle trisectors. Keywords: altitudes, angle bisector, isogonal lines, angle trisectors, symmedians. MSC 2000: 51F20.

Scopul prezentei note este de a stabili cˆateva rezultate ˆıntr-un triunghi, legate de cevienele izogonale care sunt ˆın acela¸si timp ¸si trisectoare. Fie ABC un triunghi oarecare ¸si A1 , A2 ∈ (BC). Se spune c˘a cevienele (AA1 , (AA2 sunt izogonale dac˘a Ö1 ≡ CAA Ö2 ¸si c˘ b dac˘ BAA a sunt trisectoare ale unghiului A a Ö × Ö BAA1 ≡ A1 AA2 ≡ A2 AC. Afirmat¸ia urm˘atoare are o demonstrat¸ie imediat˘a.

A

O B

A1

H

..

A2

C

Propozit¸ie. Dac˘ a (AA1 ¸si (AA2 sunt ceviene izogonale ¸si una din ele este bisecb toarea unghiului dintre o latur˘ a ¸si cealalt˘ a, atunci ele sunt trisectoarele unghiului A. Ö Demonstrat¸ie. S˘ a presupunem c˘a (AA1 este bisectoarea unghiului BAA2 (la Ö Ö fel demonstr˘am dac˘a (AA2 ar fi bisectoarea unghiului A 1 AC). Avem, deci, BAA1 ≡ × Ö Ö Ö1 ≡ A si AA2 sunt izogonale), rezult˘a c˘a BAA 1 AA2 . Cum BAA1 ≡ A2 AC (AA1 ¸ × Ö A1 AA2 ≡ A2 AC, ceea ce trebuia demonstrat. ˆ triunghiul ascut¸itunghic Consecint¸a 1. (C.O.: 5004 din G.M.-2/2009, p.102). In Õ ABC se consider˘ a ˆın˘ alt¸imea AH. S˘ a se arate c˘ a, dac˘ a bisectoarea unghiului BAH trece prin centrul cercului circumscris triunghiului ABC, atunci semidreapta (AH Õ este trisectoare a unghiului BAC. Laura Constantinescu Demonstrat¸ie. Se ¸stie c˘a ˆın˘ alt¸imea coborˆat˘a din A ¸si diametrul ce are A ca o extremitate a sa sunt izogonale. Dar, prin ipotez˘a, acest diametru este bisectoarea Õ Conform propozit¸iei precedente (AH este trisectoare a unghiului A). b unghiului BAH. Consecint¸a 2. Dac˘ a AH este ˆın˘ alt¸ime, O este centrul cercului circumscris triÕ atunci (AO unghiului ascut¸itunghic ABC, iar (AH este bisectoarea unghiului OAC, Õ este trisectoare a unghiului BAC. Demonstrat¸ie. Se procedeaz˘a ca ˆın Consecint¸a 1. Observat¸ie. Dac˘a una dintre (AH sau (AO este trisectoare, atunci ¸si cealalt˘a este trisectoare. 1 Elev,

cl. a IX-a, Colegiul Nat¸ional, Ia¸si

105

Urm˘ atoarele rezultate pot fi privite ca reciproce ale consecint¸elor de mai sus. Reciproc˘ a (a Consecint¸ei 1). Dac˘ a (AA2 este o trisectoare a triunghiului ascuÖ2 , atunci A2 este ˆın˘ ¸tit-unghic ABC ¸si (AO este bisectoarea unghiului BAA alt¸ime. Demonstrat¸ie. Din condit¸iile ipotezei, rezult˘a c˘a (AA2 ¸si (AO fac cu laturile A triunghiului unghiuri de m˘asur˘a . Atunci ¸si (AH, ca izogonal˘a a lui (AO, face cu 3 A AC un unghi de m˘asur˘ a . Ca urmare (AA2 coincide cu (AH. 3 Reciproc˘ a (a Consecint¸ei 2). Dac˘ a (AA1 este o trisectoare a triunghiului ascut¸itÖ unghic ABC ¸si (AH este bisectoarea unghiului A 1 AC, atunci O ∈ (AA1 . Demonstrat¸ie. La fel ca pentru reciproca precedent˘a. Observat¸ie. ˆIn loc de ”ˆın˘alt¸ime-diametru”, putem lua o alt˘a pereche de ceviene izogonale; de exemplu, ”median˘a-simedian˘ a”. Trecerea rezultatelor de mai sus la noua pereche se face cu u¸surint¸˘a. ˆIn ˆıncheiere, vom rezolva o problem˘a strˆans legat˘a de cele de mai sus. Problem˘ a. Fie un triunghi ABC, AB > AC. Fie (AM ) mediana relativ la latura (BC) ¸si (AN ) simediana corespunz˘ atoare. Ar˘ atat¸i c˘ a, dac˘ a (AM sau (AN AB A Õ atunci este trisectoare a unghiului BAC, = 2 cos . AC 3 Demonstrat¸ie. Ambele vor fi trisectoare. Cu ajutorul ariilor, avem Ö = AC · AM sin CAM Ö, AB · AM sin BAM

de unde

Ö sin 2A AB A sin CAM 3 = = 2 cos . = A AC 3 Ö sin sin BAM 3

ˆIn toamna anului 1999 a ap˘arut primul num˘ar al revistei Recreat¸ii Matematice. ˆIn anul acesta se ˆımplinesc 10 ani de la aparit¸ia acesteia. Scriet¸i num˘ arul 2009 cu ajutorul numarului 10 ¸si folosind numai operat¸iile de adunare ¸si ˆımp˘ art¸ire ! Care este num˘ arul maxim de operat¸ii cu care putet¸i face acest lucru ? Dar cel minim ? (Nu se accept˘a termeni nuli !) Not˘ a. R˘ aspunsurile la aceste ˆıntreb˘ari sunt date la pag.

106

Matrices ` a coefficients dans un corps fini Adrien REISNER1 Abstract. It is considered the set Ap of matrices of order 2 with their entries in Zp , defined by Ap = {a = λM + µI; λ, µ ∈ Zp }, and some properties of this set are presented (Theorems 1,3,6,7,9). Keywords: unitary ring, the order of an element, the field Zp , isomorphism. MSC 2000: 15A33.

A ´etant un anneau unitaire d′´el´ements neutre e on appelle ordre d′ un ´el´ement inversible a de A le plus petit entier positif n del que an = e; dans ce cas l′ ensemble Ga = {e, a, a2 , . . . , an−1 } forme un sous-groupe du groupe G des ´el´ements inversibles de l′ anneau A: l′ ordre n de l′´el´ement a ∈ G est le cardinal du ce sous-groupe Ga . Pour toute matrice carr´ee M `a coefficients dans un corps on d´esigne par T et ∆ respectivement les deux applications suivantes: T :M →T (M )=trace de la matrice M, ∆ : M → ∆(M )=d´eterminant de la matrice M . p d´esignant un nombre premier strictement sup´erieur ` a 3, on consid`ere le corps fini Zp des classes r´esiduelles modulo p. M et I ´etant les deux matrices suivantes `a coefficients dans Zp :     b b 4 b 1 1 b 0 M= , I= b b , 0 1 −b 1 b 0 on consid`ere l′ ensemble Ap des matrices d′ ordre 2 `a coefficinets dans Zp d´efini par: Ap = {a = λM + µI; λ, µ ∈ Zp }. Th´ eor` eme 1. L′ ensemble Ap est un anneau commutatif unitaire pour les op´erations usuelles. De plus: Card Ap = p2 . D´ emonstration. On a imm´ediatement M 2 = b 4M − I; on en d´eduit compte tenu de la structure de l′ ensemble M2 (Zp ) que Ap est un alg`ebre associative et unitaire. Enfin, la commutativit´e se v´erifie directement. D′ autre part, (I, M ) ´etant une base de l′ alg`ebre Ap , on a Ap ≃ Z2p et par suite: Card Ap = (Card Zp )2 = p2 . La proposition suivante se v´erifie imm´ediatement par le calcul. Proposition 2. Pour toute matrice A = λM + µI ∈ Ap , on a: a) T (A) = b 2(b 2λ + µ), ∆(A) = λ2 + µ2 + b 4λµ; 2 b) T (A ) = T 2 (A) − 2∆(A). En particulier, T (A2 ) = b 2(b 7λ2 + µ2 + b 4λµ). Th´ eor` eme 3. Pour A ∈ Ap , deux quelconques des conditions suivantes impliquent la troisi`eme: a) T (A) = b 0, b) ∆(A) = b 1, c) A est une matrice d′ ordre 4. 1 Centre

de Calcul E.N.S.T., Paris; e-mail: [email protected]

107

D´ emonstration. a) + b) ⇒ c) Le th´eor`eme de Cayley-Hamilton appliqu´ee `a la matrice A donne: A2 − T (A)A + ∆(A)I = O. Il vient alors avec les hypoth`eses a) et b): A2 = −I et A4 = I. L′ ordre de la matrice A devise donc 4 et comme A2 ̸= I (p ´etant impair), 2 n′ est pas multiple de cet ordre. Finalement, A est d′ ordre 4. a) + c) ⇒ b) Supposant les conditions a) et c) v´erifi´ees, on a: A2 = −∆(A)I et I = A4 = ∆2 (A)I. On en d´eduit: ∆2 (A) = b 1, ∆(A) ̸= −b 1 soit ∆(A) = b 1. b) + c) ⇒ a) Compte tenu de b) et c), A2 = T (A)A − I d′ o` u I = A4 = T 2 (A)A2 − b 2T (A)A + I = T (A)[T 2 (A) − b 2]A + [b 1 − T 2 (A)]I. On peut distinguer deux cas: I A et I sout li´es, i.e. A = µI. Dans ce cas, A2 = µ2 I, A4 = µ4 I, d′ o´ u, A ´etant d′ ordre 4: µ2 ̸= b 1, µ4 = b 1, µ2 = −b 1 et A2 = −I, T (A)A = 0. ′ b b Si T (A) ̸= 0, alors A = O d o` u T (A) = 0, impossible. Par suite T (A) = b 0. II {A, I} est une famille libre. Dans ce cas l′´egalit´e I = T (A)[T 2 (A) − b 2]A + [b 1 − T 2 (A)]I implique: b 1=b 1 − T 2 (A) d′ o´ u T (A) = b 0. Le th´eor`eme est ainsi d´emontr´e. On consid`ere la suite des entiers {Yk }k≥0 d´efinie par Y0 = 2 et Yk+1 = 2Yk2 − 1, k ≥ 0, dont les premiers terms sont: Y0 = 2, Y1 = 7, Y2 = 97, Y3 = 18817, . . . k

Th´ eor` eme 4. Pour tout k ≥ 0, on a 2Yk ∈ T (M 2 ). D´ emonstration. Par r´ecurrence sur k. Pour k = 0 la propri´et´e se v´erifie trivialment puisque 2Y0 = 4 ∈ b 4 = T (M ). Supposant la propri´et´e v´erifi´ee `a l′ ordre k, d´emontrons-l` a pour k + 1. On a imm´ediatment d′ une part: 2Yk+1 = 4Yk2 − 2 = k+1 k k (2Yk )2 − 2 et d′ autre part T (M 2 ) = T [(M 2 )2 ] = T 2 (M 2 ) − b 2, compte tenu de l′ assertion b) de la Proposition 2 et puisque ∆(M ) = b 1. Donc, compte tenu de k+1 l′ hypoth`ese de r´ecurrence: 2Yk+1 = (2Yk )2 − 2 ∈ T (M 2 ). Th´ eor` eme 5. La matrice M est d′ ordre 2k si et seulement si p|Yk−2 . k D´ emonstration. Supposons la matrice M d′ ordre 2k , i.e. M 2 = I. On a:   c b 15 4 M2 = et par suite l′ ordre de M ne divise pas 2 soit k ≥ 2. En posant −b 4 −b 1 k−2 k−1 k alors A = M 2 on a, M ´etant d′ ordre 2k : A2 = M 2 ̸= I, A4 = M 2 = I. A ∈ Ap est donc une matrice d′ ordre 4 v´erifiant aussi ∆(A) = b 1. Compte tenu du th´eor`eme 3, b) + c) ⇒ a), il vient alors: T (A) = b 0 soit d′ apr`es le th´eor`eme pr´ec´edent: 2Yk−2 ∈ T (A) = b 0, i.e. p divise 2Yk−2 et finalement p ´etant impair p|Yk−2 . R´eciproquement, avec les mˆemes notations si p|Yk−2 i.e. Yk−2 ≡ 0 (mod p), alors T (A) = b 0. Mais, comme ∆(A) = b 1, le th´eor`eme 3, a) + b) ⇒ c), montre que la matrice k k−1 A est d′ ordre 4 soit M 2 = I et M 2 = A2 ̸= I. L′ ordre de M divisant 2k mais non 2k−1 est donc ´egal `a 2k . 108

Th´ eor` eme 6. Les deux assertions suivantes sont ´equivalentes: ′ b a) 3 n est pas le carr´e d′ un ´el´ement de Zp ; b) Ap est un corps. D´ emonstration. a) ⇒ b) Si b 3 n′ est pas le carr´e d′ un ´el´ement de Zp , alors pour A ∈ Ap : ∆(A) = b 0 ⇒ A = O. En effet, supposons ∆(A) = b 0. Pour A ∈ Ap on a: ∆(A) = b 0 = (λ + b 2µ)2 − b 3µ2 . −1 2 µ ̸= b 0 impliquerait b 3 = (λµ + b 2) ce qui est exclut par hypoth`ese. Donc µ = b 0 et ∆(A) = b 0 = λ2 soit: λ = b 0 et finalement: A = O. Donc l′ ensemble Ap est form´e de la matrice nulle et de matrices inversibles dans M2 (Zp ). A ̸= O ´etant une matrice de Ap , l′ application Ap → Ap d´efinie par X → AX est lin´eaire et injective, donc surjective (dim Ap = 2). Pour A ̸= O il existe B ∈ Ap telle que: AB = 1 soit A−1 = B ∈ Ap : les inverse des matrices non nulles de Ap sont dans Ap . b) ⇒ a) Nous allons montrer que, non a) ⇒ non b). Supposons qu′ il existe a ∈ Zp tel que a2 = b 3. Dans ce cas: ∆(A) = [λ + (b 2 − a)µ][λ + (b 2 + a)µ]. Pour la matrice 2 b b b b b A = (a − 2)M + I(a ̸= 2 puisque 2 − 3 = 1) on a ∆(A) = b 0 et (Cayley-Hamilton) A(A − T (A)I) = O avec A ̸= O et A ̸= T (A)I. On en d´eduit que Ap n′ est pas int`egre et par suite que Ap n′ est pas un corps. Th´ eor` eme 7. En supposant que b 3 est un carr´e dans Zp : a) M est semblable ` a une matrice diagonale; b) Ap est isomorphe ` a l′ anneau produit Zp × Zp ; c) dans Ap il y a p − 1 ´el´ements de d´eterminant b 1 et (p − 1)2 ´el´ements inversibles. D´ emonstration. a) L′´equation caract´eristique de la matrice M s′´ecrit X 2 − b 4X + b 1 = (x − b 2)2 − a2 = 0 o` u a est tel que a2 = b 3. M ayant deux valeurs propre distinctes λ1 = b 2 + a et λ2 = b 2 − a, on en d´ eduit que la matrice M est diagonalisable i.e. il   b b 2+a 0 −1 P . existe P ∈ GL2 (Zp ) telle que M = b b 0 2−a b) L′ application R → P −1 RP qui est un automorphisme pour la structure d′ anneau, transforme toute matrice A = λM +µI de Ap en une matrice diagonale. Or l′ ensemble Dp des matrices diagonales de M2 (Zp ) ayant comme cardinal p2 (je rappelle que Card Ap = p2 ), l′ application Ap → Dp , A → P −1 AP est donc un isomorphisme.   α b 0 D′ autre part, l′ application Zp × Zp → Dp , (α, β) → b , ´etant de fa¸con 0 β imm´ediate un isomorphism, on en d´eduit que: Ap ≃ Zp × Zp (cet isomorphisme est mˆeme un isomorphisme d′ alg`ebre). c) Compte tenu de l′ isomorphisme pr´ec´edent, comme ∆(A) = ∆(P −1 AP ), le nombre de matrices A ∈ Ap telles que ∆(A) = b 1 est ´egal aux nombre de couple (α, β) v´erifiant αβ = b 1. Il y en a p − 1 (choisir d′ abord α). La d´emonstration de l′ implication a) ⇒ b) du Th´eoreme 6 a montr´e que ∆(A) ̸= b 0 ⇒ A−1 ∈ Ap . On en d´eduit que le nombre de matrices A inversibles de Ap (i.e. le 109

nombre des matrices A ∈ Ap telles que ∆(A) ̸= b 0) est ´egal aux nombre des couples 2 b (α, β) ∈ Zp × Zp v´erifiant αβ = 0 soit (p − 1) . Corollaire 8. Dans le cas o` ub 3 est un carr´e dans Zp : ′ a) l ordre de la matrice M divise p − 1; b) si p|Yk−2 , alors 2k |p − 1. D´ emonstration. a) L′ ensemble des matrices de Ap de d´eterminant b 1 forment un sous-groupe multiplicatif de A∗p , groupe des matrices inversibles de Ap (si A ∈ Ap et ∆(A) ̸= b 0, alors A−1 ∈ Ap –voir a) ⇒ b) du th´eor`eme 6). Compte tenu de l′ assertion c) du th´eor`eme pr´ec´edent ce sous-groupe est fini de cardinal p − 1. Or la matrice M appartient `a ce sous-groupe. L′ ordre de M divise par suite p − 1 (ordre de ce sous-groupe). b) Si p divise Yk−2 , alors la matrice M est d′ ordre 2k d′ apr´es le Th´eor`eme 5. ′ L assertion a) de ce corollaire permet alors de conclure. Th´ eor` eme 9. En supposant que b 3 n′ est pas un carr´e dans Zp : a) ∆ est un homomorphisme du groupe multiplicatif des ´el´ements non nuls de Ap dans celui des ´el´ements non nuls de Zp ; b) il existe k tel que p − 1 = Card (Im ∆) et Card Ker∆ = k(p + 1); c) il existent p + 1 ´el´ements de d´eterminant b 1 dans Ap . D´ emonstration. a) Ap ´etant un corps d′ apr`es le Th´eor`eme 6, l′ assertion a) est ´evidente. b) On en d´eduit l′ isomorphisme Im ∆ ≃ A∗p /Ker∆ et par suite: Card A∗p = p2 −1 = (Card Im ∆)(Card Ker ∆). De plus, Im ∆ est un sous-groupe de Z∗p : il existe k tel que p − 1 = k Card (Im ∆), d′ o` u Card Ker ∆ = k(p + 1). Notons que 1 ≤ k ≤ p − 1. c) Les matrices A de Ker ∆ v´erifient ∆(A) = b 1. Il s′ agit de montrer que Card ′ b Ker ∆ = p + 1. L ´egalit´e ∆(A) = 1 entraˆıne, compte tenu de l′ assertion a) de la Proposition 2: λ2 + µ2 + b 4λµ − b 1=b 0 (1). λ ∈ Zp ´etant donn´e, il existe donc 0, 1 ou 2 ´el´ements µ ∈ Zp tels que ∆(A) = b 1 (Zp ´etant un corps), donc il existe au plus 2p couples (λ, µ) v´erifiant l′´equation (1). D′ autre part, le nombre de tels couples est ´egal `a-voir b)-k(p + 1) = Card Ker ∆. On en d´eduit finalement que k = 1 et par suite Card Ker ∆ = p + 1. Corollaire 10. Dans le cas o` ub 3 n′ est pas un carr´e dans Zp : ′ a) l ordre de la matrice M divise p + 1; b) si p|Yk−2 , alors 2k |p + 1. D´ emonstration. a) La matrice M de d´eterminant b 1 appartient au sous-groupe Ker ∆. L′ ordre de M divise par suite l′ ordre p + 1 de ce sous-groupe Ker ∆ -voir l′ assertion c) du Th´eor`eme 9. b) De mˆeme qu′ au Corollaire 8, si p divise Yk−2 , alors la matrice M est d′ ordre k ′ 2 d apr´es le Th´eor`eme 5. L′ assertion a) de ce corollaire permet alors de conclure. Remarque. Dans le cas o` ub 3 n′ est pas un carr´e dans Zp , l′ ordre de toute matrice b A de Ap v´erifiant ∆(A) = 1 divise p + 1 - ordre de sous-groupe Ker ∆. Par suite ∀A ∈ Ap : Ap+1 = I. 110

O metod˘ a de rezolvare a problemelor Maria MIHET ¸1 ˆIn aceast˘a not˘a vom evident¸ia o strategie important˘a de rezolvare a problemelor: demonstrat¸ia prin exprimarea ˆın dou˘a moduri a unor m˘arimi. Aceast˘a metod˘a s-a folosit ˆınc˘ a ˆın primele clase de gimnaziu, pentru obt¸inerea unor ecuat¸ii. Am ˆıntˆalnit-o, de asemenea, ˆın demonstrat¸ia formulei pentru suma unghiurilor unui poligon convex: cu ajutorul unui punct O din interiorul poligonului se trianguleaz˘a poligonul ¸si se exprim˘a ˆın dou˘a moduri suma unghiurilor triunghiurilor obt¸inute. Prin exprimarea ˆın dou˘a moduri a ariei sau volumului se pot afla anumite distant¸e (ne amintim, de exemplu, cum se poate afla ˆın˘ alt¸imea din vˆarful unghiului drept ˆıntr-un triunghi dreptunghic prin exprimarea ˆın dou˘a moduri a ariei triunghiului), iar multe identit˘a¸ti combinatoriale pot fi demonstrate folosind num˘ararea ˆın dou˘a moduri. Am exemplificat metoda prin cˆateva probleme tip, iar la sfˆar¸situl lucr˘arii am ˆıntocmit o list˘a de probleme ˆınsot¸ite de indicat¸ii de rezolvare. Am dori ca cititorii s˘a ˆıncerce mai ˆıntˆıi s˘a le rezolve f˘ar˘ a a apela la indicat¸ii. Aceast˘a not˘a se adreseaz˘a ˆın special elevilor din clasele VI-VIII. Ea poate fi ˆıns˘a completat˘ a cu multe exemple de nivel liceal. Problema 1. Fie q un num˘ ar real, iar n ∈ N, n ≥ 2. S˘ a se calculeze, ˆın funct¸ie de q, suma Sn = 1 + q + q 2 + . . . + q n . Solut¸ie. Exprim˘am Sn+1 ˆın dou˘a moduri: mai ˆıntˆai Sn+1 = Sn + q n+1 , iar, pe de alt˘a parte, Sn+1 = qSn + 1. Rezult˘a c˘a Sn + q n+1 = qSn + 1, de unde obt¸inem c˘a (q − 1)Sn = q n+1 − 1.

(∗)

Astfel, dac˘a q ̸= 1, atunci Sn = obt¸ine direct c˘a Sn = n + 1. A¸sadar,

q n+1 − 1 . Dac˘a q = 1, prin ˆınlocuire ˆın Sn se q−1

8 AC vedet¸i u¸sor c˘a intersect¸ia M este astfel ˆıncˆ at C se afl˘a ˆıntre B ¸si M . Vom numi atunci bisectoare exterioar˘ a din A a triunghiului ABC segmentul [AM ]. E clar c˘a, dac˘a AB = AC, bisectoarea exterioar˘a nu poate fi considerat˘a ˆın sensul acestei definit¸ii. Nu mult dup˘a aceea, sau chiar deodat˘a, elevii afl˘a c˘a ¸si reciprocele acestor afirmat¸ii sunt adev˘arate. Dar, g˘asindu-se la ˆınceputul studiului geometriei, le va fi mult mai greu s˘a le justifice. Faptul c˘a un triunghi cu dou˘a mediane congruente (sau cu dou˘a ˆın˘alt¸imi congruente) este isoscel nu prea poate fi demonstrat cu una, cu dou˘a, deoarece necesit˘a cazuri speciale de congruent¸˘ a. Iar despre a ar˘ata c˘a un triunghi cu dou˘a bisectoare congruente este isoscel nici nu poate fi vorba atˆata vreme cˆat el face obiectul unei renumite teoreme care a pus la grea ˆıncercare chiar mari matematicieni. Dar este adev˘arat, ceea ce nu se mai poate afirma ˆın cazul bisectoarelor exterioare (¸si despre asta vom vorbi mai departe). Profesorul C.L. Lehmus, ˆın 1840, a cerut pentru prima dat˘a o solut¸ie sintetic˘ a a problemei, care a fost g˘asit˘a, printre alt¸ii, de marele geometru Jacob Steiner (se pare c˘a Lehmus ˆınsu¸si avea s˘a g˘aseasc˘a ulterior o demonstrat¸ie); 1 Profesori,

Colegiul Nat¸ional ”Gheorghe Ro¸sca Codreanu”, Bˆ arlad

117

teorema a r˘amas cunoscut˘a cu aceste nume: teorema Steiner-Lehmus. Mai multe despre istoria ei ¸si demonstrat¸ia ˆın cadrul geometriei euclidiene clasice se g˘asesc, de exemplu, pe internet la adresa http://forumgeom.fau.edu/FG2005volume5/FG200525.pdf [2] (ca s˘a d˘am un exemplu la ˆındemˆana cititorilor mai tineri; altminteri, orice carte serioas˘a de geometrie euclidian˘a elementar˘a cuprinde ¸si aceast˘a teorem˘a). Noi vom ˆıncepe cu o demonstrat¸ie prin calcul a acestui renumit rezultat, probabil foarte cunoscut˘a cititorilor. Vom considera un triunghi oarecare ABC ˆın care vom folosi notat¸iile obi¸snuite: a, b, c sunt lungimile laturilor (opuse vˆarfurilor A, B, respectiv C), la este lungimea bisectoarei obi¸snuite (interioare) a unghiului cu vˆarful ˆın A. Formula a2 bc la2 = bc − (b + c)2 care furnizeaz˘a lungimea acestei bisectoare este notorie, nu mai insist˘am asupra ei. Ca ¸si asupra urm˘atorului calcul, pe care-l recomand˘am celor ce nu-l cunosc ˆınc˘a. Exercit¸iul 1. S˘ a se arate c˘a, cu notat¸iile de mai sus, avem 

la2 − lb2 ab(a2 + b2 + c2 + ab + 2ac + 2bc) = (b − a) 1 + c (a + c)2 (b + c)2



.

Evident, de aici se obt¸ine imediat teorema Steiner-Lehmus ¸si chiar mai mult, se vede c˘a avem la < lb ⇔ a > b (deoarece expresia din parantez˘a este strict pozitiv˘a). Aceast˘a proprietate este valabil˘a ¸si pentru mediane sau ˆın˘alt¸imi, adic˘a ma < mb ⇔ a > b ¸si ha < hb ⇔ a > b (dac˘a not˘am ma ¸si ha lungimea medianei, respectiv ˆın˘ alt¸imii din A), dar vom vedea c˘a pentru bisectoarele exterioare nu este cˆatu¸si de put¸in adev˘arat˘ a. Colateral, v˘a putet¸i ocupa ¸si de Exercit¸iul 2. Demonstrat¸i c˘a ma < mb ⇔ a > b ¸si ha < hb ⇔ a > b. Acum s˘a vorbim ¸si despre bisectoarele exterioare. Vom not˘a cu ea lungimea bisectoarei exterioare din A (dac˘a b ̸= c). Formula e2a =

a2 bc − bc (b − c)2

este ¸si ea destul de cunoscut˘a (dac˘a n-o ¸stit¸i, o putet¸i obt¸ine fie din relat¸ia lui Stewart, fie dintr-un calcul cu arii, folosind teorema bisectoarei exterioare pentru a exprima ˆın funct¸ie de a, b, c lungimile segmentelor determinate pe [BC] de bisectoare, respectiv folosind expresia lui cos(A/2)). Cu ajutorul ei obt¸inem imediat, la fel ca mai sus 

e2a − e2b ab(a2 + b2 + c2 + ab − 2ac − 2bc) = (a − b) 1 + c (a − c)2 (b − c)2



.

Desigur, putem considera ea ¸si eb (¸si calcula aceast˘a expresie) numai ˆıntr-un triunghi neisoscel, deci mai departe presupunem c˘a oricare dou˘a dintre a, b, c sunt distincte. Exercit¸iul 3. Continuat¸i acest calcul pentru a obt¸ine (b − a)(a + b − c)(c3 − (a + b)c2 + 3abc − ab(a + b)) e2a − e2b = . c (a − c)2 (b − c)2 118

E clar acum c˘a nu avem cum s˘a obt¸inem egalitatea laturilor din egalitatea bisectoarelor exterioare, de¸si acesta este unul din cazuri: dac˘a presupunem ea = eb rezult˘a c˘a unul din factorii de la num˘ ar˘ ator trebuie s˘a fie zero. Dintre ace¸sti factori doar de a + b − c putem fi siguri c˘a este nenul (pozitiv pentru a, b, c lungimile laturilor unui triunghi). R˘amˆ ane totu¸si al treilea factor, care ne permite s˘a demonstr˘am urm˘atoarea Propozit¸ie. Pentru orice numere reale pozitive ¸si distincte a ¸si b exist˘ a c pozitiv ¸si diferit de a ¸si de b astfel ˆıncˆ at a, b, c pot fi laturile unui triunghi, iar ˆın acest triunghi ea = eb . Demonstrat¸ie. S˘a presupunem, de exemplu, c˘a a < b. Pentru funct¸ia continu˘a f definit˘ a prin f (x) = x3 − (a + b)x2 + 3abx − ab(a + b), ∀x ∈ R, avem f (b − a) = −2a3 < 0 ¸si f (b) = ab(b − a) > 0, deci exist˘a c ∈ (b − a, b) astfel ˆıncˆat f (c) = 0. Acest c este diferit ¸si de a, deoarece f (a) = ab(a − b) < 0; pentru c˘a b > a, b > c ¸si c > b − a implic˘ a b < a + c, triunghiul cu laturile a, b ¸si c exist˘a ¸si este neisoscel. Exprimarea de mai sus a diferent¸ei e2a − e2b ne arat˘a c˘a, ˆın acest triunghi, ea = eb , ceea ce ˆıncheie demonstrat¸ia. Se mai observ˘a c˘a semnul acestei diferent¸e nu poate fi stabilit numai ˆın funct¸ie de semnul lui b − a, de aceea un analog al propriet˘a¸tii la < lb ⇒ a > b nu funct¸ioneaz˘a pentru bisectoarele exterioare. ˆIn plus, de¸si tocmai am ar˘atat c˘a exist˘a triunghiuri neisoscele cu dou˘a bisectoare exterioare congruente, cititorul va vedea u¸sor c˘a nu se poate ca toate bisectoarele exterioare s˘a aib˘a lungimi egale. Exercit¸iul 4. S˘a se arate c˘a ˆıntr-un triunghi neisoscel, nu se poate ca toate cele trei bisectoare exterioare s˘a fie congruente. ˆIn sfˆar¸sit, s˘a ne mai punem o ˆıntrebare fireasc˘a: putem da un exemplu concret de asemenea triunghi (neisoscel, dar care are dou˘a bisectoare exterioare congruente)? C˘aci una e s˘a spui: exist˘a un astfel de triunghi, ¸si alta e dac˘a afirmi r˘aspicat c˘a triunghiul cu laturile de anumite lungimi (bine precizate) are respectiva proprietate. Avem un asemenea exemplu! √ √ Exercit¸iul 5. S˘a se arate c˘a ˆın triunghiul cu laturile a = 5 − 7, b = 5 + 7 ¸si c = 6 avem ea = eb . Mai mult, putem g˘asi toate triunghiurile cu aceast˘a proprietate. Propozit¸ie. Un triunghi neisoscel ABC are proprietatea ea = eb dac˘ a ¸si numai dac˘ a exist˘ a un num˘ ar p din intervalul (0, 1) ¸si un num˘ ar pozitiv c astfel ˆıncˆ at lungimile laturilor sale s˘ a fie date de   È c BC = 3p + 1 − (1 − p)(p2 + 3p + 4) , 2(p + 1)

  È c 3p + 1 + (1 − p)(p2 + 3p + 4) ¸si AB = c. 2(p + 1) Demonstrat¸ie. Desigur, ca s˘a avem ea = eb ˆıntr-un triunghi neisoscel trebuie ca ultimul factor de la num˘ ar˘ atorul expresiei e2a − e2b s˘a fie zero, adic˘a trebuie s˘a aib˘a loc egalitatea c3 − (a + b)c2 + 3abc − ab(a + b) = 0.

AC =

119

Cu notat¸iile x = a/c ¸si y = b/c aceasta se mai scrie (dup˘a ˆımp˘art¸irea cu c3 ): 1 − (x + y) + 3xy − xy(x + y) = 0 ⇔ 1 − s + 3p − sp = 0, dac˘a punem s = x + y ¸si p = xy. Desigur, x ¸si y sunt solut¸iile ecuat¸iei de gradul al doilea t2 − st + p = 0, iar dac˘a ¸stim pe x ¸si pe y, cunoa¸stem ¸si laturile triunghiului: BC = cx, AC = cy ¸si BC = c. Relat¸ia de mai sus ne furnizeaz˘a s = (3p + 1)/(p + 1) ¸si un calcul rapid ne arat˘a c˘a discriminantul ecuat¸iei de gradul al doilea este atunci 

∆ = s2 − 4p =

3p + 1 p+1

‹2

− 4p =

(1 − p)(4p2 + 3p + 1) (p + 1)2

care va fi pozitiv dac˘a ¸si numai dac˘a p ∈ (0, 1) (de la bun ˆınceput e clar c˘a p trebuie s˘a fie pozitiv, iar din ∆ > 0 rezult˘a c˘a p < 1). Dac˘a ∆ > 0, ecuat¸ia va avea dou˘a solut¸ii pozitive ¸si distincte care conduc exact la formulele din enunt¸ pentru lungimile laturilor. Interesant este c˘a aceste valori ale lui x ¸si y produc ˆıntotdeauna (f˘ar˘ a s˘a fie nevoie de vreo condit¸ie suplimentar˘a) lungimile laturilor unui triunghi (ˆın care, credem c˘a e foarte clar, ea = eb ). ˆIntr-adev˘ar, ordinea lor se vede c˘a este a < c < b, ceea ce corespunde inegalit˘a¸tilor x < 1 < y (care rezult˘a repede din (x − 1)(y − 1) = p(p − 1)/(p + 1) < 0), caz ˆın care mai trebuie s˘a verific˘am doar b < a + c ⇔ b − a < c ⇔ y − x < 1. Dar y−x=



∆=1−

4p3 b care au loc ¸si pentru celelalte linii importante obi¸snuite ale triunghiului) nu se p˘astreaz˘a deloc pentru bisectoarele exterioare. L˘as˘ am ˆın seama cititorului interesat (¸si mai bun prieten cu geometria euclidian˘a clasic˘a) s˘a explice pur geometric aceste fapte aparent stranii. Ment¸ion˘am c˘a existent¸a triunghiurilor neisoscele cu dou˘a bisectoare exterioare congruente e cunoscut˘a de mult: de exemplu, se vorbe¸ste despre ele ˆın [1], la pagina 235 (unde sunt numite triunghiuri pseudoisoscele). Bibliografie 1. F. G.-M. – Exercices de g´eom´etrie, 1912. 2. K.R.S. Sastry – A Gergonne Analogue of the Steiner - Lehmus Theorem, Forum Geometricorum, 5 (2005), 191-195. 120

Metoda falsei ipoteze - variante de utilizare ˘ D.M. BATINET ¸ U-GIURGIU 1 ˆIn clasele primare elevii ˆıntˆ alnesc m˘arimi care sunt dependente una de alta. De exemplu, dac˘a ˆın produsul F1 · F2 cre¸ste (descre¸ste) unul din factori de un num˘ar de ori, acela¸si lucru se ˆıntˆ ampl˘ a ¸si cu produsul, iar dac˘a unul din factori cre¸ste (descre¸ste) de un num˘ ar de ori ¸si produsul r˘amˆ ane neschimbat, atunci cel˘alalt factor descre¸ste (cre¸ste) de acela¸si num˘ ar de ori. Problemele ˆın care apar astfel de m˘arimi se rezolv˘a uneori cu metoda falsei ipoteze, care comport˘a parcurgerea urm˘atoarelor etape: 1) se face o ipotez˘a arbitrar˘a asupra m˘arimilor de aflat, ce va fi, ˆın general, ˆın contradict¸ie cu datele problemei; 2) se analizeaz˘a sursele e¸secului ¸si erorii comise ¸si se trag concluziile corespunz˘atoare; 3) se modific˘a ipoteza (adic˘a valorile date arbitrar m˘arimilor) pe baza concluziilor punctului precedent, ˆın scopul obt¸inerii solut¸iei; 4) se verific˘a corectitudinea solut¸iei g˘asite. Deoarece ˆın programa claselor primare nu mai este inclus˘a aceast˘a metod˘a, d˘am un exempul de problem˘a pe care vom ilustra aceast˘a cale de rezolvare. Problem˘ a. Un produc˘ ator a vˆ andut la piat¸˘ a 1200 kg mere de 2 lei/kg, 3 lei/kg ¸si 4 lei/kg, pe care a ˆıncasat 3300 lei. S ¸ tiind c˘ a produc˘ atorul a avut cantit˘ a¸ti egale de mere de 2 lei/kg ¸si 3 lei/kg, s˘ a se afle cˆ ate kilograme de mere de 2 lei/kg, 3 lei/kg ¸si 4 lei/kg a vˆ andut produc˘ atorul. Prezent˘ am ¸sase variante de rezolvare, utilizˆand de fiecare dat˘a metoda falsei ipoteze, cu o ipotez˘a sau cu dou˘a ipoteze arbitrare. Varianta 1. Presupunem c˘a toate merele sunt de 4 lei/kg. ˆIn aceast˘a ipotez˘a se ˆıncaseaz˘ a 4 × 1200 = 4800 lei. Suma real˘a este dep˘a¸sit˘a cu 4800 − 3300 = 1500 lei. Aceast˘a dep˘a¸sire apare din faptul c˘a am ˆınlocuit x kilograme mere de 2 lei/kg ¸si x kilograme mere de 3 lei/kg cu 2x kilograme mere de 4 lei/kg. ˆInlocuind 1 kg de mere de 2 lei/kg ¸si 1 kg de mere de 3 lei/kg cu 2 kg mere de 4 lei/kg, suma real˘a se m˘are¸ste cu 8 − (2 + 3) = 3 lei. Avem un num˘ar de 1500 : 3 = 500 ˆınlocuiri, ceea ce ˆınseamn˘ a c˘a avem 500 kg mere de 2 lei/kg, 500 kg de mere de 3 lei/kg ¸si 200 kg mere de 4 lei/kg. Se verific˘ a u¸sor c˘a aceste cantit˘a¸ti ˆındeplinesc condit¸iile problemei: 500 × 2 + 500 × 2 + 200 × 4 = 3300 lei. Varianta 2. Presupunem c˘a avem numai mere de 2 lei/kg ¸si de 3 lei/kg. ˆIn aceast˘a ipotez˘a suma ˆıncasat˘ a de produc˘ator este 600 × 2 + 600 × 3 = 3000 lei. Diferent¸a dintre suma real˘a ¸si cea din presupunere este 3300 − 3000 = 300 lei. Aceast˘a diferent¸˘a 1 Profesor,

Colegiul Nat¸ional ”Matei Basarab”, Bucure¸sti

121

provine din faptul c˘a, de x ori am ˆınlocuit cˆate 2 kg de mere de 4 lei/kg cu 1 kg de mere de 2 lei/kg ¸si 1 kg de mere de 3 lei/kg, adic˘a de cˆate ori se cuprinde (8 − 2 − 3) ˆın 300, de unde afl˘am c˘a x = 100. ˆInseamn˘a c˘a produc˘atorul a vˆandut 2 × 100 = 200 kg mere de 4 lei/kg, 500 kg de mere de 2 lei/kg ¸si 500 kg de mere de 3 lei/kg. Verificarea se face la fel. Varianta 3. Presupunem c˘a cele trei categorii de mere sunt ˆın cantit˘a¸ti egale. ˆIn aceast˘a ipotez˘a suma ˆıncasat˘a este 400 × 2 + 400 × 3 + 400 × 4 = 3600 lei. Diferent¸a dintre suma presupus˘a ¸si cea real˘a este 3600 − 3300 = 300 lei. Evident, cantit˘a¸tile nu pot fi egale. Trebuie s˘a ˆınlocuim de x ori cˆate 2 kg mere de 4 lei/kg cu 1 kg de mere de 2 lei/kg ¸si 1 kg de mere de 3 lei/kg. Obt¸inem x = 300 : (8 − 2 − 3) = 100, ceea ce ˆınseamn˘ a c˘a trebuie s˘a mic¸sor˘am cantitatea de mere de 4 lei/kg cu 2 × 100 = 200 kg. Am aflat c˘a produc˘atorul a vˆandut 200 kg mere de 4 lei/kg, 500 kg mere de 2 lei/kg ¸si 500 kg mere de 3 lei/kg. Varianta 4. Observ˘am ˆıntˆai c˘a produc˘atorul a vˆandut cantit˘a¸ti de mere exprimate prin numere naturale pare. S˘a presupunem c˘a produc˘atorul a vˆandut 500 kg mere de 4 lei/kg. ˆIn acest caz suma ˆıncasat˘a este 350 × 2 + 350 × 3 + 500 × 4 = 3750 lei. Diferent¸a dintre suma din presupunere ¸si cea real˘a este 3750 − 3300 = 450 lei. Evident, cantitatea de mere de 4 lei/kg trebuie mic¸sorat˘a. ˆInlocuim de x ori cˆate 2 kg de mere de 4 lei/kg cu 1 kg de mere de 2 lei/kg ¸si 1 kg mere de 3 lei/kg, diminuˆand suma presupus˘a cu 8−(2+3) = 3 lei. Cantitatea de mere de 4 lei/kg trebuie mic¸sorat˘a cu 450 : 3 × 2 = 300 kg, ceea ce ˆınseamn˘a c˘a produc˘atorul a vˆandut 200 kg mere de 4 lei/kg, 500 kg mere de 2 lei/kg ¸si 500 kg mere de 3 lei/kg. Varianta 5. Presupunem c˘a produc˘atorul a vˆandut 300 kg mere de 2 lei/kg, 300 kg mere de 3 lei/kg ¸si 600 kg mere de 4 lei/kg. ˆIn aceast˘a ipotez˘a produc˘atorul ˆıncaseaz˘ a 300 × 2 + 300 × 3 + 600 × 4 = 3900 lei. Diferent¸a dintre suma presupus˘a ¸si cea real˘a este de 3900 − 3300 = 600. Trebuie s˘a ˆınlocuim de x ori cˆate 2 kg de mere de 4 lei/kg cu 1 kg mere de 2 lei/kg ¸si 1 kg mere de 3 lei/kg, iar suma ˆıncasat˘a ˆın aceast˘a ipotez˘a se mic¸soreaz˘a cu 8 − (2 + 3) = 3 lei. Cantitatea de mere de 4 lei/kg trebuie mic¸sorat˘ a cu 600 : 3 × 2 = 400 kg, ceea ce ˆınseamn˘a c˘a produc˘atorul a vˆandut 200 kg mere de 4 lei/kg, 500 kg mere de 2 lei/kg ¸si 500 kg mere de 3 lei/kg. Varianta 6. Prima ipotez˘a: produc˘atorul a vˆandut 320 kg mere de 2 lei/kg, 320 kg mere de 3 lei/kg ¸si 560 kg mere de 4 lei/kg. Suma ˆıncasat˘a ˆın prima ipotez˘a este 320 × 2 + 320 × 3 + 560 × 4 = 3840 lei. Ipoteza a doua: produc˘atorul a vˆandut 340 kg mere de 2 lei/kg, 340 kg mere de 3 lei/kg ¸si 520 kg mere de 4 lei/kg. ˆIn aceast˘a ipotez˘a suma ˆıncasat˘ a este 340 × 2 + 340 × 3 + 520 × 4 = 3780 lei. Se observ˘a c˘a la o cre¸stere de 40 kg mere de 4 lei/kg, prima sum˘a presupus˘a se mic¸soreaz˘a cu 3840 − 3780 = 60 lei. Prima sum˘a presupus˘a trebuie s˘a scad˘a cu 3840 − 3300 = 540 lei. Cantitatea de 560 kg mere de 4 lei/kg trebuie s˘a scad˘a cu 540 : 60 × 40 = 360 kg, ceea ce ˆınseamn˘a c˘a produc˘atorul a vˆandut 200 kg mere de 4 lei/kg, 500 kg mere de 2 lei/kg ¸si 500 kg mere de 3 lei/kg. Verificarea este imediat˘a.

122

Liceul Teoretic ”Garabet Ibr˘ aileanu”1 La 110 ani de la ˆınfiint¸are

La prima vedere Liceul Teoretic ”Garabet Ibr˘ aileanu”, cu arhitectura sa u¸sor confundabil˘ a cu aceea a ¸scolilor ridicate ˆın ”epoca de aur”, pare o institut¸ie ¸scolar˘a de dat˘a recent˘ a. Nimic mai fals. Liceul acesta are mai bine de 100 de ani de la ˆınfiint¸are ¸si istoria sa reflect˘a convulsiile prin care a trecut societatea romˆaneasc˘a, drama intelectualit˘ a¸tii ¸si chiar ”drama psihologiei” pentru a prelua titlul unei c˘art¸i de Vasile Pavelcu, fost profesor la aceast˘a ¸scoal˘a. Legea ˆınv˘ a¸t˘ amˆ antului secundar promulgat˘a de Spiru Haret cerea candidat¸ilor la profesorat s˘a fi trecut un examen de capacitate. Dar pentru a se putea prezenta la acest examen candidat¸ii trebuiau s˘a frecventeze un Seminar Pedagogic, s˘a parcurg˘a modulul pedagogic, am spune noi ast˘azi, ¸si implicit s˘a fac˘a un num˘ar de ore de practic˘a pedagogic˘a. De aceea s-a creat Seminarul Pedagogic Universitar cu propria ¸scoal˘ a de aplicat¸ie. Aceasta se ˆıntˆ ampla la 1 noiembrie 1899, localul ¸scolii fiind casa Aslan de la vale de Universitatea nou˘a din Copou. S¸coala de aplicat¸ie avea un dublu rol: pentru student¸i sau alsolvent¸i era locul unde ˆı¸si desf˘a¸surau practica pedagogic˘a, pentru profesorii universitari era locul ˆın care ˆı¸si puneau ˆın practic˘a metodele moderne de predare, un laborator de pedagogie experimental˘a. Se observ˘a din capul locului coerent¸a care exista ˆıntre activitatea didactic˘a, pregatirea pedagogic˘a a viitorilor profesori ¸si activitatea de cercetare a universitarilor. Nu este de mirare c˘a de aici au plecat majoritatea cadrelor didactice din liceele Moldovei. Mult¸i absolvent¸i 1 N. R. Pentru informat ¸ii mai ample recomand˘ am cartea Magda Negrea – Liceul de Aplicat ¸ie al Semninarului Pedagogic Universitar – Ia¸ si cu un studiu introductiv de prof.dr. Adrian Neculau.

123

ai ¸scolii de aplicat¸ie au revenit la SPU ca practicant¸i, iar unii chiar ca profesori. De asemenea unii profesori ai SPU au pus bazele unor ¸scoli asem˘an˘atoare la Cern˘aut¸i (Constantin Narly)) ¸si Cluj sau a Universit˘a¸tii din Chi¸sin˘au (Constantin Fedele¸s). Liceul de Aplicat¸ie al SPU a funct¸ionat cu succes pˆan˘a la reforma ˆınv˘a¸t˘ amˆ antului din 1948. O istorie de jum˘atate de veac marcat˘a de personalitatea celor doi directori, ambii pedagogi de frunte ai ¸scolii ie¸sene: Ioan G˘ av˘ anescul ¸si S ¸ tefan Bˆ ars˘ anescu. Ioan G˘av˘ anescul, pedagog cu studii ˆın Germania, a pus bazele unui regulament organic al institut¸iei. ˆınainte de deschiderea ¸scolii a convocat profesorii ˆın ¸sase conferint¸e ˆın care a fixat normele ¸si principiile generale didactice ¸si de ordine ¸si disciplin˘a. Ideea c˘al˘ auzitoare a fost ”educat¸ia ca scop, instruct¸ia ca mijloc”. S¸coala trebuie s˘a dezvolte simt¸ul practic ¸si spontaneitatea mental˘a. De aceea ˆın predare se va folosi metoda activ˘ a, intuit¸ia ¸si corelat¸ia ˆıntre obiectele de studiu. Educat¸ia nu trebuie s˘a fie teoretic˘a ¸si intelectual˘a ci ”practic˘ a ¸si voluntar˘ a”, elevul, asemeni lui Robinson Crusoe, s˘a ¸stie ˆın orice ˆımprejurare ”a g˘ asi mijloacele practice pentru a ˆınvinge orice greutate”. Pe lˆang˘ a activitatea teoretic˘a, elevul era antrenat ˆın multe activit˘a¸ti practice: cartonaj, modelaj, tˆampl˘arie. La SPU din Ia¸si s-a introdus pentru prima dat˘a lucrul manual la o ¸scoal˘a de cultur˘a general˘a. Ioan G˘av˘anescul a fost director pˆan˘ a ˆın 1932, cˆand a ie¸sit la pensie, dar a continuat s˘a ¸tin˘a conferint¸e, era membru al partidului conservator, a avut convingeri de dreapta. La venirea comuni¸stilor la putere, de¸si se apropia de 90 de ani a fost arestat ¸si moare ˆın ˆınchisoare ˆın 1951. ˆIntre 1932 ¸si 1938 director este Paul Nicorescu, istoric, fiind ajutat de Vasile Pavelcu, responsabil cu practica pedagogic˘a ¸si de E. Briul care r˘aspundea de activitatea dirigint¸ilor ¸si disciplina ¸scolar˘a. Sub directoratul lui Paul Nicorescu se construie¸ste aripa de vest a ¸scolii, care exist˘a ¸si azi la adresa din Toma Cozma, nr. 2. S ¸ tefan Bˆ ars˘ anescu, care ˆıi urmeaz˘a ca director, continu˘a s˘a construiasc˘a ˆın ciuda vremurilor de r˘azboi. Sub coordonarea lui S¸tefan Bˆars˘anescu se construie¸ste latura din centru ¸si se toarn˘a fundat¸ia pentru aripa est care ar fi ˆınchis acest amsamblu. ˆın aceast˘a activitate, S¸tefan Bˆars˘anescu este secundat de directorul administrativ I. Rick ¸si de directorul adjunct D. Gafit¸anu. Odat˘ a cu venirea lui S¸tefan Bˆars˘anescu la conducerea Seminarului Pedagogic Universitar, ˆıncepe o perioad˘a de dezvoltare distinct˘a a acestei institut¸ii. Cu studii la Universitatea din Ia¸si ¸si dou˘a doctorate ˆın Germania, S¸tefan Bˆars˘anescu are toate atributele unui mare pedagog. Spre deosebire de Ioan G˘av˘anescul, care punea accent pe formarea caracterului ¸si preg˘atirea practic˘a, S¸tefan Bˆars˘anescu pune accent pe lect¸ie, pe transmiterea de valori. Totul graviteaz˘a ˆın jurul ideii de cultur˘a. Pentru S¸tefan Bˆars˘ anescu un ”om cultivat” este acela care se conduce ˆın activitatea sa dup˘a ”valorile ve¸snice” ˆımbog˘a¸tindu-¸si ¸si ordonˆandu-¸si n˘azuint¸ele dup˘a adev˘ ar (¸stiint¸˘ a), bine(moral˘ a) ¸si frumos(art˘ a). ˆIn privint¸a student¸ilor practicant¸i, S¸tefan Bˆars˘anescu ˆıi roag˘a pe profesori ”s˘ a-i cˆ a¸stige ca pe ni¸ste prieteni, s˘ a nu fie descurajat¸i, s˘ a se simt˘ a ca ˆıntr-o mare familie. Practicant¸ii asistau la un num˘ar impresionant de ore pˆan˘a erau l˘asat¸i s˘a fac˘a lect¸ii de prob˘a. Toate lect¸iile erau discutate. Fiecare practicant ˆı¸si ˆıntocmea un dosar consistent cu observat¸ii, rapoarte, planuri de lect¸ii, un adev˘arat ghid cu care pornea ˆın activitatea la catedr˘a. Dar ˆın martie 1944 tot Ia¸sul pleac˘a ˆın refugiu. Tot ce s-a putut demonta ¸si ambala a fost pus ˆın 14 vagoane ¸si transportat ˆın Banat. Ia¸sul est bombardat, trei bombe 124

c˘azˆand ¸si pe Liceul de aplicat¸ie. ˆIn martie 1945, Liceul revine din refugiu ¸si pe 9 aprilie se ˆıncep cursurile. Zece vagoane din cele 14 sunt recuperate, se fac reparat¸ii, ˆıncˆat la 1 oct. 1945 cl˘adirea ad˘aposte¸ste atˆat Liceul de aplicat¸ie cˆat ¸si clase de la Liceul Internat ¸si de la ”Oltea Doamna”. Se fac demersuri pentru a continua construct¸ia aripei de est la care de mult¸i ani era turnat˘a fundat¸ia. Totul p˘area ˆınc˘a posibil, dar reforma ˆınv˘ a¸t˘ amˆ antului din 1948 face totul imposibil. Seminarul Pedagogic se desfiint¸eaz˘ a, Liceul de aplicat¸ie ˆı¸si schimb˘a aceast˘a menire ¸si devine Liceul Clasic. Se poate spune c˘a ˆıncepe drama psihologiei ¸si pedagogiei. Disciplinile acestea aproape c˘a dispar din ˆınv˘ a¸t˘ amˆ antul universitar, practica pedagogic˘a se reduce ca num˘ar de ore ¸si ca important¸˘ a, dar, ˆın mod paradoxal, se face cu prec˘adere tot la fostul Liceu de aplicat¸ie. S¸tefan Bˆars˘ anescu este ˆındep˘artat atˆat de la liceu cˆat ¸si de la Universitate. Abia dac˘a ocup˘a un post de bibliotecar. Este reprimit la catedr˘a ˆın 1957. Avea nu numai cultur˘a, ci ¸si har ˆı¸si aminte¸ste studentul de atunci Adrian Neculau, acum profesor de psihologia grupului social ¸si fervent admirator al S.P.U. de alt˘adat˘a. Liceul de aplicat¸ie fusese doar de b˘aiet¸i, Liceul Clasic este mixt ¸si are 12 clase. Din 1953 se nume¸ste S ¸ coala Medie Clasic˘ a, apoi din anul urm˘ator se numeste S ¸ coala Medie nr. 2 de fete. Toate aceste schimb˘ari, adev˘arate convulsii pentru ˆınv˘a¸t˘amˆant ˆın general ¸si pentru liceul de care ne ocup˘am ˆın special, precum ¸si politizarea ˆın exces a lect¸iilor au drept scop ruperea de istoria ¸scolii, uitarea bunei tradit¸ii. Din anul 1956, ¸scolile medii devin mixte ¸si se num˘ ar˘ a din nou, liceul nostru semicentenar devenind S ¸ coala Medie nr. 4. Pentru ca uitarea s˘a se a¸stearn˘ a ¸si mai bine pe istoria ¸scolii, ˆın loc s˘a se termine de construit aripa est, se ˆıncepe construirea unui nou local pe strada Oastei nr. 1. S¸coala Medie nr. 4, care ˆıntre timp a c˘ap˘atat ¸si numele de Liceul ”Garabet Ibr˘ aileanu”, se mut˘ a ˆın noul local ˆın 1962. ˆIn localul vechi continu˘a s˘a funct¸ioneze internatul pentru mai mult¸i ani, apoi aici se instaleaz˘a dispensarul pentru elevi ¸si student¸i. Vis-`a-vis, ˆın vechea ¸scoal˘a de ˆınv˘a¸t˘atoare, funct¸ioneaz˘a Facultatea de psihologie ¸si pedagogie, adic˘a fostul SPU. E doar ironia sort¸ii sau contratimpul tragic care ne marcheaz˘ a pe noi, romˆanii? Vechea cl˘adire din Toma Cozma nr. 2 a¸steapt˘a ¸si acum s˘a fie terminat˘a ¸si s˘a serveasc˘a drept ¸scoal˘a ˆıntre marile ¸scoli ale Ia¸sului: Universitate, Facultatea de psihologie, Liceul Internat ¸si nu departe Liceul Nat¸ional ¸si fosta ”Oltea Doamna”. ˆIn noul local din dealul Copoului, liceul are profilul real - umanist ¸si din 1967 devine ¸scoal˘ a pilot pentru ˆınv˘ a¸tarea limbilor str˘aine, ˆın spet¸˘a limba francez˘a, din ciclul primar. ˆIn 1977 ˆı¸si schimb˘ a iar numele ¸si profilul, devenind Liceul Industrial Hidrotehnic. Dar pe fiecare an de studiu se p˘astreaz˘a o clas˘a de real˘a ¸si una de filologie. ˆIn 1990 revine la profilul real - umanist. Pe baza tradit¸iei de studii clasice, la fiecare an de studiu a existat pˆan˘ a ˆın 2004 o clas˘a de limbi clasice (limba latin˘a ¸si greaca veche). Actualmente liceul este teoretic ¸si are clase de informatic˘a, ¸stiint¸e ale naturii, filologie. Povestea Liceului ”Garabet Ibr˘ aileanu” e povestea noastr˘a a tuturor, mereu ”sub vremuri”. S¸i totu¸si spiritul Liceului de aplicat¸ie, s˘amˆant¸a bun˘a sem˘anat˘a de acei dasc˘ali de demult, cu dragoste de ¸tar˘a ¸si de profesie nu a pierit ¸si aducerea ˆın memorie a unui secol de ¸scoal˘ a ne ˆıncurajeaz˘a ¸si ne oblig˘a.

Prof. Magda NEGREA 125

LIDIA COHAL (1930-2009) Evenimente a c˘aror ˆımpletire este mai presus de dorint¸e ¸si rat¸iuni ne oblig˘a la oarece z˘abav˘ a. Pe lˆang˘ a numeroasele, nobilele ¸si acaparantele obligat¸ii ale profesorilor de matematic˘a, ni se relev˘a una mai rar evident¸iat˘a, dar pe care o g˘asim plin˘a de semnificat¸ii: activitatea ˆın familia substant¸ial extins˘a a matematicienilor. Lidia s-a n˘ascut la 30 octombrie 1930 ˆın satul Ciutule¸sti, judet¸ul B˘alt¸i, de partea mai zbuciumat˘a a Prutului. Absolv˘a ¸scoala primar˘a ˆın satul natal ¸si continu˘a primele patru clase de liceu la Liceul ”Regina Maria” din ora¸sul B˘alt¸i. Nu consider˘am necesar s˘a explic˘am fracturile sociale care au determinat ca ultimele patru clase s˘a ¸si le desf˘a¸soare la Liceul ”Ecaterina Teodoroiu” din Tˆargu Jiu. Lidia devine student˘ a la Facultatea de matematic˘a ¸si fizic˘a la Universitatea ”Al. I. Cuza” Ia¸si. Ca recunoa¸stere a meritelor de la ˆınceputul anului IV este numit˘a preparator. La absolvirea facult˘a¸tii, ˆın anul 1952, a fost ˆıncadrat˘a ca asistent universitar la Catedra de geometrie, bine reputat˘a inclusiv la nivel internat¸ional. ˆImpreun˘a cu colegele ¸si prietenele Alice Corduneanu ¸si Ligia Papuc (deja c˘as˘atorite cu distin¸si matematicieni) se lanseaz˘a ˆın activit˘a¸ti ¸stiint¸ifice ¸si didactice, devenind ¸si o harnic˘a secretar˘a a Filialei Ia¸si a SSM. ˆIn septembrie 1959 i se desface contractul de munc˘a: originea social˘a, fiic˘a de preot, era privit˘a drept ”necorespunz˘atoare”. Conducerea SSM se dovede¸ste mai put¸in influent¸abil˘ a ¸si decide retribuirea fructuoasei sale activit˘a¸ti de secretar˘a de filial˘a. Este locul s˘a spunem c˘a Societatea preluase aproape integral sarcinile de perfect¸ionare a profesorilor de matematic˘a prin ¸scoli de var˘a; filiala ie¸sean˘a activ˘a ˆın aceast˘a directie extrem de sust¸inut ¸si materialele respective au fost incorporate ˆın patru volume de ”Probleme actuale de matematic˘a”. ˆIn septembrie 1960 este ˆıncadrat˘a ¸si ˆın ˆınv˘a¸t˘amˆantul liceal din Ia¸si, p˘astrˆandu-¸si ¸si atribut¸iile de ”secretar˘a de filial˘a model”: extrem de precis˘a g˘asea mereu zˆambetul prietenesc stimulator. A funct¸ionat la liceele ie¸sene ”Nat¸ional”, ”Emil Racovit¸˘a” ¸si ”Mihai Eminescu”. Din perioada ˆın care era ˆıncadrat˘a ca asistent universitar, Lidia ˆıncepuse documentarea ¸si lucrul la o carte destinat˘a ”moderniz˘arii pred˘arii geometriei”. Coordona eforturile profesorul Adolf Haimovici, pre¸sedinte al filialei Ia¸si ¸si mai colaborau prietenele mai sus numite Alice ¸si Ligia. Cartea Elemente de geometrie a planului a ap˘arut abia ˆın 1968 la Editura Didactic˘a ¸si Pedagogic˘a. Lidia s-a pensionat ˆın septembrie 1987. De multe ori se omite amintirea unor realiz˘ari ˆın pretent¸ioasa activitate de sot¸ie; nu vom face aici aceast˘a gre¸seal˘a. Prin optimismul ei molipsitor ¸si prin tonusul ei excelent, Lidia a potent¸at neˆıntrerupt activitatea sot¸ului ei, distinsul profesor ie¸sean Traian Cohal. Unii oameni nu mor de tot.

ˆ Prof.dr. Dan BRANZEI Pre¸ sedintele Filialei Ia¸ si a SSM 126

Concursul de matematic˘ a “Al. Myller” Edit¸ia a VII-a, Ia¸si, 28 martie 2009 Clasa a VII-a 1. Determinat¸i n ∈ N∗ cu proprietatea c˘a n!+3·2n = 6n−2 , unde n! = 1·2·3 . . . n. Artur B˘ al˘ auc˘ a 2. Fie triunghiul ABC ¸si punctul D situat pe latura [BC]. Ar˘atat¸i c˘a AB · DC + AC · BD ≥ AD · BC. *** 3. Fie p un num˘ ar natural impar. Se ¸stie c˘a oricare divizor al lui p are ultima cifr˘a diferit˘a de 3 ¸si 7. S˘a se arate c˘a num˘arul 5p + 1 nu este p˘atrat perfect. Mircea Fianu 4. Fie triunghiul echilateral ABC ¸si punctul D situat pe latura (AC). Bisectoarea unghiului ∠ABD intersecteaz˘ a paralela prin A la dreapta BC ˆın punctul E. Ar˘atat¸i c˘a AE + DC = BD. Cristian Laz˘ ar

Clasa a VIII-a 1. Fie un tetraedru regulat cu muchia de lungime 3. Pe suprafat¸a acestuia se consider˘ a 37 de puncte. Ar˘atat¸i c˘a printre aceste puncte exist˘a dou˘a astfel ˆıncˆat distant¸a dintre ele este cel mult egal˘a cu 1. *** 2. Determinat¸i perechile de numere (a, b) ∈ Z × Z care verific˘a egalitatea 2(a + b)2 + 3(a + b) + ab + 4 = 0. Petru R˘ aducanu 3. Fie a ≥ b ≥ c ≥ d ≥ 0 astfel ˆıncˆat a2 + b2 + c2 + d2 = 1. S˘a se arate c˘a a + b ≥ 1 ≥ c + d. Gheorghe Iurea 4. Numim piramid˘ a Myller o piramid˘a SABCD cu baza ABCD, care are SA = SB = SC = SD, ∠ASB ≡ ∠ASD ¸si ∠BSC ≡ ∠DSC, iar lungimile SA, AB, BC, CD, DA, AC, BD sunt numere naturale nenule. Aflat¸i piramida Myller de volum minim. Cristian Laz˘ ar

Clasa a IX-a 1. Determinat¸i n ∈ N∗ pentru care exist˘a o mult¸ime A ⊂ R cu n elemente, avˆand proprietatea a(b3 + 6) ≤ b(a3 + 6), ∀a, b ∈ A. Gheorghe Iurea 127

2. Care este num˘ arul minim de elemente care trebuie eliminate din mult¸imea {1, 2, 3, . . . , 100} astfel ˆıncˆ at ˆın mult¸imea r˘amas˘a s˘a nu existe trei elemente x, y, z pentru care xy = z? *** 3. Fie triunghiul ABC ¸si punctele M ∈ (AB), N ∈ (AC) astfel ˆıncˆat BM +CN = M N + BC. Not˘ am ρ raza cercului ˆınscris ˆın triunghiul AM N . Ar˘atat¸i c˘a È È √ √ ρ( bc + (p − b)(p − c)) ≤ r( bc − (p − b)(p − c)).

Dan Brˆ anzei 4. Num˘ arul ˆıntreg m are proprietatea c˘a, pentru orice num˘ar natural n, exist˘a an ∈ Z astfel ˆıncˆ at |nm − 80an + 1| < 20. Ar˘atat¸i c˘a 80 divide m. Dinu S ¸ erb˘ anescu

Clasa a X-a 1. Determinat¸i valorile lui n ∈ N∗ pentru care 41 are un multiplu de forma a00 . . . 0b} unde a, b sunt cifre zecimale nenule. | {z n+2 cifre

Mihai B˘ alun˘ a 2. Fie ABC un triunghi, k ∈ R\{1} ¸si punctele M ∈ BC, N ∈ CA, P ∈ AB MB NC PA astfel ˆıncˆ at = = = k. Se ¸stie c˘a AM = BN = CP. Demonstrat¸i c˘a MC NA PB triunghiul ABC este echilateral. I. V. Maftei 3. Pentru x, y ∈ R definim f (x; y) = distant¸a de la |x − y| la cel mai apropiat ˆıntreg, iar pentru o mult¸ime finit˘a M ⊂ [0, 1] definim s(M ) =

X

f (x, y).

x,y∈M,x 2, un num˘ ar prim ¸si f un polinom cu coeficient¸i ˆıntregi, de grad p−1, cu proprietatea c˘a pentru orice a, b numere ˆıntregi pentru care p divide f (a) − f (b), rezult˘a c˘a p divide a − b. Ar˘ atat¸i c˘a f are coeficientul dominant divizibil cu p. Marian Andronache

R˘ aspunsul la ”recreat¸ia” de la pag. ... este: Num˘ arul maxim de operat¸ii cu care se poate scrie 2009 este 4017 : 2009 = 10 : 10 + 10 : 10 + · · · + 10 : 10 (2009 termeni). Num˘ arul minim de operat¸ii este 209: 2009 = (10 + 10 + · · · + 10) + (10 : 10 + 10 : 10 + · · · + 10 : 10) ( 200 termeni ˆın prima parantez˘ a ¸si 9 ˆın cea de-a doua). 129

Concursul de matematic˘ a ”Florica T. Cˆ ampan” Etapa judet¸ean˘ a, 21 februarie 2009 Clasa a IV-a (Subiect elaborat de Dumitru Pˆarˆaial˘a ¸si C˘at˘alin Budeanu) 1. Fiica, tat˘al ¸si bunica au ˆımpreun˘a 90 de ani. Peste doi ani, tat˘al va avea de opt ori vˆarsta fiicei, iar bunica de dou˘a ori vˆarsta actual˘a a tat˘alui. Aflat¸i vˆarsta fiec˘aruia ˆın prezent. 2. Un blocnotes are 100 de pagini, numerotate de la 1 la 100. Se rup din acesta, la ˆıntamplare, 30 de foi, apoi se face suma numerelor ce marcheaz˘a paginile r˘amase. Este posibil ca aceast˘a sum˘a s˘a fie egal˘a cu 800? Justificat¸i! 3. O cantitate de 120 kg de prune a fost ˆımp˘art¸it˘a ˆın mod egal ˆın mai multe l˘azi. ˆIn vederea transportului, pentru a se evita pierderile, s-a transferat cˆate un sfert din cantitatea de prune din fiecare lad˘a, obt¸inˆadu-se astfel o nou˘a lad˘a cu tot atˆatea kilograme de prune cˆate au r˘amas ˆın fiecare din l˘azile init¸iale. a) Cˆate l˘azi au fost la ˆınceput?Justificat¸i! b) Cˆate kilograme de prune erau init¸ial ˆın fiecare lad˘a? c) Cˆa¸ti lei s-au ˆıncasat din vˆanzarea prunelor din lada nou format˘a, dac˘a un kilogram de prune de calitatea ˆıntˆai cost˘a 4 lei, un kilogram de prune de calitatea a doua cost˘a 3 lei, iar la fiecare 2 kilograme de prune de calitatatea ˆıntˆai exist˘a cˆate 3kg de prune de calitatea a doua?

Clasa a V-a (Subiect elaborat de Gheorghe Iurea ¸si Andrei Nedelcu) 1. Not˘am cu a1 , a2 , ....., a2009 numerele 1, 2, . . . , 2009, scrise ˆın alt˘a ordine. Ar˘atat¸i c˘a num˘ arul N = (1 + a1 ) · (2 + a2 ) · . . . · (2009 + a2009 ) este par. Este posibil ca N s˘a nu fie divizibil cu 4 ? 2. ˆIntr-un p˘atrat 7 × 7 sunt scrise 49 numere naturale diferite. De pe fiecare linie se consider˘a cel mai mic num˘ar, iar cel mai mare dintre aceste ¸sapte numere se noteaz˘a cu x. De pe fiecare coloan˘a se consider˘a cel mai mare num˘ar, iar cel mai mic dintre aceste ¸sapte numere se noteaz˘a cu y. Justificat¸i c˘a x ≤ y. 3. a) Ar˘atat¸i c˘a num˘ arul 3 · 102009 − 8 nu este p˘atrat perfect. b) Ar˘atat¸i c˘a num˘ arul 221506 are cel put¸in 2009 cifre.

Clasa a VI-a (Subiect elaborat de Claudiu S¸tefan Popa ¸si Ciprian Baghiu) c a 1. a) Fie proport¸ia = , cu tot¸i termenii numere naturale, astfel ˆıncˆat d este b d divizibil ¸si cu b, ¸si cu c, iar b ¸si c sunt prime ˆıntre ele. Demonstrat¸i c˘a bcd este p˘atrat perfect. b) Dac˘a un extrem al unei proport¸ii cu tot¸i termenii numere naturale nenule este divizibil cu fiecare din mezii ei, ar˘atat¸i c˘a suma tuturor termenilor este divizibil˘a cu cel˘alalt extrem. Claudiu S ¸ tefan Popa 130

2. Din cauza crizei economice, pret¸ul unei m˘arfi se reduce s˘apt˘amˆanal cu 50%, timp de n s˘ apt˘ amˆ ani, unde n ∈ N, n ≥ 2. a) Consider˘am c˘a pret¸ul init¸ial al m˘arfii este de 32 de lei. S¸tiind c˘a, atunci cˆand este cazul, se procedeaz˘a la rotunjirea pret¸ului dup˘a o anumit˘a reducere, pentru ca el s˘a poat˘a fi pl˘atit cu unit˘a¸tile monetare existente ¸si c˘a ˆın a n-a s˘apt˘amˆan˘a pret¸ul afi¸sat este acela¸si cu cel din s˘apt˘ amˆ ana n – 1, s˘a se afle n. b) Presupunˆand acum c˘a pret¸urile succesiv obt¸inute sunt exprimate f˘ar˘a rotunjire ¸si c˘a pret¸ul init¸ial al m˘arfii este a lei, aratat¸i c˘a suma ˆıntre pret¸urile m˘arfii din primele n – 1 s˘apt˘ amˆ ani ¸si dublul pret¸ului m˘arfii din a n-a s˘apt˘amˆan˘a este a lei. Claudiu S ¸ tefan Popa 3. Consider˘am triunghiul isoscel ABC, cu AB = AC ¸si m(]BAC) < 900 . Punctele M ¸si N sunt ˆın a¸sa fel ˆıncˆ at AB separ˘a C ¸si M, AC separ˘a B ¸si N, m(]M AB) = m(]N AC) = 900 , iar AM = AN = AB. Ar˘atat¸i c˘a: a) M C = N B; b) M B ̸= N B; c) dac˘a, ˆın plus, triunghiul MNB este isoscel, calculat¸i m(]BAC).

Clasa a VII-a (Subiect elaborat de Vasile Nechita ¸si Ionel Nechifor )

√ 1. a) Fie a, b, c numere reale strict pozitive astfel ˆıncˆat 5a + 3c = 4b ¸si 2b= 15ac. S˘a se arate c˘a numerele a, b, c sunt direct √ proport¸ionale cu numerele 6, 15 ¸si 10. b) Determinat¸i x ∈ N pentru care x2 + 9x + 13 ∈ N. 2. a) Pe o mas˘a sunt 20 monede. Radu ¸si Bogdan joac˘a urm˘atorul joc: ia fiecare, pe rˆand, de pe mas˘a, cˆate 1, 2 sau 3 monede; cˆa¸stig˘atorul este cel care ia ultima moned˘a. Cine va cˆa¸stiga jocul? (justificat¸i!) Dar dac˘a num˘arul monedelor de pe mas˘a este 21? b) Fiind dat un paralelogram ABCD ¸si o dreapt˘a d, care taie dreptele AB, BC, M A N B P C QD CD, DA ˆın punctele M, N, P, respectiv Q, demonstrat¸i c˘a · · · = 1. M B N C P D QA 3. Andrei, Bogdan, Costel ¸si Dan cump˘ar˘a trei torturi de ˆınghet¸at˘a de form˘a p˘atrat˘a, care au fiecare aceea¸si grosime ¸si compozit¸ie. S¸tim c˘a perimetrele torturilor 1 1 1 sunt invers proport¸ionale cu numerele , , , iar suma laturilor celor trei 36 160 164 torturi este 90cm. Explicat¸i cum vor ˆımp˘art¸i copiii cele trei torturi, pentru a lua fiecare port¸ii egale (f˘ar˘ a cˆant˘ arire).

Clasa a VIII-a (Subiect elaborat de Gabriel Popa ¸si Cristian Laz˘ar ) 1. a) Determinat¸i numerele ˆıntregi a ¸si b cu proprietatea c˘a a2 b + ab2 + a + b + 1 = 0. Recreat¸ii Matematice, 1/2000 b) Fie n un num˘ ar natural dat. Determinat¸i numerele reale strict pozitive x, y, z pentru care xn = yz, y n = zx ¸si z n = xy. Neculai Hˆ art¸an , Recreat¸ii Matematice, 2/2003 √ 2. Despre un num˘ ar natural n vom spune c˘a√are proprietatea (P ) dac˘a 24n + 1 ∈ Q ¸si vom spune c˘a are proprietatea (Q) dac˘a 24n + k ∈ R\Q, ∀k ∈ {2, 3, 4, ..., 99} . 131

a) Dat¸i exemplu de un num˘ar natural care are proprietatea (P ), dar nu are proprietatea (Q). b) Dat¸i exemplu de un num˘ar natural care are proprietatea (Q), dar nu are proprietatea (P ). c) Dat¸i exemplu de un num˘ar natural care nu are nici proprietatea (P ), nici proprietatea (Q). d) Determinat¸i (cu justificare) cel mai mic num˘ar natural care are atˆat proprietatea (P ), cˆat ¸si proprietatea (Q). e) G˘asit¸i toate numerele naturale care au atˆat proprietatea (P ), cˆat ¸si proprietatea (Q). Cristian Laz˘ ar 3. Un zmeu din carton, care are forma unui triunghi ABC, las˘a pe p˘amˆant o umbr˘ a avˆ and forma unui triunghi A′ B ′ C ′ , care este asemenea cu triunghiul ABC. a) Dac˘a zmeul se afl˘a sub razele soarelui la amiaz˘a, demonstrat¸i c˘a ∆ABC ≡ ∆A′ B ′ C ′ . b) Dac˘a zmeul se afl˘a sub becul unui stˆalp de iluminat, mai rezult˘a ˆın mod necesar c˘a ∆ABC ≡ ∆A′ B ′ C ′ ? Constantin Cocea, Gabriel Popa

Etapa interjudet¸ean˘ a, 21 martie 2009 Clasa a IV-a (Subiect elaborat de Dan Brˆanzei ¸si C˘at˘alin Budeanu) 1. Aflat¸i suma resturilor ˆımp˘art¸irii a 2009 numere naturale consecutive la 15, ¸stiind c˘a ultimul se ˆımparte exact la 15. 2. Suma a dou˘a numere naturale este 26. ˆImp˘art¸ind primul num˘ar la al doilea ¸si apoi pe al doilea la primul se obt¸ine, de fiecare dat˘a, aceea¸si sum˘a dintre cˆat ¸si rest, aceasta fiind cu 8 mai mic˘a decˆat unul dintre numere. Aflat¸i numerele. 3. Fie num˘ arul N = 510152025 . . . 725730735 . . . 20020052010. a) Cˆate cifre are num˘ arul N ? b) Care este a 1000-a cifr˘a a lui N ? 4. (facultativ ) Centrul unei piet¸e are forma unui p˘atrat de latur˘a 7 ¸si este ocupat de o statuie cu un soclu p˘atrat de latur˘a 1. Se poate acoperi suprafat¸a r˘amas˘a cu patru dreptunghiuri avˆ and dimensiunile 3 ¸si 4?

Clasa a V-a (Subiect elaborat de Gheorghe Iurea ¸si Andrei Nedelcu) 1. La un turneu de fotbal ˆın sal˘a particip˘a patru echipe. Se acord˘a 2 puncte pentru victorie, 1 punct pentru egal ¸si 0 puncte la ˆınfrˆangere. Fiecare echip˘a joac˘a cu fiecare cˆate un singur meci. ˆIn clasamentul final nu sunt dou˘a echipe la egalitate de puncte. Care este num˘ arul minim de puncte pe care ˆıl poate avea echipa cˆa¸stig˘atoare? Realizat¸i o distribut¸ie a rezultatului meciurilor care s˘a corespund˘a situat¸iei de mai sus. 132

2. Consider˘am mult¸imea A = {2a · 3b · 5c |a, b, c ∈ N}. Ar˘atat¸i c˘a printre oricare 9 elemente ale mult¸imii A exist˘a cel put¸in dou˘a a c˘aror produs este p˘atrat perfect. 3. Un p˘atrat cu latura 5 se ˆımparte ˆın p˘atrate cu latura 1, care se numeroteaz˘a cu numere de la 1 la 25. Se calculeaz˘a sumele de pe fiecare linie ¸si de pe fiecare coloan˘a. Exist˘a o numerotare astfel ˆıncˆ at exact o sum˘a s˘a fie num˘ar par? 4. (facultativ ) Num˘arul 31 organizeaz˘a o petrecere ˆın ˆımp˘ar˘a¸tia Numerelor Naturale. Dac˘a x ∈ N, atunci 9x – 2 ¸si 9x + 2 sunt p˘arint¸ii lui x, iar 9x + 4 este bunicul lui x. Se ¸stie c˘a, dac˘a bunicul unui num˘ar se afl˘a la petrecere, atunci este invitat ¸si nepotul s˘au. De asemenea, dac˘a un num˘ar este invitat la petrecere, atunci sunt invitat¸i ¸si p˘arint¸ii s˘ai. Ar˘atat¸i c˘a num˘arul 2009 este invitat la petrecere.

Clasa a VI-a (Subiect elaborat de Ionel Nechifor ¸si Ciprian Baghiu) m + 2267 n + 5388 < . 178 m+3 n + 7291 2. Pe un teren de forma unui triunghi echilateral cu latura de 8m se planteaz˘a 5 brazi. a) Demonstrat¸i c˘a oricum am planta brazii, vor exista cel put¸in doi avˆand ˆıntre ei o distant¸˘ a nu mai mare de 4 metri. b) ˆIn cˆate moduri se pot planta cei 5 brazi, astfel ˆıncˆat s˘a nu existe doi la distant¸˘a strict mai mic˘a de 4 metri? 1. S˘a se arate c˘a oricare ar fi m, n ∈ N, atunci

3. Fie a un num˘ ar natural nenul. Ar˘atat¸i c˘a exist˘a b ∈ N∗ astfel ˆıncˆat 9b − 3b s˘a fie divizibil cu a. Ciprian Baghiu 4.(facultativ ) Determinat¸i toate valorile num˘arului natural n, pentru care fract¸ia 2n2 + 5 este reductibil˘a. 3n2 + 5n + 10

Clasa a VII-a (Subiect elaborat de Vasile Nechita ¸si Claudiu S¸tefan Popa) 1. Luˆ andu-se dup˘a harta care a apart¸inut piratului Supernegru, pentru a g˘asi comoara fabuloas˘a (adic˘a o supercomoar˘a) ascuns˘a de acesta cˆandva, undeva ˆın Caraibe, patru superc˘aut˘ atori trebuie s˘a caute ˆın interiorului patrulaterului NESV situat pe un teren plat ¸si obt¸inut astfel: din acela¸si punct O, fiecare dintre cei 4 merge, ˆın linie dreapt˘a; primul face b pa¸si spre nord pˆan˘a ˆın N, al doilea c pa¸si spre est pˆan˘a ˆın E, al treilea a pa¸si spre sud pˆan˘ a ˆın S ¸si ultimul a pa¸si spre vest pˆan˘a ˆın V (a,b,c numere naturale). S¸tiind c˘a ˆıntre N ¸si E sunt exact a pa¸si ¸si c˘a lungimea pasului fiec˘aruia dintre cei 4 este aceea¸si ¸si constant˘ a, ar˘atat¸i c˘a suprafat¸a patrulaterului NESV are aria exprimat˘a printr-un num˘ ar natural (unitatea de m˘asur˘a este p˘atratul cu latura de 1 pas). Este posibil ca aria patrulaterului s˘a fie egal˘a cu aria unui p˘atrat de latur˘a 6 pa¸si ? Claudiu S ¸ tefan Popa 2. a) Fie a, b, cnumere reale pozitive. Demonstrat¸i c˘a a este media geometric˘a a 1 1 1 + . lui b ¸si c dac˘ a ¸si numai dac˘a = a a+b a+c 133

b) Fie O intersect¸ia diagonalelor patrulaterului convex ABCD. Demonstrat¸i c˘a 1 1 1 AB|| CD dac˘a ¸si numai dac˘a = + . AAOD AACD AABD Claudiu S ¸ tefan Popa 3. Rezolvat¸i ˆın mult¸imea numerelor ˆıntregi ecuat¸ia x2 (y − 1) + y 2 (x − 1) = 1. Gabriel Mˆır¸sanu

Clasa a VIII-a (Subiect elaborat de Gabriel Popa ¸si Cristian Laz˘ar ) 1. Se d˘a mult¸imea M = {1, 2, 3, 4} . Se formeaz˘a toate sumele cu termeni distinct¸i din M, luˆand ˆın seam˘a inclusiv sumele cu un singur termen. Dou˘a sume se consider˘a a fi distincte dac˘a difer˘a fie prin cel put¸in un termen, fie prin ordinea termenilor (de exemplu, sumele 4, 1 + 3, 3 + 1, 1 + 2 + 3 sunt distincte). a) Cˆate sume diferite se pot forma? b) Ar˘atat¸i c˘a orice num˘ ar cuprins ˆıntre 1 ¸si 10 poate fi obt¸inut ca rezultat ˆın urma calcul˘arii unei astfel de sume. c) Dou˘a numere cuprinse ˆıntre 1 ¸si 10 vor fi numite ˆınrudite dac˘a se obt¸in de acela¸si num˘ ar de ori ca rezultat al unor sume ca cele din enunt¸. Determinat¸i perechile de numere ˆınrudite. Gabriel Popa 2. Fie VABCD ¸si SABCD dou˘a √piramide patrulatere regulate, avˆand ca baz˘a comun˘ a p˘atratul ABCD de latur˘a 12 2cm, vˆarfurile V ¸si S de o parte ¸si de alta a planului (ABCD) ¸si ˆın care ˆın˘alt¸imile ¸si muchiile laterale se exprim˘a (ˆın centimetri) prin numere ˆıntregi. a) Demonstrat¸i c˘a punctele V, A, S ¸si C sunt coplanare. b) Aflat¸i valorile posibile ale ˆın˘alt¸imii VO a piramidei VABCD. c) Ar˘atat¸i c˘a patrulaterul VASC este circumscriptibil. d) Dac˘a patrulaterul VASC este inscriptibil, determinat¸i lungimea segmentului VS. Cristian Laz˘ ar 3. Trei greieri ”sar capra”: un greiere, aflat ˆın punctul A, sare peste un alt greiere, aflat ˆın B, ¸si ajunge ˆın C, unde C este simetricul lui A fat¸˘a de B. Apoi, acela¸si greiere sau un altul sare peste un partener de joac˘a ¸si tot a¸sa. Dac˘a init¸ial cei trei greieri se aflau ˆın trei dintre vˆarfurile unui p˘atrat, se poate ca la un moment dat, ˆın cursul jocului, unul dintre greieri s˘a ajung˘a ˆın cel de-al patrulea vˆarf al acelui p˘atrat? Mircea Ganga

Vizitat¸i noua pagina web a revistei:

http://www.recreatiimatematice.ro

134

Solut¸iile problemelor propuse ˆın nr. 2/2008 Clasele primare P.154. Dorina are 15 baloane ro¸sii ¸si albastre. Cˆ ate baloane ro¸sii poate avea, dac˘ a num˘ arul acestora este mai mic decˆ at num˘ arul baloanelor albastre ¸si este cel put¸in egal cu 3? (Clasa I ) Inst. Maria Racu, Ia¸si Solut¸ie. Num˘arul baloanelor ro¸sii poate fi 3, 4, 5, 6 sau 7. P.155. Dintr-o carte lipsesc cˆ ateva pagini, de la num˘ arul 71 la num˘ arul 94. Cˆ ate foi lipsesc din aceast˘ a carte? (Clasa I ) Ionela B˘ ar˘ agan, elev˘ a, Ia¸si Solut¸ie. Prima foaie care lipse¸ste are paginile 71 ¸si 72, iar ultima are paginile 93 ¸si 94. ˆIn total lipsesc 12 foi. P.156. La concursul ”Desene pe asfalt”, elevii claselor I-IV de la S ¸ coala ”Otilia Cazimir” au acumulat 50 de puncte ¸si cel put¸in 2 premii din fiecare categorie. Care este cel mai mare num˘ ar de premii pe care-l pot obt¸ine elevii, dac˘ a pentru premiul I s-au acordat 10 puncte, pentru premiul al II-lea s-au acordat 6 puncte, iar pentru premiul al III-lea s-au acordat 4 puncte? ˆ (Clasa a II-a) Inv. Elena Porfir, Ia¸si Solut¸ie. Num˘ arul maxim de premii se obt¸ine ˆın cazul: 2 premii I, 3 premii II ¸si 3 premii III, ˆın total 8 premii. P.157. Prin golirea unui singur vas, ales dintre cele de mai jos, putem face ca restul vaselor s˘ a aib˘ a cantit˘ a¸ti egale de lichid. Care vas trebuie golit?

(Clasa a II-a) Amalia Cantemir, elev˘ a, Ia¸si Solut¸ie. Distribuim lichidul din vasul de 15 litri ˆın primele 6 vase astfel: 5 l, 4 l, 3 l, 2 l, 1 l, 0 l. ˆIn acest mod, ˆın fiecare vas vom avea 14 litri. P.158. Aflat¸i trei numere naturale ¸stiind c˘ a, adunˆ andu-le dou˘ a cˆ ate dou˘ a, obt¸inem 100, 89, respectiv 141. (Clasa a III-a) Inst. Maria Racu, Ia¸si Solut¸ie. Obt¸inem c˘a dublul sumei celor trei numere este 330, deci suma celor trei numere este 330 : 2 = 165. Astfel, primul num˘ar este 165 − 100 = 65, al doilea este 165 − 89 = 76, iar al treilea este 165 − 141 = 24. P.159. Se consider˘ a numerele: a = 1 + 4 + 7 + 10 + · · · + 2008, b = 2 + 5 + 8+ + · · · + 2009, c = 3 + 6 + 9 + · · · + 2010. Ar˘ atat¸i c˘ a suma a + b + c se ˆımparte exact la 3, f˘ ar˘ a s˘ a calculat¸i aceast˘ a sum˘ a. (Clasa a III-a) Iuliana Moldovan, elev˘ a, Ia¸si 135

Solut¸ie. Putem scrie a + b + c = (1 + 2 + 3) + (4 + 5 + 6) + (7 + 8 + 9) + . . . + (2008 + 2009 + 2010). ˆIn fiecare parantez˘a avem suma a trei numere consecutive, care se ˆımparte exact la 3, deci a + b + c se ˆımparte exact la 3. P.160. Num˘ arul a este de forma xy0, iar num˘ arul b este de forma uv. S˘ a se afle a ¸si b ¸stiind c˘ a a + b = 22 zeci. (Clasa a III-a) Drago¸s Toma, elev, Ia¸si Solut¸ie. Condit¸ia din problem˘a se scrie xy0 + uv = 220, din care se deduce v = 0, iar xy + u = 22. Pentru perechea (a, b) avem posibilit˘a¸tile: (210, 10); (200, 20); (190, 30) . . . (130, 90). P.161. Fie a ¸si b dou˘ a numere naturale astfel ˆıncˆ at diferent¸a lor este de 5 ori mai mic˘ a decˆ at suma lor. S˘ a se arate c˘ a num˘ arul cel mai mare se ˆımparte exact la 3, iar cel mai mic se ˆımparte exact la 2. (Clasa a IV-a) Diana T˘ an˘ asoaie, elev˘ a, Ia¸si Solut¸ie. Fie a ¸si b cele dou˘a numere. Din a + b = 5(a − b) obt¸inem 2a = 3b, deci b trebuie s˘a fie par. ˆInlocuind b = 2c, rezult˘a c˘a a = 3c, prin urmare a se ˆımparte exact la 3. P.162. Maria are 9 s˘ aculet¸i cu monede. Cel put¸in un s˘ aculet¸ cˆ ant˘ are¸ste un ˆ orice grupare de 5 s˘ kilogram. In aculet¸i, cel put¸in 3 s˘ aculet¸i au aceea¸si mas˘ a, iar ˆın orice grupare de 6 s˘ aculet¸i, cel mult 5 s˘ aculet¸i au aceea¸si mas˘ a. Care este cel mai mare num˘ ar de s˘ aculet¸i de 1 kg pe care ˆıl poate avea Maria? (Clasa a IV-a) Petru Asaftei, Ia¸si Solut¸ie. Nu putem avea mai mult de 5 s˘aculet¸i de 1 kg, deoarece am g˘asi o grupare de 6 s˘aculet¸i de aceea¸si mas˘a. Num˘arul maxim posibil de s˘aculet¸i de 1 kg este 5 ¸si poate fi atins, de exemplu, dac˘a Maria are 5 s˘aculet¸i de 1 kg ¸si ˆınc˘a 4 de o alt˘a mas˘a, aceea¸si, caz ˆın care ar fi ˆındeplinit˘a ¸si prima condit¸ie (conform principiului cutiei). P.163. Jum˘ atatea produsului a dou˘ a numere naturale consecutive, ˆımp˘ art¸it˘ a cu 3, nu poate da niciodat˘ a restul 2. Recreat¸ii S ¸ tiint¸ifice, Anul I (1883)- nr. 4, p.119 Solut¸ie. Dac˘ a unul dintre cele dou˘a numere consecutive se ˆımparte exact la 3, restul ˆımp˘ art¸irii din enunt¸ este 0. ˆIn caz contrar, produsul celor dou˘a numere consecutive, care este par, va da restul 2 la ˆımp˘art¸irea prin 3, iar jum˘atatea se va da la ˆımp˘ art¸irea prin 3 restul 1.

Clasa a V-a V.95. Dou˘ a numere naturale se scriu ˆın baza 10 folosind doar cifrele 1, 4, 6 ¸si 9. Poate fi unul dintre numere de 2008 ori mai mare decˆ at cel˘ alalt? C˘ at˘ alin Budeanu, Ia¸si Solut¸ie. Dac˘a numerele A ¸si B se scriu doar cu ajutorul cifrelor 1, 4, 6 ¸si 9, atunci U (2008A) ∈ {2, 8}, prin urmare 2008A ̸= B. R˘aspunsul la ˆıntrebarea din enunt¸ este negativ. V.96. Determinat¸i k, n ∈ N∗ astfel ˆıncˆ at (1 + 1 · n)+(2 + 2 · n)+· · ·+(k + k · n) = 3 · 4 · 5 · 6. Petru Asaftei, Ia¸si 136

Solut¸ie. Deoarece (1 + 1 · n) + (2 + 2 · n) + . . . + (k + k · n) = (1 + 2 + . . . + k(k + 1) , k) + n(1 + 2 + . . . + k) = (1 + 2 + . . . + k)(n + 1) ¸si 1 + 2 + . . . + k = 2 ecuat¸ia dat˘a este echivalent˘ a cu k(k + 1)(n + 1) = 2 · 3 · 4 · 5 · 6. Atunci k ¸si k + 1 sunt divizori ai num˘ arului 2 · 3 · 4 · 5 · 6, iar k(k + 1) ≤ 2 · 3 · 4 · 5 · 6. G˘asim solut¸iile (k, n) ∈ {(1, 359); (2, 119); (3, 59); (4, 35); (5, 23); (8, 9); (9, 7); (15, 2)}. V.97. Ar˘ atat¸i c˘ a num˘ arul N = 17n + 21n + 25n , n ∈ N, nu poate fi p˘ atrat perfect. Virginia Grigorescu, Craiova Solut¸ie. Deoarece (M a+1)n = M a+1, deducem c˘a 17n = M 4+1, 21n = M 4+1, n 25 = M 4 + 1, deci N = M 4 + 3, prin urmare N nu poate fi p˘atrat perfect. V.98. Fie n ∈ N∗ . S˘ a se demonstreze c˘ a num˘ arul N = 5050 . . . 505 (2n + 1 cifre) se scrie ca sum˘ a a 4n + 2 p˘ atrate perfecte distincte. Veronica Pl˘ ae¸su ¸si Dan Pl˘ ae¸su , Ia¸si Solut¸ie. Observ˘am c˘a N = 5 · 102n + 5 · 102n−2 + . . . + 5 · 102 + 5. Cum 5 = 22 + 12 , iar 5 · 102k = 500 · 102k−2 = (162 + 122 + 82 + 62 ) · (10k−1 )2 = (16 · 10k−1 )2 + (12 · 10k−1 )2 + (8 · 10k−1 )2 + (6 · 10k−1 )2 , cerint¸a problemei este demonstrat˘a. V.99. Se consider˘ a num˘ arul N = 1 + 11 + 101 + 1001 + · · · + 100 . . . 01}. | {z n cifre

a) Pentru n ∈ N, n ≥ 5, ar˘ atat¸i c˘ a 5 | N ⇔ 5 | n. b) Precizat¸i care dintre propozit¸iile ”3 | n ⇒ 3 | N ” ¸si ”3 | N ⇒ 3 | n” este adev˘ arat˘ a pentru orice n ≥ 3. Temistocle Bˆırsan, Ia¸si . . 11} +(n− Solut¸ie. a) Cum N = 1+10+100+. . .+100 . . . 0 + 1 + 1 + . . . + 1 = |11 .{z | {z } |

ncifre

{z

n−1 termeni

}

ncifre

1) = 11 . . 10} +n. Cum 5| 11 . . 10}, rezult˘a cerint¸a a). | .{z | .{z ncifre

n

b) Niciuna dintre implicat¸ii nu este adev˘arat˘a. ˆIntr-adev˘ar, pentru n = 3 avem c˘a . N = 1 + 11 + 101 = 113/.. 3. Apoi, pentru n = 5 avem c˘a N = 1 + 11 + 101 + 1001 + . . 10001 = 11115..3, ˆın timp ce 5/.. 3. V.100. Determinat¸i numerele naturale nenule a ¸si b pentru care exist˘ a n ∈ N a 3n + 2 astfel ˆıncˆ at = ¸si 3a + 2b < 100. b 7n + 5 Gheorghe Iurea, Ia¸si Solut¸ie. Dac˘ a d = (3n + 2, 7n + 5), atunci d|3n + 2 ¸si d|7n + 5, de unde d|3(7n + 3n + 2 este ireductibil˘a. Deoarece 5) − 7(3n + 2), cu alte cuvinte d = 1, deci fract¸ia 7n + 5 a 3n + 2 = , deducem c˘a a = k(3n + 2) ¸si b = k(7n + 5), unde k ∈ N∗ , iar inegalitatea b 7n + 5 3a + 2b < 100 devine k(23n + 16) < 100. Pentru n = 0 obt¸inem k ∈ {1, 2, 3, 4, 5, 6}, a = 2k ¸si b = 5k; pentru n = 1 avem k ∈ {1, 2}, a = 5k ¸si b = 12k. Dac˘a n = 2, atunci k = 1, a = 8, b = 19, iar pentru n = 3, k = 1, a = 11, b = 26. Pentru n ≥ 4, problema nu admite solut¸ii. 137

an + b , unde n, a, b, c, d ∈ N∗ , astfel ˆıncˆ at b ¸si d au cn + d parit˘ a¸ti diferite, iar a ¸si c au aceea¸si paritate. Ar˘ atat¸i c˘ a, dac˘ a ad − bc = 2k , k ∈ N∗ , atunci fract¸ia este ireductibil˘ a. Cosmin Manea ¸si Drago¸s Petric˘ a, Pite¸sti Solut¸ie. Presupunem prin absurd c˘a fract¸ia este reductibil˘a. Fie p un divizor comun al numerelor an+b ¸si cn+d, p ∈ N∗ , p ≥ 2; rezult˘a c˘a p|(a·n+b) ¸si p|(c·n+d), de unde p|ad − bc, deci p|2k ¸si atunci p este par. Totodat˘a, p|n(a + c) + b + d ¸si cum n(a + c) + b + d este num˘ ar impar, rezult˘a c˘a p este impar, contradict¸ie. V.101. Consider˘ am fract¸ia

Clasa a VI-a

a1 a2 VI.95. Determinat¸i numerele naturale nenule a1 , a2 ,. . . , a2008 , ¸stiind c˘ a = 1·2 a2 a3 a2007 a2088 = ··· = , iar a1 + a2008 = 2009. 2·3 2007 · 2008 Gheorghe Iurea, Ia¸si a1 a3 Solut¸ie. Considerˆand rapoartele dou˘a cˆate dou˘a, obt¸inem c˘a = = ... = 1 3 a2007 a2 a4 a2008 , iar = = ... = . Astfel, relat¸ia a1 + a2008 = 2009 devine a1 + 2007 2 4 2008∗ 1004a2 = 2009 ¸si cum a1 , a2 ∈ N , atunci a1 = 1, a2 = 2. Deducem c˘a ai = i, ∀i = 1, 2008. VI.96. Determinat¸i p ∈ N pentru care numerele p, p + 12, p + 22, p + 52, p + 72, p + 102 ¸si p + 132 sunt prime. Damian Marinescu, Tˆ argovi¸ste Solut¸ie. Pentru p = 7, numerele sunt 7, 19, 29, 59, 79, 109, 139 ¸si sunt toate prime. Dac˘a p ∈ {2, 3, 5}, obt¸inem numerele compuse 2 + 12, 3 + 12, respectiv 5 + 22. Dac˘a p ≥ 11, considerˆand p = M7 + r, cu r ∈ {0, 1, . . . , 6}, g˘asim ˆın fiecare caz cˆate un num˘ ar compus printre cele date. ˆIn concluzie, singura valoare convenabil˘a este p = 7. VI.97. a) Dac˘ a a, b, c, d, e, f ∈ N∗ sunt astfel ˆıncˆ at (a, b) = (c, d) = (e, f ) = a c e (b, d) = 1, iar t = − + ∈ N, ar˘ atat¸i c˘ a f = bd. b d f 4 1 7 b) Determinat¸i a, b ∈ N pentru care − + ∈ N. 2a + 1 2b 6 Cosmin Manea ¸si Drago¸s Petric˘ a, Pite¸sti e e Solut¸ie. a) Deoarece t · bd = ad − bc + bd · ∈ N, rezult˘a c˘a bd · ∈ N ¸si, cum f f c (e, f ) = 1, deducem c˘a f |bd. Pe de alt˘a parte, din t · bf = af − bf · + ed ∈ N ¸si d a c a t · df = df · − cf + de ∈ N, rezult˘a c˘a bf · ¸si df · sunt numere naturale. Folosind b d b faptul c˘a (b, a) = (d, b) = (d, c) = 1, obt¸inem c˘a d|f ¸si b|f ¸si, cum (b, d) = 1, atunci bd|f . Din f |bd ¸si bd|f , rezult˘a c˘a f = bd. b) Suntem ˆın condit¸iile punctului precedent; deducem c˘a 6 = (2a + 1) · 2b , de unde a = b = 1. Pentru aceste valori, vom avea t = 2 ∈ N. VI.98. Determinat¸i cel mai mic num˘ ar natural n cu proprietatea c˘ a num˘ arul zerourilor ˆın care se termin˘ a num˘ arul (n + 10)! este cu 2008 mai mare decˆ at num˘ arul zerourilor ˆın care se termin˘ a n! (unde n! = 1 · 2 · 3 · · · n). C˘ at˘ alin Budeanu, Ia¸si 138

Solut¸ie. Ipoteza problemei revine la faptul c˘a num˘arul (n + 1)(n + 2) . . . (n + 10) se termin˘a ˆın 2008 zerouri, ceea ce se ˆıntˆampl˘a atunci cˆand produsul (n + 1)(n + 2) . . . (n + 10) se divide cu 52008 ¸si nu se divide cu 52009 . Printre factorii produsului precedent, exist˘a exact doi care sunt divizibili cu 5, iar dintre ace¸stia unul nu mai este divizibil cu nicio alt˘a putere a lui 5, deci cel˘alalt se va divide cu 52007 . Cum dorim n minim, atunci n + 10 = 52007 , prin urmare n = 52007 − 10. VI.99. Un patrulater convex are dou˘ a laturi opuse congruente ¸si diagonalele congruente. Ar˘ atat¸i c˘ a patrulaterul este trapez isoscel sau dreptunghi. Ioan S˘ ac˘ aleanu, Hˆ arl˘ au Solut¸ie. Presupunem c˘a (AB) ≡ (CD) ¸si (AC) ≡ (BD). Din congruent¸a triÕ ≡ DCB, Õ iar din congruent¸a triunghiurilor unghiurilor ABC ¸si DCB, rezult˘a c˘a ABC Õ ≡ CDA. Õ Cum suma unghiurilor unui patrulater este ABD ¸si DCA, rezult˘a c˘a BAD ◦ Õ Õ 360 , deducem c˘a m(ABC) + m(BAD) = 180◦ , deci AD∥BC. Dac˘a AB∥CD, atunci ABCD este un paralelogram cu diagonalele congruente, deci este un dreptunghi. Dac˘a AB ∦ CD, atunc ABCD este un trapez cu diagonalele congruente, adic˘a un trapez isoscel. b ≥ 90◦ . S˘ Ò = 2m(C) Ò dac˘ VI.100. Fie △ABC cu m(A) a se arate c˘ a m(B) a ¸si numai dac˘ a exist˘ a M ∈ [BC] astfel ˆıncˆ at AB = AM = M C. Petru Asaftei, Ia¸si Ò = 2m(C) Ò ¸si fie M intersect¸ia mediatoarei laturii Solut¸ie. Presupunem c˘a m(B) b ≥ 90◦ , vom avea c˘ A [AC] cu BC; cum m(A) a M ∈ [BC]. Evident atunci c˘a △M AC este isoscel cu M A = M C ¸si Ö Ö cum AM B este unghi exterior, deducem c˘a m(AM B) = Ò Ò 2m(C) = m(B). Astfel, △ABM va fi isoscel cu AB = C M B AM, ceea ce ˆıncheie justficarea afirmat¸iei directe. Reciproc, dac˘a exist˘a M ∈ [BC] cu AB = AM = M C, din triunghiurile isoscele Ò = m(AM Ö Ò ABM ¸si M AC obt¸inem c˘a m(B) B) = 2m(C). b = 90◦ ¸si CD bisectoarea VI.101. Fie ABC un triunghi dreptunghic cu m(A) Ò Ò interunghiului C, D ∈ (AB). Perpendiculara din D pe bisectoarea unghiului B secteaz˘ a ipotenuza BC ˆın E. Dac˘ a P este punctul de intersect¸ie a bisectoarelor unghiurilor triunghiului ABC, iar M este punctul de intersect¸ie dintre EP ¸si AC, ar˘ atat¸i Ö Õ c˘ aM PA ≡ P BE. Nela Ciceu, Bac˘ au ¸si Titu Zvonaru, Com˘ ane¸sti Solut¸ie. Din enunt¸ rezult˘a c˘a triunghiul BDE este isoscel ¸si c˘a BP este mediÕ = m(CDB) Õ − C atoarea segmentului DE. Totodat˘a, m(CDE) 1 1 ◦ ◦ ◦ Ò − m(B) Ò − [180 − m(B)] Ò = 45 . Õ = 180 − m(C) m(EDB) 2 2 Prin urmare, triunghiul DP E este dreptunghic isoscel. Astfel, Ò rezult˘ CP ⊥ M E ¸si cum CP este bisectoarea unghiului C, a Ö c˘a triunghiul CM E este isoscel. Deducem c˘a m(M P A) = E P Ö Ö 180◦ − m(P M A) − 45◦ = 135◦ − 180◦ + m(CM E) = −45◦ + M 1 Ò = 1 [90◦ − m(C)] Ò = 1 m(B) Ò = m(P Õ [180◦ − m(C)] BE). A B D 2 2 2

139

Clasa a VII-a M B A VII.95. Fie ABCD p˘ atrat, M un punct oarecare pe (AB), N iar N ∈ (BC) este astfel ˆıncˆ at M N ⊥ M D. Ar˘ atat¸i c˘ a AM · 2 AB + CN · CB = DM . Ovidiu Pop, Satu Mare Gh. Sz¨ oll¨ osy, Sighetul Marmat¸iei Solut¸ie. Fie a latura p˘atratului x = AM ¸si y = CN ; atunci D C DM 2 +M N 2 = DN 2 ⇔ a2 +x2 +(a−x)2 +(a−y)2 = a2 +y 2 ⇔ 2a2 − 2ax + 2x2 − 2ay = 0 ⇔ a2 + x2 = ax + ay ⇔ DM 2 = AM · AB + CN · CB. F VII.96. Fie [AD] median˘ a ˆın △ABC, M mijlocul lui [AD], {E} = BM ∩ AC, iar punctul F pe dreapta AB este astfel ˆıncˆ at CF ∥ AD. Demonstrat¸i c˘ a punctele D, E ¸si F sunt coliniare. Mirela Marin, Ia¸si A . P Solut¸ie. Cum ADCF este trapez, M este mijlocul bazei mici, E . iar B este punctul de intersect¸ie a laturilor neparalele, ˆınseamn˘a . c˘a {P } = BM ∩ CF este mijlocul lui [CF ]. Cu reciproca teoremei M liniei mijlocii se arat˘a c˘a A este mijlocul lui [BF ], prin urmare B C D CA ¸si BP sunt mediane ˆın △BCF , iar E va fi centrul de greutate al acestui triunghi. Coliniaritatea dorit˘a rezult˘a observˆand c˘a a treia median˘a a triunghiului este F D. VII.97. Fie C1 (O1 , r1 ) ¸si C2 (O2 , r2 ), r1 < r2 , dou˘ a cercuri tangente exterior. Consider˘ am punctele A′ ∈ C1 , B ′ ∈ C2 , de aceea¸si parte a dreptei O1 O2 , astfel ˆıncˆ at A′ O1 ∥ B ′ O2 . Dac˘ a AB este tangent˘ a comun˘ a exterioar˘ a a cercurilor (A ∈ C1 , B ∈ C2 ), demonstrat¸i c˘ a dreptele AB, A′ B ′ ¸si O1 O2 sunt concurente. Romant¸a Ghit¸˘ a ¸si Ioan Ghit¸˘ a, Blaj Solut¸ie. Not˘ am {P } = AB ∩ O1 O2 , {P ′ } = A′ B ′ ∩ O1 O2 . Din asem˘an˘arile P O1 P ′ O1 r1 △P O1 A ∼ △P O2 B ¸si △P ′ O1 A′ ∼ △P ′ O2 B ′ obt¸inem c˘a = ′ = < 1. P O2 P O2 r2

B A O2

O1 B

. .

A

P

P

Astfel, punctele P ¸si P ′ se afl˘a de aceea¸si parte cu O1 pe dreapta O1 O2 ¸si ˆımpart segmentul [O1 O2 ] ˆın acela¸si raport, prin urmare P = P ′ ¸si de aici rezult˘a concurent¸a dorit˘a. VII.98. S˘ a se determine numerele naturale nenule a ¸si b, ¸stiind c˘ a sunt direct proport¸ionale cu b − 6 ¸si a ¸si invers proport¸ionale cu a + 12 ¸si b. Constantin Apostol, Rm. S˘ arat a b 2 Solut¸ie. Din datele problemei obt¸inem c˘a = ¸si a(a + 12) = b . Din prima b−6 a ecuat¸ie rezult˘a c˘a a2 − b2 = −6b, iar din a doua a2 − b2 = −12a, deci −6b = −12a. 140

Atunci b = 2a ¸si, folosind prima ecuat¸ie, g˘asim c˘a a = 4 ¸si apoi b = 8. VII.99. Fie a, b ∈ Z ¸si numerele A = 119a5 + 5b3 − 4a ¸si B = 119b5 + 5a3 − 4b. S˘ a se arate c˘ a A se divide cu 120 dac˘ a ¸si numai dac˘ a B se divide cu 120. Dan Nedeianu, Drobeta-Tr. Severin Solut¸ie. Avem c˘a A + B = 120(a5 + b5 ) − (a5 − 5a3 + 4a) − (b5 − 5b3 + 4b). Cum . a5 − 5a2 + 4a = (a − 2)(a − 1)a(a + 1)(a + 2) se divide la 120, rezult˘a c˘a A + B ..120 ¸si concluzia de impune. VII.100. Ar˘ atat¸i c˘ a 2a2 + 15b2 + 7c2 ≥ 10ab − 6ac + 20bc, ∀a, b, c ∈ R. Alexandru Negrescu, student, Ia¸si Solut¸ie. Inegalitatea se scrie: 4a2 − 4a(5b − 3c) + 30b2 + 14c2 − 40bc ≥ 0 ⇔ (2a − 5b + 3c)2 + 30b2 + 14c2 − 40bc − (5b − 3c)2 ≥ 0 ⇔ (2a − 5b + 3c)2 + 5(b − c)2 ≥ 0, evident adev˘arat. Egalitate avem pentru a = b = c. VII.101. Pentru n ∈ N∗ , not˘ am cu d (n) num˘ arul divizorilor primi ai lui n. a) Determinat¸i cardinalul mult¸imii A = {n ∈ N∗ | n ≤ 208, d (n) = 3}. b) Aflat¸i cel mai mic ¸si cel mai mare element al mult¸imii B = {k ∈ N | ∃n ∈ N∗ , n ≤ 2008, a.ˆı. d (n) = k} . Gabriel Popa, Ia¸si α3 α2 1 Solut¸ie. a) Observ˘am c˘a n ∈ A dac˘a ¸si numai dac˘a n = pα 1 · p2 · p3 , cu ∗ p1 < p2 < p3 numere prime, iar α1 , α2 , α3 ∈ N . Cum 5 · 7 · 11 = 385 > 208, ˆınseamn˘a c˘a p1 ∈ {2, 3}, prin urmare (p1 , p2 , p3 ) ∈ {(2, 3, 5); (2, 3, 7); (2, 3, 11); (2, 3, 13); (2, 3, 17); (2, 3, 19); (2, 3, 23); (2, 3, 29); (2, 3, 31); (2, 5, 7); (2, 5, 11); (2, 5, 13); (2, 5, 17); (2, 5, 19); (2, 7, 11); (2, 7, 13); (3, 5, 7); (3, 5, 11); (3, 5, 13)}. α3 α2 1 Pentru tripletele subliniate, ˆın produsul n = pα 1 · p2 · p3 vom avea obligatoriu α1 = α2 = α3 = 1; obt¸inem astfel 13 elemente ale lui A. Dac˘a (p1 , p2 , p3 ) = (2, 3, 5), ˆın n = 2α1 · 3α2 · 5α3 putem avea α3 = 1, (α1 , α2 ) ∈ {(1, 1); (2, 1); (3, 1); (1, 2); (2, 2)} sau α3 = 2, α1 = α2 = 1; g˘asim ˆınc˘a 6 elemente din A. Dac˘a (p1 , p2 , p3 ) = (2, 3, 7), ˆın n = 2α1 · 3α2 · 7α3 este obligatoriu ca α3 = 1, iar (α1 , α2 ) ∈ {(1, 1); (2, 1); (3, 1); (1, 2)}; obt¸inem 4 noi elemente ale lui A. Dac˘a (p1 , p2 , p3 ) = (2, 3, 11), atunci α3 = 1, (α1 , α2 ) ∈ {(1, 1); (2, 1); (1, 2)}, deci ˆınc˘a 3 elemente. Dac˘a (p1 , p2 , p3 ) ∈ {(2, 3, 13); (2, 3, 17); (2, 5, 7)}, vom avea ˆın fiecare caz (α1 , α2 , α3 ) ∈ {(1, 1, 1); (2, 1, 1)}, adic˘a ˆınc˘a 3 × 2 = 6 elemente. ˆIn total, |A| = 13 + 6 + 4 + 3 + 6 = 32. b) Evident c˘a min B = 0, atins pentru n = 1. Ar˘at˘am c˘a max B = 4: dac˘a ar exista n ≤ 2008 cu d(n) ≥ 5, atunci n ≥ p1 p2 p3 p4 p5 ≥ 2 · 3 · 5 · 7 · 11 = 2310 > 2008, absurd, iar pentru n = 2 · 3 · 5 · 7 avem d(n) = 4.

Clasa a VIII-a VIII.95. Pentru a, b, c ∈ R∗ , not˘ am α = num˘ arul x =

a3 b3 c3 + + ˆın funct¸ie α ¸si β. b3 c3 a3 141

a b c a c b + + , β = + + . Calculat¸i b c a c b a Elena Nicu, Malu-Mare (Dolj)

a b Solut¸ie. Fie x = , y = , z = b c xy + yz + zx 1 1 1 = + + = β, de unde xyz x y z

c ; atunci xyz = 1, x + y + z = α, iar a xy + xz + yz = β. Folosind identitatea

x3 + y 3 + z 3 − 3xyz = (x + y + z)3 − 3(x + y + z)(xy + xz + yz), rezult˘a c˘a x3 + y 3 + z 3 = α3 − 3αβ + 3. VIII.96. Rezolvat¸i ˆın numere naturale ecuat¸ia x2 + y 2 + xy = x2 y 2 . Mihail Bencze, Bra¸sov Solut¸ie. Putem presupune c˘a x ≤ y. Dac˘a x = 0, obt¸inem c˘a y = 0. Dac˘a x = 1, deducem c˘a y = −1 ∈ / N. Pentru x = 2, g˘asim c˘a 3y 2 − 2y − 4 = 0, ecuat¸ie care nu are ˆ solut¸ii naturale. In cazul ˆın care x ≥ 3, atunci y ≥ 3 ¸si vom avea c˘a x2 − 1 > x + 1, y 2 −1 > y+1, inegalit˘a¸ti care, ˆınmult¸ite, conduc la (x2 −1)(y 2 −1) > (x+1)(y+1). ˆIns˘a (x+1)(y+1) > xy+1, prin urmare (x2 −1)(y 2 −1) > xy+1, adic˘a x2 +y 2 +xy < x2 y 2 . ˆIn concluzie, singura solut¸ie a ecuat¸iei date este (x, y) = (0, 0). Solut¸ia 2. Vom rezolva ecuat¸ia ˆın mult¸imea numerelor ˆıntregi. Fie (x, y) o solut¸ie. Scriind ecuat¸ia sub forma (x + y)2 = xy(xy + 1), deducem c˘a xy(xy + 1) este p˘atrat perfect. Cum pentru xy ≥ 1, (xy)2 < xy(xy + 1) < (xy + 1)2 , iar pentru xy ≤ −2 (xy + 1)2 < xy(xy + 1) < (xy)2 , nu avem solut¸ii ˆın aceste cazuri. Rezult˘a c˘a xy = 0 sau xy = −1. Obt¸inem solut¸iile: (0, 0); (1, −1) ¸si (−1, 1). Solut¸ia 3 (Gheorghe Iurea). Fie (x, y) o solut¸ie cu x, y ∈ Z. Ecuat¸ia este echivalent˘ a cu (x + y)2 = (xy)2 + xy ⇔ 4(x + y)2 = (2xy + 1)2 − 1 ⇔ [2xy + 1 − 2(x + y)][2xy + 1 + 2(x + y)] = 1, deci 2xy + 1 − 2(x + y) = 2xy + 1 + 2(x + y) ∈ {−1, 1}. Obt¸inem solut¸iile (0, 0); (1, −1) ¸si (−1, 1). VIII.97. Fie d1 , d2 , d3 , d lungimile diagonalelor fet¸elor, respectiv diagonalei 2d2 d2 a se arate c˘ a paralelipipedul are unui paralelipiped dreptunghic. Dac˘ a d21 = 2 2 32 , s˘ √d2 + d3 d 3 o muchie de lungime cel put¸in egal˘ a cu . 3 Gheorghe Molea, Curtea de Arge¸s Solut¸ie. Fie d21 = a2 + b2 , d22 = b2 + c2 , d23 = a2 + c2 , d2 = a2 + b2 + c2 , unde a, b, c sunt lungimile muchiilor paralelipipedului. Relat¸ia din ipotez˘a se scrie succesiv: d21 =

2d22 d23 2(b2 + c2 )(c2 + a2 ) 2 2 ⇔ a + b = ⇔ a4 + b4 = 2c4 . d22 + d23 b2 + c2 + c2 + a2

ˆIns˘a 2(a4 + b4 ) ≥ (a2 + b2 )2 , deci 4c4 ≥ (a2 + b2 )2 ⇔ 2c2 ≥ a2 + b2 ⇔ 3c2 ≥ √ d 3 2 2 2 2 2 . Avem egalitate pentru a = b = c, deci ˆıntr-un a + b + c ⇔ 3c ≥ d , deci c ≥ 3 cub. VIII.98. Fie VABCD piramid˘ a patrulater˘ a regulat˘ a. Not˘ am u = m((VÛ BC),(ABC)), v = m((V Û BC) , (V CD)) ¸si t = m((V Û BC) , (V AD)). Ar˘ atat¸i c˘ a u + v + t > 180◦ . Claudiu S ¸ tefan Popa, Ia¸si Solut¸ie. Fie M ¸si N mijloacele laturilor [BC], respectiv [AD], iar P proiect¸ia 142

V Õ lui B pe V C; se arat˘a c˘a u = m(VÖ M N ), v = m(BP D), iar Ö ˆ t = m(M V N ). In △V M N avem c˘a u + u + t = 180◦

¸si atunci concluzia problemei rezult˘a dac˘a am ar˘ata c˘a v > u. Evident c˘a u < 90◦ , prin urmare dac˘a v ≥ 90◦ , demonstrat¸ia este ˆıncheiat˘ a. Presupunem c˘a v < 90◦ ; BD · P O VO atunci v > u ⇔ sin v > sin u ⇔ . Cum > 2 BP VM V O · BD V M · BC PO = , iar BP = , ultima inegali2V C VC 2 BD · V C > 1. ˆIns˘a BD2 = 2BC 2 , iar tate revine la 2BC 2 · V M V C > V M ¸si astfel solut¸ia problemei este complet˘a. 

P

. . . . D

N

A

C

M

O

B

2 VIII.99. Pentru n ∈ N∗ , consider˘ am A = 12 , 22 , 32 , . . . , n ¸i n, √ . Determinat √ ¸stiind c˘ a exist˘ a o funct¸ie f : A → A astfel ˆıncˆ at f (x) − f (y) = x − y, ∀x, y ∈ A. Cristian Laz˘ ar, Ia¸si √ √ Solut ¸ ie. Avem c˘ a f (x) − x = f (y) − y, ∀x, y ∈ A, deci ∃k ∈ R astfel ˆıncˆat √ √ f (x) − x = k, ∀x ∈ A, de unde f (x) = x + k, ∀x ∈ A. Deducem c˘a, pentru x < y din A, avem c˘a f (x) < f (y) ¸si atunci, cum A este finit˘a, rezult˘a c˘a f (12 ) = 12 , f (22 ) = 22 , . . . , f (n2 ) = n2 , de unde 1 + k = 1 ¸si n + k = n2 . Deci, k = 0 ¸si n = 1.

VIII.100. Rezolvat¸i ˆın ˆın N2 ecuat¸ia x2 − 8n + 1287 = 0. Mihai Cr˘ aciun, Pa¸scani Solut¸ie. Dac˘ a n este impar, atunci 8n = (9 − 1)n = M9 − 1. Cum x2 d˘a la . ˆımp˘art¸irea prin 9 unul dintre resturile 0, 1, 4 sau 7, iar 1287..9, cantitatea din membrul stˆang al ecuat¸iei d˘a la ˆımp˘ art¸irea prin 9 unul dintre resturile 1, 2, 5 sau 8 ¸si se ajunge la o contradict¸ie. Dac˘a n = 2k, ecuat¸ia se scrie: (8k − x)(8k + x) = 32 · 11 · 13. Analizˆand pe rˆand cazurile care se obt¸in, g˘asim solut¸ie doar cˆand 8k − x = 11, 8k + x = 32 · 13, cˆand vom avea k = 2, x = 53. Deci, singura solut¸ie a ecuat¸iei date este (n, x) = (4, 53). VIII.101. Se calculeaz˘ a suma cifrelor pentru fiecare dintre numerele de la 1 la n, n > 10. Pentru fiecare sum˘ a dintre cele n se calculeaz˘ a din nou suma cifrelor, repetˆ andu-se aceast˘ a operat¸ie pˆ an˘ a cˆ and obt¸inem n numere formate din cˆ ate o singur˘ a cifr˘ a. S˘ a se afle n, ¸stiind c˘ a ˆın mult¸imea astfel obt¸inut˘ a cifrele 1, 2, 3 ¸si 4 se repet˘ a de cˆ ate 101 ori fiecare, iar cifrele 5, 6, 7, 8 ¸si 9 de cˆ ate 100 ori fiecare. Mihai Haivas, Ia¸si Solut¸ie. Restul ˆımp˘art¸irii unui num˘ar prin 9 este egal cu restul ˆımp˘art¸irii sumei cifrelor sale prin 9. Atunci, cifra 1 se obt¸ine din acele numere care dau restul 1 la ˆımp˘art¸irea prin a din numerele 1, 10, 19 . . . . Pentru a obt¸ine 1 de 101 ori, h n9,i adic˘ trebuie s˘a avem + 1 = 101, deci n ∈ {900, 901, . . . , 904}. Se verific˘a faptul c˘a 9 doar n = 904 satisface ¸si celelalte condit¸ii din ipotez˘a.

Clasa a IX-a

a ax2 + bx + c ≤ p, ∀x ∈ [−1, 1], atunci IX.91. Fie a, b, c, p ∈ R, p > 0. Dac˘ cx2 + bx + a ≤ 2p, ∀x ∈ [−1, 1].

Dorin M˘ arghidanu, Corabia 143

Solut¸ie. Fie f (x) = ax2 + bx + c, g(x) = cx2 + bx + a, x ∈ [−1, 1]. Din ipotez˘a se obt¸ine c˘a |a + b + c| = |f (1)| ≤ p, |a − b + c| = |f (−1)| ≤ p, iar |c| = |f (0)| ≤ p. Folosind inegalitatea modulului, avem: x+1 1−x + (a − b + c) · |≤ |g(x)| = |cx2 + bx + a| = |c(x2 − 1) + (a + b + c) · 2 2 |x + 1| |1 − x| ≤ |c| · |x2 − 1| + |a + b + c| · + |a − b + c| · ≤ 2 2  ‹  ‹ |x + 1| |1 − x| x+1 1−x ≤ p · |x2 − 1| + + + = p 1 − x2 + = 2 2 2 2 = p(2 − x2 ) ≤ 2p, ∀x ∈ [−1, 1]. IX.92. Fie n ∈ N, n ≥ 3, iar α, β ∈ R astfel ˆıncˆ at n + α + β ̸= 0. Ar˘ atat¸i c˘ a X (1 + α) · · · (n + α) i+1 − (1 + α) · · · (n − 1 + α) + (−1) (1 + α) · · · (n − i − 1 + α)× n+α+β i=1 n−2

n

×β (β + 1) · · · (β + i − 1) + (−1) β · · · (β + n − 2) = (−1)

n

β (β + 1) · · · (β + n − 1) . n+α+β

Gheorghe Costovici, Ia¸si Solut¸ie. Not˘ am cu Sn membrul stˆang al egalit˘a¸tii. Pentru n = 3, avem c˘a S3

=

(1 + α)(2 + α)(3 + α) − (1 + α)(2 + α) + (1 + α) · β − 3+α+β β(β + 1)(β + 2) , −β(β + 1) = . . . = − 3+α+β

calculele fiind de rutin˘a. Fie n ≥ 4, iar σn−2 = Sn − (−1)n β(β + 1) . . . (β + n − 2). Ar fi suficient s˘a dovedim c˘a 1 (∗) σk = (−1)k−1 (1+α) . . . (n−k−1+α)β(β+1) . . . (β+k)· , ∀k ∈ 1, n − 2 n+α+β odat˘a demonstrat˘a (∗), vom obt¸ine c˘a Sn

= σn−2 + (−1)n β(β + 1) . . . (β + n − 2) =  ‹ 1+α = (−1)n β(β + 1) . . . (β + n − 2) 1 − = n+α+β β(β + 1) . . . (β + n − 1) = (−1)n · , n+α+β

adic˘a tocmai ceea ce trebuia demonstrat. Justfic˘ am (∗) prin induct¸ie dup˘a k: σ1

(1 + α) . . . (n + α) − (1 + α) . . . (n − 1 + α) + (1 + α) . . . (n − 2 + α)β = n+α+β −β = (1 + α) . . . (n − 1 + α) · + (1 + α) . . . (n − 2 + β) · β = n+α+β (1 + α) . . . (n − 2 + α)β(β + 1) , = n+α+β

=

144

deci (∗) este adev˘arat˘ a pentru k = 1. Presupunem (∗) adev˘arat˘a pentru k ∈ {1, 2, . . . , n− 3}; atunci σk+1

= σk + (−1)k (1 + α) . . . (n − k − 2 + α)β(β + 1) · . . . · (β + k) =  ‹ n−k−1+α k = (−1) (1 + α) . . . (n − k − 2 + α)β · . . . · (β + k) 1 − = n+α+β 1 = (−1)k (1 + α) . . . (n − k − 2 + α)β · . . . · (β + k + 1) · , n+α+β

deci (∗) este adev˘arat˘ a ¸si pentru k + 1 ¸si astfel solut¸ia problemei este complet˘a. b = 90◦ ¸si AB = 3 , iar D mijlocul lui IX.93. Fie △ABC dreptunghic cu m(A) AC 2 [AC]. Not˘ am cu E punctul de intersect¸ie a cercurilor C1 (A, AD) ¸si C2 (B, BC), aflat Õ de aceea¸si parte a dreptei AB ca ¸si punctul C. Determinat¸i m˘ asura unghiului CAE. C˘ at˘ alin T ¸ ig˘ aeru, Suceava Solut¸ie. Fie {M, N } = AB ∩ C , cu N ∈ (AB), iar a = AD; atunci AC = 2a, 1 √ √ AB = 3a, BC = a 13, BD = a 10, iar BM = 4a. C Cum BD < BC < BM, ˆınseamn˘ a c˘a cercurile C1 ¸si Õ < 180◦ . Aplic˘ C2 sunt secante, iar 90◦ < m(BAE) am D E teorema cosinusului ˆın △ABE : BE 2 = AB 2 +AE 2 − Õ ⇔ 13a2 = 9a2 + a2 − 2 · 3a · a · 2AB · AE · cos(BAE) A N B Õ de unde obt¸inem cos(BAE) Õ = − 1 , adic˘ cos(BAE), a M 2 Õ = 120◦ . Deducem c˘ Õ = 120◦ − 90◦ = 30◦ . m(BAE) a m(CAE) IX.94. ˆ In △ABC, I este centrul cercului ˆınscris, iar {M } = AI∩BC. Demonstrat¸i Ö c˘ a bisectoarea unghiului AM C, BI ¸si AC sunt trei drepte concurente dac˘ a ¸si numai ◦ b dac˘ a m(A) = 120 . Vlad Emanuel, student ¸si Andrei Cozma, elev, Bucuret¸i Solut¸ie. Not˘am cu a, b, c lungimile laturilor ¸si fie {P } = BI ∩ AC; avem c˘a 2bc A ab AP c AM = ·cos , M C = , iar = . Atunci: A b+c 2 b+c PC a Ö BI, AC ¸si bisectoarea unghiului AM C sunt concurente P AP AM Ö I ⇔ M P este bisectoarea lui AM C ⇔ = ⇔ PC MC c 2c A A 1 b = 120◦ . = cos ⇔ cos = ⇔ m(A) a a 2 2 2 M B C IX.95. Dac˘ a xi ∈ [0, a], i = 1, n ¸si xn+1 = x1 , demonstrat¸i c˘ a

..

.

n X

xi+1 (a − xi )

: d(a + d + 1) − (ad − bc) = 1

(1) (2) (3) (4)

Din ecuat¸iile (1) ¸si (4) rezult˘a c˘a (a + d + 1)(a − d) = 0 ¸si cum a + d + 1 ̸= 0, 1 1 atunci d = a. Din ecuat¸iile (2) ¸si (3) obt¸inem c˘a b = c = = . a+d+1 2a + 1 1 Folosind acum ecuat¸ia (1), avem c˘a a(2a + 1) − (a2 − ) = 1, echivalent cu (2a + 1)2 3 1 a(a + 1)(4a2 + 4a − 3) = 0, deci a ∈ {− , , −1, 0}. G˘asim solut¸iile 2 2  ‹  ‹  ‹  ‹ 1 1 1 1 3 1 0 1 −1 −1 X1 = , X2 = , X3 = , X4 = − . 1 0 −1 −1 2 1 1 2 1 3 XI.93. Studiat¸i convergent¸a ¸sirului (un )n≥1 definit prin u1 ≥ 0, un+1 = ∀n ∈ N∗ .

un + 1 , u2n + 1

Gheorghe Costovici ¸si Adrian Corduneanu, Ia¸si Solut¸ie. Vom ar˘ata c˘a (un ) este convergent c˘atre 1. Dac˘a u1 ∈ {0, 1}, afirmat¸ia este imediat˘a. Cazurile u1 ∈ (0, 1) ¸si u1 ∈ (1, ∞) tratˆandu-se asem˘an˘ator, ne vom fixa atent¸ia asupra primului. Evident c˘a un > 0, ∀n ∈ N ¸si se arat˘a u¸sor prin induct¸ie c˘ a u2n−1 < 1 ¸si u2n > 1, ∀n ∈ N∗ . Vom demonstra acum c˘a u2n+2 < u2n ¸si 148

u2n+1 > u2n−1 , ∀n ∈ N∗ ; efectu˘am calculele doar pentru prima afirmat¸ie: u2n+2 < u2n

u2n+1 + 1 < u2n ⇔ ⇔ 2 u2n+1 + 1



u2n + 1 +1 u22n + 1

‹ Á 

u2n + 1 u22n + 1

‹2



+1

u42n + u32n + 3u22n + u2n + 2 < u2n ⇔ u52n − u42n + 2u22n − u42n + 3u22n + 2u2n + 2 −u22n + u2n − 2 > 0 ⇔ (u2n − 1)(u42n + 2u22n + u2n + 2) > 0,

< u2n ⇔

adev˘arat. ˆIn concluzie, sub¸sirul (u2n−1 )n≥1 este cresc˘ator ¸si m˘arginit, iar sub¸sirul (u2n )n≥1 este descresc˘ator ¸si m˘arginit inferior de 1; ˆınseamn˘a c˘a ambele sunt convergente spre α, respectiv β. Trecˆ and la limit˘a ˆın relat¸ia de recurent¸˘a, obt¸inem c˘a β+1 α+1 ˆ α= 2 , iar β = 2 . Inlocuind, g˘asim pentru α ecuat¸ia α5 − α4 + 2α3 − α2 + β +1 α +1 α − 2 = 0, deci (α − 1)(α4 + 2α2 + α + 2) = 0 ¸si, cum α > 0, ˆınseamn˘a c˘a α = 1. Deducem apoi c˘a β = 1, prin urmare lim un = 1. n→∞

Not˘ a. ˆIn aceea¸si manier˘a, autorii problemei au stabilit c˘a pentru orice a, b ∈ uan + b (0, ∞), ¸sirul (un )n≥1 definit prin u1 ≥ 0, un+1 = a+1 este convergent c˘atre 1. un + b XI.94. S˘ a se demonstreze c˘ a pentru orice n ∈ N∗ , exist˘ a numerele distincte  4 n x1 , x2 , . . . , xn ∈ (1, 2), a¸sa ˆıncˆ at x1 x2 · · · xn = . e Dan Pl˘ ae¸su , Ia¸si Solut¸ia 1 (a autorului). Consider˘am funct¸ia f : [1, 2]•→ R, f (x) = x ln˜x − x. k−1 k Aplicˆand teorema lui Lagrange funct¸iei f pe intervalele 1 + ,k = ,1 + n n ‹  ‹  k k−1 k , k = 1, 2, . . . , n, astfel ˆıncˆat f 1 + − 1, 2, . . . , n, determin˘am xk ∈ 1 + ,1 + n n n  ‹ k−1 1 f 1+ = f ′ (xk ), k = 1, 2, . . . , n. Rezult˘a c˘a f ′ (x1 ) + f ′ (x2 ) + . . . + f ′ (xn ) = n n ‹  ‹‹ n   P k k−1 f 1+ −f 1+ = f (2) − f (1). Folosind faptul c˘a f ′ (x) = ln x n· n n k=1 4 ¸si c˘a f (2) − f (1) = ln , din ultima relat¸ie obt¸inem concluza problemei. e Solut¸ia 2 (Gabriel Popa). Construct¸ia ¸sirului (xn )n≥1 se poate face inductiv: 4 lu˘am x1 = ∈ (1, 2), iar dac˘a presupunem existent¸a numerelor x1 , x2 , . . . , xn ∈ (1, 2) e  ‹n 4 4 , atunci numerele x1 , . . . , xn−1 , kn xn ¸si , unde pentru care x1 x2 . . . xn = e k e  ‹n+1n 4 (kn )n≥1 este un ¸sir de numere din (1, 2) cu limita 1, au produsul ¸si pot fi e f˘acute distincte, alegˆand convenabil kn (posibil, ˆıntrucˆat orice vecin˘atate la dreapta a lui 1 este nenum˘ arabil˘ a). Solut ¸ ia 3 (Gheorghe Iurea). C˘aut˘am numerele xk , k = 1, n de forma xk =   ‹a k ‹ 3 4 cu ak ∈ 0, , k = 1, n. Condit¸ia problemei devine a1 + a2 + . . . + an = n. e 2 149

1 1 1 1 1 Pentru n = 2p putem alege numerele 1 − , 1 + , 1 − , . . . , 1 − ,1+ , iar 3 3 4 p+2 p+2 1 1 1 1 1 pentru n = 2p + 1 putem alege numerele 1, 1 − , 1 + , 1 − , . . . , 1 − , 1+ . 3 3 4 p+2 p+2 



 1   1  2α 1  nα XI.95. Calculat¸i lim 1 + α + 1 + α + ··· + 1 + α − n , unde n→∞ n n n α ≥ 1 este fixat. (ˆ In leg˘ atur˘ a cu L83 din RecMat-1/2005.) Marius Olteanu, Rm. Vˆ alcea  ‹ 1 1 kα > Solut¸ie. Not˘am cu (xn )n≥1 ¸sirul a c˘arui limit˘a o c˘aut˘am. Cum 1 + α n 1, ∀k = 1, n, deducem c˘a xn > 0, ∀n ∈ N∗ . Din inegalitatea lui Bernoulli, obt¸inem c˘a  ‹ 1  ‹ 1 kα 1 1 1 1 1 1+ α < 1 + α · α , deci xn < α 1 + α + . . . + α , ∀n ∈ N∗ . Din teon n k  n n ‹2 1 1 1 1 1 · . rema Cesar`o-Stolz, lim α 1 + α + . . . + α = lim α n→∞ n n→∞ (n + 1) 2 n (n + 1)α − nα Dac˘a vom dovedi c˘a aceast˘a din urm˘a limit˘a este 0, conform criteriului cle¸stelui rezult˘a c˘a lim xn = 0. 1

1

n→∞

Aplicˆ and teorema lui Lagrange funct¸iei f : [n, n+1] → (0, ∞), f (x) = xα , deducem 1 1 1 c˘a (n + 1)α − nα = α · cα−1 , cu c ∈ (n, n + 1). Atunci · < · α (n + 1)2α−1 (n + 1)α 1 1 1 < · , ceea ce, prin trecere la limit˘a, conduce la faptul (n + 1)α − nα α (n + 1)α · nα−1 1 1 c˘a lim · = 0. n→∞ (n + 1)α (n + 1)α − nα

Clasa a XII-a

Z

XII.91. Prove that

1

x

(1 + x) ex(1+e ) dx = ee − 1.

0

Zdravko Starc, Vr˘ sac, Serbia Z

Z

e

x

0

e dt = e = e − 1. t

0

e t

1

(1+x)ex(1+e ) dx =

Solut¸ie. Facem substitut¸ia xex = t; atunci (x+1)ex dx = dt ¸si e

0

XII.92. Fie b > a > 0, iar f : [a, b] → R o funct¸ieZ continu˘ a pe [a, b] ¸si derivabil˘ a c f (x) dx = c (b − c) f (c). pe (a, b); s˘ a se arate c˘ a exist˘ a c ∈ (a, b) astfel ˆıncˆ at b a

Dan Nedeianu, Drobeta Tr.Z Severin x−b x Solut¸ie. Aplic˘am teorema lui Rolle funct¸iei g : [a, b] → R, g(x) = f (t)dt x a Z x b x−b ¸si ¸tinem cont c˘a g ′ (x) = 2 f (x). f (t)dt + x a x Z

π 2

2 cos 1 sin x dx ≤ . x c 1 Constantin Micu, Meline¸sti (Dolj)

XII.93. Demonstrat¸i c˘ a exist˘ a c ∈ (2, π) pentru care

150

h πi Solut¸ie. Se constat˘a u¸sor c˘a funct¸ia f : 1, → R, f (x) = sin x este strict 2 h πi 1 cresc˘atoare, iar g : 1, → R, g(x) = este strict descresc˘atoare; atunci, conform 2 x inegalit˘a¸tii lui Cebˆı¸sev,

π 2

Z

1 · −1

Z

π 2

1

sin x 1 ≤€ Š2 · x π 2 −1

Z 1

π 2

Z

sin xdx · 1

π 2

1 dx, x

π 2

sin x ln π − ln 2 dx ≤ 2 cos 1 · . Aplicˆand teorema lui Lagrange funct¸iei x π−2 1 ln π − ln 2 1 = ¸si astfel h : [2, π] → R, h(x) = ln x, g˘asim c ∈ (2, π) pentru care c π−2 solut¸ia problemei este complet˘a. Z 2n xa + b √ XII.94. Calculat¸i lim n dx, unde a ∈ (0, ∞) ¸si b ∈ R. n→∞ x2a+4 + 1 n Liviu Smarandache, Craiova Z x xa + b ta + b √ Solut¸ie. Fie f : [0, ∞) → R, f (x) = √ ¸si F (x) = dt, 2a+4 + 1 x2a+4 + 1  t‹ 0 Z x a t +b 1 1 b 1 1 x ≥ 0. Cum 0 ≤ F (x) ≤ , obt¸inem dt = − + − a+2 a+1 t x 2x a + 1 x (2x)a+1 0 c˘a lim F (x) = 0. Atunci de unde

x→∞

Z

ta + b F (2x) − F (x) dt = lim x(F (2x) − F (x)) = lim = 1 2a+4 x→∞ x→∞ x→∞ t +1 x x 1 1 1 2f (2x) − f (x) = lim (x2 f (x) − (2x)2 f (2x)) = 1 − = , = lim 1 x→∞ x→∞ 2 2 2 − x2 lim x

2x



xa+2 + bx2 1 deoarece lim x2 f (x) = lim √ = 1. Prin urmare, limita cerut˘a este . 2a+4 x→∞ x→∞ 2 x +1 XII.95. Fie (A, +, ·) un inel ˆın care 0 ̸= 1 ¸si 1 + 1 + 1 + 1 + 1 = 0. S˘ a se arate c˘ a, dac˘ a x3 y 2 = y 2 x3 , ∀x, y ∈ A, atunci inelul este comutativ. I.V. Maftei, Bucure¸sti ¸si Mihai Haivas, Ia¸si Solut¸ie. Fie x, y ∈ A, arbitrare. Din x3 y 2 = y 2 x3 ¸si x3 (y + 1)2 = (y + 1)2 x3 , g˘asim c˘a x3 (y + y) = (y + y)x3 . Cum 1 + 1 + 1 + 1 + 1 = 0, rezult˘a c˘a 5y = 0, deci x3 (5y) = (5y)x3 . Deducem c˘a x3 (3y) = (3y)x3 , prin urmare x3 y = yx3 . Folosind aceast˘a relat¸ie pentru x ¸si x + 1, obt¸inem x3 y = yx3 ¸si (x + 1)3 y = y(x + 1)3 , de unde (3x2 + 3x + 1)y = y(3x2 + 3x + 1). ˆInlocuind pe x cu x + 1 ¸si ¸tinˆand cont c˘a 3(x+1)2 +3(x+1)+ 1−(3x2 +3x+1) = 6x+6 = x+1, rezult˘a c˘a (x+ 1)y = y(x+1), deci xy = yx. Cum x, y sunt arbitrare din A, concluzia problemei se impune.

151

Solut¸iile problemelor pentru preg˘ atirea concursurilor propuse ˆın nr. 2/2008 A. Nivel gimnazial G146. Fie x, y, z ∈ (0, ∞) astfel ˆıncˆ at xyz = 1. Ar˘ atat¸i c˘ a yz 3 zx3 xy 3 + + ≥ 1. x4 + y + z y4 + z + x z4 + x + y Liviu Smarandache ¸si Lucian Tut¸escu, Craiova Solut¸ie. Avem c˘a x3 + y 3 ≥ xy(x + y), cu egalitate pentru x = y. Atunci X

X X X xy 3 xy 3 y3 x3 = = ≥ = 1. x4 + y + z x4 + xyz(y + z) x3 + yz(y + z) x3 + y 3 + z 3

Egalitatea se atinge pentru x = y = z = 1. G147. Fie n ∈ N, n ≥ 2, fixat, iar a, b,h c sunt numere naturale astfel ˆıncˆ at n − 1i 2 na + (n + 1) b + 2nc = n + 1. Ar˘ atat¸i c˘ a n− ≤ a + b + c ≤ n. 2 Gheorghe Iurea, Ia¸si . Solut¸ie. Cum b − 1 = n(n − a − b − 2c), ˆınseamn˘a c˘a b − 1..n. ˆIns˘a b < n, altfel

na + (n + 1)b + 2nc > n2 + 1 ¸si atunci b − 1 = 0, deci b = 1. Condit¸ia din enunt¸ devine na + n + 1 + 2nc = n2 + 1 ⇔ a + 2c = n − 1 ⇔ a + b + c = n − c. Suma a + b + c este maxim˘a cˆand c este minim, adic˘a pentru c = 0; deducem c˘a (a + b + c)max = n, maxim atins cˆand a = n •− 1, b = and ˜ 1, c = 0. Suma a + b + c este minim˘ • a cˆ ˜ c este n−1 n−1 maxim, deci pentru c = ; obt¸inem c˘a (a + b + c)min = n − , minim 2 • 2 ˜ • ˜ n−1 n−1 atins cˆand a = n − 1 − 2 , b = 1, c = . 2 2 G148. Fie a1 a2 . . . ap ∈ N. S˘ a se arate c˘ a orice num˘ ar natural are un multiplu de forma a1 a2 . . . ap a1 a2 . . . ap . . . a1 a2 . . . ap 0 . . . 0. Marian Pant¸iruc, Ia¸si Solut¸ie. Fie n ∈ N; consider˘am numerele: a1 a2 . . . ap , a1 a2 . . . ap a1 a2 . . . ap , . . . , a1 a2 . . . ap a1 a2 . . . ap . . . a1 a2 . . . ap , ˆın num˘ar de (n + 1). Prin ˆımp˘art¸irea acestora la n obt¸inem (n + 1) resturi ¸si, cum resturile posibile sunt 0, 1, . . . , n − 1, rezult˘a c˘a cel put¸in dou˘a resturi sunt egale. Fie a = a1 a2 . . . ap . . . a1 a2 . . . ap (format din k1 numere a1 a2 . . . ap ¸si b = a1 a2 . . . ap . . . a1 a2 . . . ap (format din k2 numere a1 a2 . . . ap ) dou˘a dintre numerele de mai sus, care dau acela¸si rest la ˆımp˘art¸irea cu n. Diferent¸a acestora se divide cu n, deci a−b = a1 a2 . . . ap a1 a2 . . . ap . . . a1 a2 . . . ap 00 . . . 0 verific˘a cerint¸a problemei. G149. a) Determinat¸i dou˘ a numere prime p, q astfel ˆıncˆ at p < q, iar p2 − 1 are 2 mai mult¸i divizori naturali decˆ at q − 1. b) Determinat¸i toate numerele prime p pentru care p2 − 1 are exact opt divizori naturali. Dan Popescu, Suceava 152

Solut¸ie. a) De exemplu, putem lua p = 19, q = 23. b) Dac˘a p ̸= 2, atunci p2 ≡ 1 (mod 8). Dac˘a p ̸= 3, atunci p2 ≡ 1 (mod 3). Cum (3, 8) = 1, ˆınseamn˘ a c˘a dac˘a p ∈ / {2, 3}, atunci p2 ≡ 1 (mod 24), prin urmare . p2 − 1..24. Pentru p ≥ 7, avem, c˘a p2 − 1 > 24 ¸si, cum 24 are opt divizori naturali,

p2 − 1 va avea mai mult de opt divizori. Pentru p ∈ {2, 3}, p2 − 1 are mai put¸in de opt divizori. Dac˘a p = 5, atunci p2 − 1 = 24 are exact opt divizori, deci singurul num˘ar care satisface condit¸iile din enunt¸ este 5. G150. Fie m ¸si n numere naturale nenule cu proprietatea c˘ a m ≤ 1 + 2 + · · · + n. S˘ a se arate c˘ a m poate fi scris ca suma cˆ atorva numere distincte dintre 1, 2,. . . , n. Marian Tetiva, Bˆ arlad § ª n(n + 1) Solut¸ia 1 (Dan Mocanu, elev, Ia¸si). Acoperim mult¸imea 1, 2, 3, . . . , 2 cu urm˘atoarele sume avˆ and termenii distinct¸i: 1, 2, 3, . . . , n, n+1, n+2, . . . , n+(n−1), n + (n − 1) + 1, n + (n − 1) + 2, . . . , n + (n − 1) + (n − 2), . . . , n + (n − 1) + . . . + 1; concluzia problemei este acum imediat˘a. Solut¸ia 2 (a autorului). Demonstr˘am prin induct¸ie dup˘a n. Pentru n = 1, deci m = 1, nu avem nimic de demonstrat; de asemenea se verific˘a u¸sor cazul n = 2 (deci m = 1, 2 sau 3). Vom presupune mai departe n ≥ 2 ¸si c˘a afirmat¸ia este adev˘arat˘a pentru n − 1 ¸si o demonstr˘am pentru n. Dac˘a m este unul dintre numerele 1, 2, . . . , n nu avem ce ar˘ata. Dac˘a m ≥ n + 1, avem c˘a 1 ≤ m − n ≤ 1 + 2 + . . . (n − 1) ¸si, conform ipotezei de induct¸ie, m − n este suma unor termeni distinct¸i din mult¸imea {1, 2, . . . , (n − 1)}. Rezult˘a c˘a m este suma dintre ace¸sti termeni ¸si n, ceea ce ˆıncheie demonstrat¸ia. G151. Bazele unei prisme sunt poligoane cu 2008 vˆ arfuri. Numerot˘ am cu 1, 2,. . . , 2008 vˆ arfurile bazei inferioare ¸si, corespunz˘ ator, cu a1 , a2 ,. . . , a2008 vˆ arfurile bazei superioare, unde {a1 , a2 , . . . , a2008 } = {1, 2, . . . , 2008}. a) Demonstrat¸i c˘ a putem g˘ asi o numerotare pentru baza superioar˘ a astfel ˆıncˆ at . i + ai .. 8, ∀i ∈ {1, 2, . . . , 2008}. b) Demonstrat¸i c˘ a nu putem g˘ asi o numerotare pentru baza superioar˘ a astfel ˆıncˆ at .. i + ai . 9, ∀i ∈ {1, 2, . . . , 2008}. Gabriel Popa ¸si Gheorghe Iurea, Ia¸si Solut¸ie. a) De exemplu, putem lua a1 = 2007, a2 = 2006, . . . ,a2007 = 1, a2008 = 2008. . b) Dac˘a ar exsta o numerotare pentru care i + ai ..9, ∀i ∈ {1, 2, . . . , 2008}, atunci . . (1 + a1 ) + (2 + a2 ) + . . . + (2008 + a2008 )..9, prin urmare 2(1 + 2 + . . . + 2008)..9. Am . obt¸ine astfel c˘a 2008 · 2009..9, contradict¸ie. ˆ triunghiul isoscel ABC (AB = AC) not˘ G152. In am cu B ′ , C ′ picioarele ˆın˘ alt¸i′ ′ milor din B, respectiv C. Dac˘ a AB = 2 B C , s˘ a se determine unghiurile triunghiului. Nela Ciceu, Bac˘ au ¸si Titu Zvonaru, Com˘ ane¸sti b este ascut¸it sau obtuz. Solut¸ie. Vom analiza dou˘a cazuri, dup˘a cum unghiul A a) Fie M mijlocul laturii [AB]; atunci [B ′ M ] va fi median˘a ˆın △ABB ′ 153

1 1 AB. Avem ¸si c˘a B ′ C ′ = AB, 2 2 ′M B ≡ B ′ C ′ A. ˆ ′C ′A ≡ Ö Ö Ö prin urmare B ′ C ′ = B ′ M , deci B Ins˘a B ′ M C ′ ) = 180◦ − Ò (deoarece B ′ C ′ ∥BC) ¸si atunci m(B) Ò = m(B × B M ◦ ◦ ′ Ö b b 2m(M BB ) = 180 −2[90 −m(A)] = 2m(A), relat¸ie care ˆımpreun˘a C b Ò = 180◦ conduce la m(A) b = 36◦ , m(B) Ò = m(C) Ò = cu m(A)+2m( B) 72◦ . B b este obtuz, atunci △B ′ M A va fi isoscel ¸si b) Dac˘a A B cu un rat¸ionament asem˘an˘ator celui de mai sus obt¸inem A ′ C ′ M ) = 180◦ − 2m(B ′ AB) = 180◦ − 2 · Ò = m(B × Ö c˘a m(B) Ò = 180◦ − 4m(B), Ò prin urmare m(B) Ò = m(C) Ò = 36◦ , 2m(B) M ◦ b iar m(A) = 108 .

A

dreptunghic ¸si deducem c˘a B ′ M =

B

B C C

C

ˆ triunghiul ABC, M este mijlocul laturii [BC], m(ABC) Õ = 30◦ ¸si G153. In ◦ Õ m(ACB) = 105 . Perpendiculara din C pe AM taie AB ˆın Q. Calculat¸i valoarea QA raportului . QB Neculai Roman, Mirce¸sti (Ia¸si) Solut¸ie. Ducem CE⊥AB, E ∈ (AB). Folosind triunghiul CEM echilateral ¸sitriunghiul AEC dreptunghic isoscel, g˘asim c˘a M E = EC = EA, deci triunghiul Ö ) = 15◦ , deci m(M Ö AEM este isoscel cu m(AEM ) = 150◦ . Atunci m(EAM AC) = 30◦ Ö Õ = 45◦ ¸si m(ACQ) Õ = 60◦ . Obt¸inem c˘ ¸si m(AM C) = 45◦ . Prin urmare, m(BCQ) a QA AACQ BC · CQ sin BCQ AC sin 60◦ = = = . QB ABCQ BC sin 45◦ Õ AC · CQ √sin ACQ √ √ BC 2 QA 3 Dar AC = EC 2 = ¸si atunci = . 2 QB 2 G154. Fie D mijlocul laturii [BC] a triunghiului echilateral ABC de latur˘ a 1, iar P un punct mobil pe [CD]. Not˘ am cu M ¸si N proiect¸iile pe AP ale punctelor B, respectiv C. Aflat¸i aria locului geometric descris de segmentul [M N ]. Marius Olteanu, Rm. Vˆ alcea Solut¸ie. Observˆand c˘a patrulaterele ABDM A ¸si ACN D sunt inscriptibile, deducem c˘a punctele M ¸si N se afl˘a pe cercurile C1 (circumscris triunghiului ABD), respectiv C2 (circumscris triE F unghiului ACD). Dac˘a E este mijlocul laturii ÷ al [AC], se constat˘a c˘a M parcurge arcul mic DE ÷ M P cercului C1 , ˆın timp ce N parcurge arcul mic CD C2 al cercului C2 . Astfel, locul geometric m˘aturat de C D N [M N ] este suprafat¸a ha¸surat˘a ˆın figur˘a. Observ˘am B C 1 c˘a segmentele de disc m˘arginite de C1 ¸si [DE], respectiv de C2 ¸si [CD], sunt congruente; atunci aria locului geometric va fi egal˘a cu √ 3 . aria triunghiului echilateral CDE, adic˘a 16 154

G155. Fie C cercul circumscris △ABC ascut¸itunghic. Not˘ am cu P punctul de intersect¸ie al tangentelor duse la cerc ˆın B ¸si C, {D} = AP ∩ C, iar M ¸si N sunt ⌢



mijloacele arcului mic BC, respectiv arcului mare BC. S˘ a se arate c˘ a dreptele AM , DN ¸si BC sunt concurente. Gabriel Popa, Ia¸si Õ rezult˘ Solut¸ie. Fie {T } = AM ∩BC. Cum AM este bisectoarea unghiului BAC, a c˘a N BT AB (1) = . A TC AC C Õ atunci Vom ar˘ata c˘a DT este bisectoare pentru BDC; D, T, N vor fi coliniare ¸si de aici concluzia problemei. Õ ≡ P Õ Cum BAP BD, rezult˘a c˘a △P AB ∼ △P BD, PB AB T deci = . Analog se arat˘a c˘a △P AC ∼ △P CD, C B PD BD PC AC ˆ AB de unde = . Ins˘a P B = P C, prin urmare = PD CD BD D M AC AB BD ⇔ = . T ¸ inˆand cont de (1), obt¸inem c˘a CD AC CD BT BD Õ ceea ce = , adic˘a DT este bisectoarea lui BDC, TC DC ˆıncheie rezolvarea. P

. ..

B. Nivel liceal ˆ plan se consider˘ L146. In a dreptele d1 , d2 ,. . . , dn+1 , oricare dou˘ a neparalele. ◦ Ø Not˘ am cu αk = m(d , d ), α ≤ 90 , k = 1, n. Pe d se consider˘ a un segk k+1 k 1 ment de lungime 2 care se proiecteaz˘ a pe d2 , apoi segmentul obt¸inut se proiecteaz˘ a pe d3 ¸si tot a¸sa, pˆ an˘ a cˆ and pe d se obt ¸ ine un segment de lungime 1. S ¸ tiind c˘ a € ¦ ©Š È √ n+1 n tg min αi | i = 1, n = 4 − 1, determinat¸i unghiurile αk , k = 1, n. Cristian S˘ avescu, student, Bucure¸sti Solut¸ie. Fie A1 B1 segmentul de lungime 2 de pe d1 , iar Ak = prdk Ak−1 , Bk = prdk Bk−1 , k = 2, n + 1. Cum Ak+1 Bk+1 = Ak Bk · cos αk , atunci An+1 Bn+1 = 1 A1 B1 cos α1 cos α2 . . . cos αn , de unde cos α1 cos α2 . . . cos αn = . Fie αp = min{αi |i = 2  πi 1, n}; cum cosinusul este descresc˘ator pe 0, , avem cos αp ≥ cos αk , ∀k = 1, n ¸si 2 È√ n È 4−1 1 1 √ astfel cosn αp ≥ , deci cos αp ≥ √ , apoi sin αp = 1 − cos2 αp ≤ , prin n n 2È 2 2 √ n urmare tg αp ≤ 4 − 1. Conform ipotezei, rezult˘a c˘a se atinge egalitatea; acest lucru are loc pentru cos α1 = cos α2 . . . = cos αn , deci cˆand α1 = α2 = . . . = αn = 1 . arccos √ n 2 L147. Se consider˘ a un poligon convex cu n laturi, n ≥ 4, avˆ and proprietatea c˘ a oricare dou˘ a diagonale nu sunt paralele ¸si oricare trei nu sunt concurente ˆın puncte diferite de vˆ arfurile poligonului. Se noteaz˘ a cu ni num˘ arul punctelor de intersect¸ie a 155

diagonalelor interioare poligonului ¸si cu ne cel al punctelor de intersect¸ie exterioare poligonului. a) S˘ a se arate c˘ a exist˘ a exact opt poligoane care verific˘ a relat¸ia ni > ne . b) S˘ a se arate c˘ a exist˘ a exact trei poligoane pentru care ni + ne = kn2 , k ∈ N∗ . Mihai Haivas, Ia¸si Solut¸ie. Fiecare punct interior de intersect¸ie a diagonalelor este unic determinat de cele dou˘a diagonale ce-l cont¸in, deci de patru vˆarfuri ale poligonului; rezult˘a c˘a n(n − 3) diagonale, care se intersecteaz˘a ˆın C 2n(n−3) puncte, ni = Cn4 . Cum sunt 2 2 2 fiecare vˆarf al poligonului fiind num˘arat de Cn−3 ori (se obt¸ine ca intersect¸ie a oricare dou˘a diagonale care trec prin acel vˆarf), obt¸inem c˘a num˘arul total de intersect¸ii, f˘ar˘a n(n − 3)(n2 − 7n + 14) 2 vˆarfuri, este ni + ne = C 2n(n−3) − nCn−3 = . Prin urmare, 8 2 n(n − 3)(n − 4)(n − 5) nl = . 12 a) Din condit¸ia ni > ne rezult˘a c˘a n3 −15n+38 < 0, cu solut¸iile n ∈ {4, 5, 6, . . . , 11}. b) Condit¸ia ni + ne = kn2 este echivalent˘a cu n3 − 10n2 + (35 − 8k)n − 42 = 0, deci n ∈ {7, 14, 12}, c˘arora le corespund valorile k ∈ {1, 11, 33}. Not˘ a. ˆIntr-o manier˘a asem˘an˘atoare a rezolvat problema dl. Daniel V˘ acaru, Pite¸sti. L148. Pe latura (AB) a triunghiului ABC consider˘ am punctul D astfel ˆıncˆ at AB = 4 AD. De aceea¸si parte a laturii AB ca ¸si punctul C, lu˘ am un punct P Õ Õ ¸si P B = 2 P D. Demonstrat¸i c˘ astfel ˆıncˆ at P DA ≡ ACB a patrulaterul ABCP este inscriptibil. Nela Ciceu, Bac˘ au ¸si Titu Zvonaru, Com˘ ane¸sti Solut¸ia 1 (a autorilor). Consider˘am punctul Q astfel ˆıncˆat AB separ˘a P AQ Õ ≡ BP Õ ¸si Q, AB = 2AQ ¸si QAB D. Atunci = A AB  ‹ 1 PD P = ¸si obt¸inem c˘a △AQB ∼ △P DB, de unde PB 2 D Õ = P Õ Õ + m(ACB) Õ = Q AQB DB. Rezult˘a c˘a m(AQB) Õ m(PÕ DB) + m(P DA) = 180◦ , ceea ce asgur˘a inscriptibilitatea patrulaterului AQBC. Din AB = 2AQ ¸si AB = C AQ AD B 4AD deducem c˘a AQ2 = AD · AB, astfel scris = , prin urmare △AQD ∼ AB AQ Õ ≡ AQB Õ ¸si cum avem ¸si AQB Õ ≡P Õ △ABQ. Obt¸inem c˘a ADQ DB, rezult˘a c˘a punctele Õ ≡ QP Õ Q, D ¸si P sunt coliniare. Ne amintim c˘a QAB B, deci patrulaterul AQBP este inscriptibil. Inscriptibilitatea patrulaterelor AQBC ¸si AQBP arat˘a c˘a punctele A, Q, B, C, P sunt conciclice, de unde concluzia problemei. Solut¸ia 2 (Daniel V˘ acaru, Pite¸sti). Folosind relat¸ia lui Stewart ˆın △P AB, AD 3 1 2 BD obt¸inem c˘a P A · − P D2 + P B 2 · = AD · DB, deci P A2 · − P D2 + P B 2 · = AB AB 4 4 3 3 3 1 AB 2 ¸si, cum P B = 2P D, deducem c˘a P A2 · = AB 2 , adic˘a P A = AB. 16 4 16 2 Aplic˘am acum teorema cosinusului ˆın △ADP : P A2 = AD2 +DP 2 −2AD ·DP ·cos C, 156

3 1 de unde DP 2 − AB · DP · cos C − AB 2 = 0, relat¸ie pe care o vom privi ca ecuat¸ie 2 16 de gradul II ˆın necunoscuta DP. Singura solut¸ie pozitiv˘a a acestei ecuat¸ii este DP = √ 1 P A2 + P B 2 − AB 2 Õ · AB · (cos C + 3 + cos2 C). Cum cos AP B= , ˆınlocuind P A 4 2P A · P B Õ Õ ¸si P D cu valorile g˘asite anterior ¸si f˘acˆand calculele, obt¸inem cos AP B = cos(ACB), Õ Õ ceea ce arat˘ prin urmare AP B ≡ ACB, a c˘a patrulaterul ABCP este inscriptibil. L149. S˘ a se determine pozit¸ia punctului P pe directoarea parabolei P, astfel ˆıncˆ at aria triunghiului P T1 T2 s˘ a fie minim˘ a, unde T1 ¸si T2 sunt punctele de contact cu parabola ale tangentelor duse din P la P. Adrian Corduneanu, Ia¸si Solut¸ia 1 (a autorului). Raport˘am parabola la un reper canonic, avˆand originea ˆın vˆarful O al parabolei ¸si drept ax˘a a absciselor, perpendiculara dus˘a din O pe directoare. Ecuat¸ia parabolei este P : y 2 = 2px(p > 0), iar cea a directoarei d : x = p a!) faptul c˘a tangentele P T1 ¸si P T2 , duse la parabol˘a din − . Se ¸stie (sau se verific˘ 2 1 punctul P ∈ d, sunt ortogonale; prin urmare, SP T1 T2 = P T1 · P T2 . 2 Fie T1 (x0 , y0 ), cu y02 = 2px0 , y0 > 0. Ecuat¸ia tangentei P T1 este yy0 = p(x+x0 ) ¸si  2  p p 2 p2 p2 y0 y02 2 cum xp = − , obt¸inem yp = − + y0 + + , deci P T1 = x0 + − = 2 2y0 2 2 2y0 2 y0 1 (y 2 + p2 )3 . Tangenta P T2 trece prin P ¸si are panta − ; ecuat¸ia sa va fi y = 4p2 y02 0 p y0 p2 − x− . Intersectˆ and aceast˘a tangent˘a cu parabol˘a, g˘asim coordonatele puncp 2y0  3  3  2  2 p2 p p p p2 p2 y0 2 , − = + tului T2 , anume . Astfel, P T + − + − = 2 2y02 y0 2y02 2 y0 2y0 2 1 (y 2 + p2 )3 . 4y04 0 P˘atratul ariei triunghiului P T1 T2 se poate scrie acum ˆın funct¸ie de parametrul y0 :  ‹2 1 1 (y02 + p2 )3 S(y0 ) = . Derivata acestei funct¸ii este S ′ (y0 ) = · P T1 · P T2 = · 2 8p y03 3 (y 2 + p2 )2 (y02 − p2 ) ¸si se anuleaz˘a doar ˆın y0 = p (deoarece y0 a fost considerat 8py04 0 pozitiv). Obt¸inem c˘a S are un minim egal cu Smin = p2 , care se atinge cˆand y0 = p, p deci pentru punctul P − , 0 . 2 Solut¸ia 2 (Gheorghe Costovici, Ia¸si). Vom rezolva problema ˆıntr-un caz ceva mai general, cˆand P nu apart¸ine numaidecˆat directoarei parabolei, ci unei drepte perpendiculare pe axa de simetrie a acesteia ¸si care nu are niciun punct comun cu parabola. Fie y 2 = ax, a > 0, ecuat¸ia parabolei, iar Ti (xi , yi ), cu yi2 = axi , x1 ̸= x2 , 1 xi ̸= 0. Tangenta ˆın Ti la parabol˘a are ecuat¸ia (y +yi ) = axxi , i = 1, 2. Intersectˆand 2 x + x  1 2 cele dou˘a tangente, obt¸inem coordonatele punctului P , anume P , ax1 x2 . 2 Fie y = k, k < 0, ecuat¸ia dreptei pe care se afl˘a P ; atunci k = ax1 x2 < 0, deci 157

putem alege x1 > 0, x2 < 0. Aria triunghiului P T1 T2 va fi

SP T1 T2

1 1 = 1 2 1

x1 + x2 2 x1 x2

  ‹  1 k a k2 − x1 k − x21 + 2 2 . = 2 ax1 2 a x1

k y1 y2

Gˆandim aceast˘a arie ca o funct¸ie ˆın nedeterminata x1 , derivata acestei funct ɸii este  ‹ ‹2 k 3 a k −k · 4 · x21 + , iar punctele critice ale funct¸iei sunt x′1 = ¸si x21 − 4 x1 É a a a −k . Studiind semnul derivatei, se observ˘a c˘a x′1 este punct de minim x′′1 = − a pentru arie ¸si obt¸inem c˘a xp = 0. ˆIn concluzie, punctul P c˘autat este intersect¸ia dreptei date cu Ox. L150. Fie tetraedrul A1 A2 A3 A4 , iar P un punct ˆın interiorul s˘ au. Not˘ am cu Aij ∈ (Ai Aj ) proiect¸iile ortogonale ale lui P pe muchiile Ai Aj ale tetraedrului. Demonstrat¸i c˘ a 1 VP A12 A13 A23 + VP A12 A14 A24 + VP A13 A14 A34 + VP A23 A24 A34 ≤ VA1 A2 A3 A4 . 4 Cˆ and se atinge egalitatea? Marius Olteanu, Rm. Vˆ alcea Solut¸ie. Fie P1 = P r(A2 A3 A4 ) A1 ; din reciproca teoremei celor trei perpendiculare, obt¸inem c˘a P1 A23 ⊥ A2 A3 , P1 A24 ⊥ A2 A4 , P1 A34 ⊥ A3 A4 , prin urmare A23 A24 A34 este triunghiul podar al punctului P1 ˆın raport cu △A2 A3 A4 . Aria s1 a acestui triunghi este cel mult un sfert din aria S1 a △A2 A3 A4 . Dac˘a mai not˘am h1 = A1 P1 , VP A23 A24 A34 x1 s1 1 x1 x1 = P P1 , atunci = ≤ · . Introducem analog x2 , x3 , x4 VA1 A2 A3 A4 h1 S 1 4 h1 ¸si h2 , h3 , h4 ; putem scrie ˆınc˘a trei inegalit˘a¸ti analoage celei precedente. Concluzia problemei se obt¸ine adunˆand cele patru inegalit˘a¸ti ¸si ¸tinˆand seama de relat¸ia lui ‹ x2 x3 x4 x1 + + + =1 . Gergonne h1 h2 h3 h4 Deoarece 4si = Si doar atunci cˆand Pi coincide cu centrul cercului circumscris fet¸ei care se opune vˆarfului Ai , rezult˘a c˘a egalitatea se atinge cˆand P este centrul sferei circumscrise tetraedrului. L151. S˘ a se demonstreze c˘ a nu exist˘ a numere naturale n ¸si k astfel ˆıncˆ at h€

2+

√ Š2n+1 i h€ √ Šk i 3 = 4 + 15 .

Cosmin Manea ¸si Drago¸ a, Pite¸sti √ √ s Petric˘ 1 2 Solut¸ie. Avem c˘a (2 + 3)2n+1 = 22n+1 + C2n+1 22n 3 + C2n+1 22n−1 · 3 + √ √ √ √ 2n+1 . . . + C2n+1 ( 3)2n+1 = an + bn 3, cu an , bn ∈ N, iar (2 − 3)2n+1 = an − bn 3. √ 2n+1 √ 2n+1 √ 2n+1 √ 2n+1 Astfel, [(2 + 3) √] = [(2 + 3) + (2 − 3) − 1√+ 1 − (2 − 3) ] = 2n+1 [2an − 1 + 1 − (2√ − 3)2n+1 ] = 2a√ 1 − (2 − 3)√ ∈ (0, 1). La fel n − 1, deoarece √ k k stabilim √ c˘a (4 + 15) = ck + dk 15, (4 + 15) = ck − dk 15, ck , dk ∈ N, iar [(4 + 15)k ] = 2ck − 1. Dac˘a presupunem c˘a exist˘a n ¸si k cu propriet˘a¸tile cerute, 158

avem an = ck , deci 22n+1 + M3 = 4k + M15 ; rezult˘a c˘a 3|2 · 4n − 4k . Dar 2 · 4n − 4k = 2(M3 + 1) − (M3 + 1) = M3 + 1 ¸si obt¸inem contradict¸ia 3|1, prin urmare nu exist˘a n ¸si k cu proprietatea cerut˘a. L152. Pentru a, b, c ∈ R ¸si x ∈ R+ , demonstrat¸i inegalitatea 2

4

9 3 (x + 1) (a + b + c) 1 1 1 — ≤” ≤ 2+ 2+ 2. 2 2 2 2 2 2 2 2 2 a +b +c a b c 3(x + 1)(a + b + c ) + 2x(a + b + c) (ab + bc + ca) I. V. Maftei ¸si Dorel B˘ ait¸an, Bucure¸sti Solut ¸ ie. Utilizˆ a nd sumarea ciclic˘ a , inegalitatea din stˆ a nga se scrie 3[3(x2 + X X X X X 2 2 2 2 4 2 1)( a ) + 2x( a) ] · ( ab) ≤ (x + 1)( a) ( a ) ¸si aceasta se obt¸ine adunˆand inegalit˘a¸tile (1)

9(

P

ab)2 ≤ (

P

a)4 ; 3(

P

ab)2 ≤ (

P

a)2 (

P

X

a2 ),

X

prima multiplicat˘ a prin (x2 + 1)( a2 ), iar a doua prin 2x( Inegalitatea din dreapta se scrie sub forma (x + 1)2 a2 b2 c2 (

X

a)4 ≤ (

X

X

a2 b2 )(

X

ab)2 [(x2 + 1)(

a)2 .

a2 ) +

2x X 2 ( a) ] 3

¸si se poate obt¸ine adunˆand inegalit˘a¸tile P

a2 b2 c2 (

(2)

P

a)4 ≤ (

P

a2 b2 )(

ab)2 (

P

a2 ); a2 b2 c2 (

prima multiplicat˘ a prin (x2 + 1), iar a doua prin 2x(

P

X

a)2 ≤

1 P 2 2 P ( a b )( ab)2 , 3

a)2 .

X

Justificarea inegalit˘a¸tilor (1) ¸si (2) revine la demonstrarea inegalit˘a¸tilor 9(

X

X

X

X

X

X

( a)4 ; ( a)2 ≤ 3( a2 ); 3abc( a) ≤ ( ab)2 ; abc( a) ≤ relativ uzuale. L153. G˘ asit¸i toate funct¸iile f : R → R cu proprietatea c˘ a €

Š

f x2 + xy + yf (y) = xf (x + y) + f 2 (y) ,

X

ab)2 ≤

a2 b2 , care sunt

∀x, y ∈ R.

Adrian Zahariuc, student, Princeton Solut¸ie. C˘ aut˘ am un z ̸= x astfel ˆıncˆat x2 +xy = z 2 +zy; g˘asim z = −x−y. Atunci xf (x + y) + f 2 (y) = f (x2 + xy + yf (y)) = f (z 2 + zy + yf (y)) = zf (z + y) + f 2 (y) = (−x − y)f (−x) + f 2 (y). Rezult˘a c˘a xf (x + y) = −(x + y)f (−x), pentru orice x, y ∈ R. f (x) Deducem c˘a (x ̸= 0) este constant˘a, deci f (x) = cx, c ∈ R. Cum pentru x = 0 x ¸si y ̸= 0 obt¸inem f (0) = 0, rezult˘a c˘a f (x) = cx, c ∈ R, pentru orice x ∈ R. Se verific˘ a u¸sor c˘a toate aceste funct¸ii au proprietatea dorit˘a. Not˘ a. O solut¸ie corect˘a, dar ceva mai laborioas˘a, a fost primit˘a din partea d-lui Daniel V˘ acaru, Pite¸sti. L154. Fie P ∈ R [X] un polinom de gradul n ¸si p : R → R funct¸ia polinomial˘ a asociat˘ a. S ¸ tiind c˘ a mult¸imea {x ∈ R | p (x) = 0} are k elemente (distincte), iar 159

funct¸ia f : R → R, f (x) = |p (x)| este derivabil˘ a pe atat¸i c˘ a num˘ arul maxim de h nR, i ar˘ − 2k. r˘ ad˘ acini complexe nereale ale lui P este egal cu 2 2 Vlad Emanuel, student, Bucure¸sti Solut¸ie. Studiem derivabilitatea funct¸iei f . Dac˘a x0 ∈ R ¸si p(x0 ) ̸= 0, atunci p are semn constant ˆıntr-o vecin˘atate a lui x0 . Pe aceast˘a vecin˘atate f (x) = p(x) (sau f (x) = −p(x). Rezult˘a c˘a f este derivabil˘ a x0 ∈ R ¸si P (x0 ) = 0, atunci a ˆın x0 . Dac˘ P (x) − P (x0 ) |P (x)| − |P (x0 )| ′ = −|P ′ (x0 )| = −|P ′ (x0 )| ¸si fs (x0 ) = x→x lim = − x→x lim s 0 0 x − x x − x 0 0 xx0 ˆIn concluzie, f este derivabil˘a ˆın x0 dac˘a ¸si numai dac˘a −|P ′ (x0 )| = |P ′ (x0 )| ⇔ ′ P (x0 ) = 0. Prin urmare, f este derivabil˘a pe R dac˘a ¸si numai dac˘a orice r˘ad˘acin˘a a lui P este cel put¸in dubl˘a. Cum P are k r˘ad˘acini reale distincte, rezult˘a c˘a P are cel put¸in 2k r˘ad˘ acini reale. Pentru n = 2p, deducem c˘a P are cel mult 2p − 2k r˘ad˘acini complexe nereale, iar pentru n = 2p + 1, P are cel mult 2p + 1 − (2k + 1) r˘ad˘acini complexe nereale h n i (am ˆ folosit faptul c˘a r˘ad˘ acinile complexe sunt perechi). In final avem cel mult 2 − 2k 2 r˘ad˘ acini complexe nereale. Cum pentru n = 2p, P = (X − 1)2 . . . (X − k)2 (X 2 + 1)p−k verific˘a ipotezele problemei ¸si are 2p − 2k r˘ad˘acini complexe nereale, iar pentru n = 2p + 1, P = (X − 1)3 (X − 2)2 . . . (X − k)2 (X 2 + 1)p−k verific˘a ipotezele ad˘acini h n i ¸si are 2p − 2k r˘ nereale, conclucion˘am c˘a num˘arul c˘autat este egal cu 2 − 2k. 2 L155. Fie A, B ∈ M2 (C) dou˘ a matrice astfel ˆıncˆ at matricea AB − BA s˘ a fie −1 inversabil˘ a. S˘ a se arate c˘ a urma matricei (I2 + AB) (AB − BA) este egal˘ a cu 1. Florina Cˆ arlan ¸si Marian Tetiva, Bˆ arlad Solut¸ie. Consider˘am plinomul f (x) = det[I2 +AB +x(BA−AB)], care are gradul doi (coeficientul lui x2 este egal cu det(BA − AB) = det(AB − BA) ̸= 0). Observ˘am 1 c˘a f (0) = f (1), ceea ce arat˘a c˘a punctul de extrem al funct¸iei f este x = . 2 Pe de alt˘a parte, f (x) = det(BA − AB)det[(I2 + AB)(BA − AB)−1 + xI2 ] = det(BA − AB)(x2 + xtr(P ) + detP ), unde P = (I2 + AB)(BA − AB)−1 , ceea ce 1 1 ˆınseamn˘ a c˘a punctul de extrem este − tr(P ) = − tr[(I2 + AB)(BA − AB)−1 ]. Prin 2 2 1 1 −1 urmare, − 2 tr[(I2 + AB)(BA − AB) ] = , de unde tr[(I2 + AB)(AB − BA)−1 ] = 1. 2 Not˘ a. Solut¸ie corect˘a a dat dl. Daniel V˘ acaru, Pite¸sti.

160

Probleme propuse1 Clasele primare P.174. Mirela are un m˘ar, o par˘a ¸si o portocal˘a. Mama ˆıi spune s˘a a¸seze fructele pe dou˘a farfurii astfel ˆıncˆ at pe fiecare farfurie s˘a fie cel mult dou˘a fructe. ˆIn cˆate moduri poate a¸seza Mirela cele trei fructe? (Clasa I ) Inst. Maria Racu, Ia¸si P.175. Scriet¸i toate numerele mai mici ca 27 care se pot descompune sub forma indicat˘a al˘aturat. (Clasa I ) Diana T˘ an˘ asoaie, elev˘ a, Ia¸si a a+1 a+2 a+3 P.176. ˆIntr-o bombonier˘a sunt cinci bomboane cu fructe ¸si ¸sapte bomboane cu ciocolat˘a. Care este cel mai mic num˘ar de bomboane pe care trebuie s˘a-l lu˘am din bombonier˘a, f˘ar˘ a s˘a ne uit˘am, pentru a avea cel put¸in dou˘a bomboane cu ciocolat˘a? (Clasa a II-a) Alexandru Dumitru Chiriac, elev, Ia¸si P.177. Cum m˘asur˘ am 1litru de ap˘a folosind dou˘a vase negradate, unul de 5litru, iar cel˘alalt de 8litri? (Clasa a II-a) Mariana Nastasia, elev˘ a, Ia¸si P.178. Ar˘atat¸i c˘a, dac˘a restul este o cincime din sc˘az˘ator, atunci desc˘azutul se ˆımparte exact la 6. Care este cel mai apropiat desc˘azut de num˘arul 100 cu aceast˘a proprietate? (Clasa a III-a) Mirela Cucoranu, elev˘ a, Ia¸si P.179. Se dau produsele: a × b = 60, a × c = 70, a × d = 95. S¸tiind c˘a b + c + d este de 9 ori mai mare decˆat a, s˘a se afle valoarea lui a. (Clasa a III-a) Andreea Amarandei, elev˘ a, Ia¸si P.180. Ar˘atat¸i c˘a din ¸sirul 7, 28, 31, 46, 61, 100 nu putem extrage patru numere a c˘aror sum˘a s˘a se ˆımpart˘ a exact la trei. (Clasa a III-a) Drago¸s Iacob, elev, Ia¸si P.181. Un triunghi ¸si un p˘atrat au acela¸si perimetru, exprimat printr-un num˘ar natural. Care este cea mai mic˘a valoare a perimetrului? Cˆate valori posibile ale perimetrului sunt cuprinse ˆıntre 100 ¸si 200? (Clasa a IV-a) Andreea Alexa, elev˘ a, Ia¸si 1 2 3 k P.182. Aflat¸i cea mai mic˘a valoare a lui k astfel ˆıncˆat + + + ··· + 21 21 21 21 s˘a fie un num˘ ar natural. (Clasa a IV-a) Ionela B˘ ar˘ agan, elev˘ a, Ia¸si P.183. Se consider˘a nou˘a numere naturale a, b, c, . . . , i. Media aritmetic˘a a numerelor a ¸si b este 1, media numerelor c, d ¸si e este 5, iar media numerelor f, g, h ¸si i este 11. Aflat¸i media aritmetic˘a a numerelor a, b, c, . . . , i ¸si 9. (Clasa a IV-a) Ionel Nechifor, Ia¸si 1 Se

primesc solut¸ii pˆ an˘ a la data de 31 iunie 2010.

161

Clasa a V-a V.109. Aflat¸i cˆatul ¸si restul ˆımp˘art¸irii num˘arului 3 · 22009 la 5 · 22007 . Damian Marinescu, Tˆ argovi¸ste V.110. Determinat¸i patru numere naturale x, y, z, t cu proprietatea c˘a 2x−1 + 3 · 22y+1 + 5 · 23z+2 + 11 · 25t+1 = 2009. C˘ at˘ alina Dr˘ agan, Galat¸i V.111. Demonstrat¸i c˘a num˘arul 20200 are 261 de cifre la scrierea ˆın baza 10. Geanina H˘ avˆ arneanu, Ia¸si § ª 3n + 4 V.112. Demonstrat¸i c˘a mult¸imea A = x = n ∈ N, 1000 ≤ n ≤ 2009 are 4n + 3 1010 elemente. Daniela Munteanu, Ia¸si 1 1 1 13 V.113. Dac˘ a S = 1 + + + ... + , demonstrat¸i c˘a S > . 2 3 2009 2 Al. Gabriel Mˆır¸sanu, Ia¸si V.114. Se consider˘a ˆın plan cinci drepte distincte, care ˆımpart planul ˆın mai multe regiuni. Ar˘atat¸i c˘a oricum am alege 2009 puncte din plan, vor exista cel put¸in 126 de puncte dintr-o aceea¸si regiune. Nicolae Iv˘ a¸schescu, Craiova V.115. O mult¸ime de numere naturale A = {a1 , a2 , . . . , a9 } are elementele aranjate strict cresc˘ator; media aritmetic˘a a primelor dou˘a elemente este 1, media urm˘atoarelor trei este 5, iar media ultimelor patru este 11. Cˆate astfel de mult¸imi exist˘a? Ionel Nechifor ¸si Gabriel Popa, Ia¸si

Clasa a VI-a VI.109. Determinat¸i a, b, c, d ∈ N pentru care 2a + 3b + 5c + 7d = 87, dac˘a: a) a, b, c, d sunt numere prime; b) a, b, c, d sunt p˘atrate perfecte. Nicolae Iv˘ a¸schescu, Craiova VI.110. Determinat¸i perechile de numere naturale care au suma 2009 ¸si produsul multiplu al num˘ arului 2009. Dan Popescu, Suceava  ‹ 1 1 VI.111. Demonstrat¸i c˘a num˘arul A = 40! · 1 + + . . . + este natural, 2 40 divizibil cu 2009 · 72 (unde 40! = 1 · 2 · . . . · 40). Mihai Haivas, Ia¸si VI.112. Fie a, b, c, d ∈ N∗ astfel ˆıncˆat ad + bc = bd. Demonstrat¸i c˘a a2009 a2008 c a2 c ac c + + . . . + + + ∈ N. 2009 2008 2 b b d b d bd d C˘ at˘ alin Budeanu, Ia¸si 162

VI.113. Dup˘ a dou˘a reduceri succesive, pret¸ul unui frigider scade de la 2000 lei la 1620 lei. S¸tiind c˘a cele dou˘a reduceri sunt proport¸ionale cu pret¸urile r˘amase ˆın urma lor, aflat¸i pret¸ul frigiderului dup˘a prima reducere. Ciprian Baghiu, Ia¸si VI.114. Pe laturile [BC], [AC], [AB] ale triunghiului isoscel ABC (AB = AC) Õ = 2m(EDC) Õ ¸si m(DAC) Õ consider˘am punctele D, E, respectiv F , astfel ˆıncˆat m(BAD) Õ = 2m(F DB). Demonstrat¸i c˘a △AEF este isoscel. Doru Buzac, Ia¸si VI.115. Dreptele a ¸si b sunt perpendiculare pe segmentul [AB] ˆın A, respectiv ˆın B. Consider˘am punctele C ∈ (AB), M ∈ a, N, P ∈ b astfel ˆıncˆat ˆıntre oricare dou˘a dintre triunghiurile ACM, BCN ¸si BCP exist˘a cˆate o congruent¸˘a. S¸tiind c˘a Õ m(BP C) = 25◦ , determinat¸i m˘asurile unghiurilor triunghiului M N P. Andrei Nedelcu, Ia¸si

Clasa a VII-a VII.109. Fie ABCD dreptunghi, O mijlocul lui [AC], M ∈ (AO), N ∈ (OC), {P } = BM ∩ AD ¸si {Q} = BN ∩ CD. Demonstrat¸i c˘a O este centrul de greutate al 1 triunghiului BP Q dac˘a ¸si numai dac˘a OM = ON = AC. 6 Petru Asaftei, Ia¸si VII.110. M˘ asurile unghiurilor A, √ B ¸si C √ ale triunghiului ABC sunt direct propor¸tionale cu 5, 4 ¸si 3, iar BC = (2 + 2 2 + 2 3) cm. Demonstrat¸i c˘a perimetrul ¸si aria triunghiului sunt numeric egale. Constantin Apostol, Rm. S˘ arat VII.111. Fie ABC un triunghi ¸si punctele D ∈ (AC), E ∈ (AB), {P } = BD ∩ DA PC PD CE. Dac˘ a = k, demonstrat¸i c˘a k − (k + 1) = 1. DC PE PB Neculai Roman, Mirce¸sti (Ia¸si) VII.112. Fie ABCD trapez cu baza mare [AB], {E} = AD ∩ BC, {O} = AC ∩ BD, iar OP ∥AB, cu P ∈ (AD). Demonstrat¸i c˘a CP ¸si CE sunt bisectoarele Õ dac˘ (interioar˘ a, respectiv exterioar˘a) unghiului ACD, a ¸si numai dac˘a AB = AC. Claudiu S ¸ tefan Popa, Ia¸si √ √ 3 · bc VII.113. a) Demonstrat¸i c˘a b2 − bc + c2 ≥ √ , ∀b, c ∈ R∗+ . b2 + bc + c2 b = 120◦ , AB = c, AC = b, interpretat¸i b) Considerˆand un triunghi ABC cu m(A) geometric inegalitatea de la a). Dan Mocanu, elev, Ia¸si VII.114. Demonstrat¸i c˘a produsul a dou˘a numere naturale nenule consecutive nu poate fi egal cu produsul altor patru numere naturale consecutive. Mihai Cr˘ aciun, Pa¸scani 1 1 1 VII.115. Demonstrat¸i c˘a + + ··· + k > k, ∀n, k ∈ N∗ . n+1 n+2 4 ·n Cosmin Manea ¸si Drago¸s Petric˘ a, Pite¸sti 163

Clasa a VIII-a

VIII.109. Fie ABCDA′ B ′ C ′ D′ un cub de muchie a. Not˘am cu E, F, G, H, K, L mijloacele muchiilor AB, BC, CC ′ , C ′ D′ , D′ A′ , respectiv A′ A. Calculat¸i volumul poliedrului B ′ EF GHKL. Adrian Corduneanu, Ia¸si VIII.110. Fie V ABCD piramid˘a patrulater˘a regulat˘a. Not˘am cu

u = m((VÛ BC), (ABC)), v = m((V Û BC), (V CD)) ¸si t = m((V Û BC), (V AD)). Stabilit¸i dac˘a printre numerele u, v, t pot exista perechi de numere egale. (ˆ In leg˘ atur˘ a cu VIII.98 din RecMat 2/2008.) Claudiu S ¸ tefan Popa, Ia¸si √ VIII.111. Fie ABC un triunghi de laturi a, b, c, astfel ˆıncˆat b + c = a 2. Demonstrat¸i c˘a triunghiul ‚ √este ascut √ ¸Œitunghic dac˘a ¸si numai dac˘a b ¸si c sunt distincte a 2 3a 2 ¸si se afl˘a ˆın intervalul , . 4 4 Romant¸a Ghit¸˘ a ¸si Ioan Ghit¸˘ a, Blaj È x ∗ 2 2 2 VIII.112. Fie x, y ∈ R astfel ˆıncˆat xy, ¸si y x + (x + 1) + x (x + 1)2 sunt y toate numere rat¸ionale. Demonstrat¸i c˘a x ¸si y sunt tot numere rat¸ionale. Dan Nedeianu, Drobeta Tr. Severin ‹  1 1 1 1 1 1 VIII.113. Dac˘ a a, b, c ∈ R∗+ , demonstrat¸i c˘a + + = 2 + + a b c a+b b+c c+a dac˘a ¸si numai dac˘a a = b = c. D.M. B˘ atinet¸u-Giurgiu, Bucure¸sti VIII.114. Demonstrat¸i c˘a oricare ar fi numerele naturale impare m, n cu m > n + 2, exist˘a numere naturale x, a, b astfel ˆıncˆat x = a(a + m) = b(b + n). Titu Zvoranu, Com˘ ane¸sti VIII.115. Demonstrat¸i c˘a 5(a2 + b2 )2 ≤ 4a4 b4 + (a + b)4 , ∀a, b ∈ [1, +∞). Lucian Tut¸escu ¸si Ion Vi¸san, Craiova

Clasa a IX-a IX.101. Prin induct¸ie matematic˘a se arat˘a c˘a are loc inegalitate a lui Bernoulli (1) (1 + x)n ≥ 1 + nx, ∀n ∈ N, n ≥ 2 ¸si ∀x ∈ [−1, ∞), egalitatea fiind atins˘a pentru x = 0. Ar˘atat¸i c˘a: a) dac˘a n = 2k, k ∈ N∗ , atunci (1) are loc ∀x ∈ R; b) dac˘a n = 2k + 1, k ∈ N∗ , atunci (1) are loc ∀x ∈ [−2, ∞); c) dac˘a n = 3, atunci (1) are loc ∀x ∈ [−3, +∞), cu egalitate cˆand x ∈ {−3, 0}, iar pentru x ∈ (−∞, −3), (1) are loc cu sens contrar. Dorin Dutkay, Orlando (U.S.A.) ¸si Florin Popovici, Bra¸sov IX.102. Rezolvat¸i ˆın R3 sistemul: 1 1 1 1 1 1 1 1 1 x+y+z = 2− − − ; x2 +y 2 +z 2 = 6− 2 − 2 − 2 ; x3 +y 3 +z 3 = 2− 3 − 3 − 3 . x y z x y z x y z Vasile Chiriac, Bac˘ au 164

IX.103. Fie x, y, z ∈ R cu 0 ≤ x ≤ y ≤ z. Dac˘a α ∈ R este astfel ˆıncˆat αx + (1 − α)z ≥ 0, demonstrat¸i c˘a αx + (1 − α)y ≥ 0 ¸si αy + (1 − α)z ≥ 0. Ovidiu Pop, Satu Mare IX.104. Fie A, B, C, D patru puncte ale cercului C(O, r), {M } = AB ∩ CD, N ¸si P mijloacele coardelor [AB], respectiv [CD], iar Ω cel de-al patrulea vˆarf al paralelogramului N M P Ω. −−→ −−→ −−→ −−→ −−→ a) Ar˘atat¸i c˘a M A + M B + M C + M D = 2M Ω. b) Paralelele prin C ¸si D la AB ¸si paralelele prin A ¸si B la CD se taie dou˘a cˆate dou˘a ˆın patru puncte ce determin˘a un paralelogram de centru Ω. c) Ω = O dac˘a ¸si numai dac˘a AB ⊥ CD. Diana Vrˆ anceanu, elev˘ a ¸si Dumitru Mihalache, Bˆ arlad IX.105. ˆIntr-un triunghi, cu notat¸iile uzuale, demonstrat¸i echivalent¸a condit¸iilor: (i) R = ra ; (ii) cos A = cos B + cos C. Temistocle Bˆırsan, Ia¸si

Clasa a X-a X.101. Calculat¸i suma S =

√ 1 2k+1 √ 2k+1 arcsin k=1 k+1 n arctg X



X.102. Rezolvat¸i ecuat¸ia

1 7 + log2 x − 2x 4

. Bencze Mih´ aly, Bra¸sov

‹

+

7 = 0. 4

Eugen Jecan, Dej −→ X.103. Fie S, U, A trei puncte distincte. Rotind vectorul SA ˆın jurul lui S, cu −→ un arc α ∈ (−π, π), obt¸inem punctul S ′ ; rotind apoi U A ˆın jurul lui U , cu un arc −−→ −−−→ β ∈ (−π, π), obt¸inem U ′ , U ′ ̸= S ′ . Fie M ∈ S ′ U ′ astfel ˆıncˆat S ′ M = k · M U ′ , unde k ∈ R\{0, −1}. Demonstrat¸i c˘a pozit¸ia punctului M nu depinde de A atunci ¸si numai atunci cˆand k = 1, β = α ± π * . Diana Vrˆ anceanu, elev˘ a ¸si Dumitru Mihalache, Bˆ arlad X.104. Fie p, la , lb , lc semiperimetrul, respectiv lungimile bisectoarelor unui triunghi.√Determinat¸i numerele reale α ¸si β, ˆın funct¸ie de p, ¸stiind c˘a solut¸iile ecuat¸iei x3 − p 3 · x2 + αx − β = 0 sunt la , lb ¸si lc . C˘ at˘ alin Calistru, Ia¸si X.105. Determinat¸i cel mai mare num˘ar real α astfel ˆıncˆat inegalitatea + sin 3x+y sin x+3y sin x + sin y x+y 4 4 ≤α· + (1 − α) sin 2 2 2 s˘a fie adev˘arat˘ a pentru orice x, y ∈ [0, π]. Marian Tetiva, Bˆ arlad

Clasa a XI-a XI.101. Pentru a ∈ R∗+ , calculat¸i lim x x→∞

* Generalizare



 a x − ea . x D.M. B˘ atinet¸u-Giurgiu, Bucure¸sti

1+

a problemei comorii din insul˘ a a lui G. Gamow, din One, Two, Theree . . . Infinity.

165

XI.102. Determinat¸i funct¸iile continue f : R → R cu proprietatea c˘a f (x2 ) − f (y 2 ) = (x + y)f (x − y), ∀x, y ∈ R. Gheorghe Iurea, Ia¸si ∗ XI.103. Fie (xn )n≥1 ⊂ R+ astfel ˆıncˆat lim n(xn+1 − xn ) = a ∈ (1, +∞); definim n→∞ n ln n . Calculat¸i lim (yn )n . ¸sirul (yn )n≥1 prin yn = n→∞ x1 + x2 + . . . + xn Cosmin Manea ¸si Drago¸s Petric˘ a, Pite¸sti  5  1 25 35 n5 XI.104. Calculat¸i lim + 2 + 3 + ... + n . n→∞ 51 5 5 5 Neculai Roman, Mirce¸sti (Ia¸si) XI.105. Consider˘am matricele Ak , Bk ∈ Mk (C), k ∈ N\{0, 1}, astfel ˆıncˆat detAk = α ∈ C∗ , ∀k ∈ N\{0, 1}. Studiat¸i convergent¸a ¸sirului (an )n≥2 definit prin n X det(Ak x + Bk ) an = lim . x→∞ k! · xk k=2 C˘ at˘ alin Calistru, Ia¸si

Clasa a XII-a XII.101. Rezolvat¸i ecuat¸ia x2 + x + 1 = 0 ˆın Z13 ¸si ˆın Z19 , apoi deducet¸i c˘a 247 n n divide (37 −1 − 1)(74 −1 − 1), oricare ar fi n ∈ N. Mihai Haivas, Ia¸si ¸si I.V. Maftei, Bucure¸sti  π cos 2x XII.102. Determinat¸i primitivele funct¸iei f : 0, → R, f (x) = . 4 (tg x + ctgx)2009 Nicoleta Bran, Craiova  π XII.103. Demonstrat¸i c˘a exist˘a c ∈ 0, pentru care 4 Z

π 4

(etg x − 1)dx ≤

0

π 2 etg c · . 32 cos2 c

Bogdan Victor Grigoriu, F˘ alticeni XII.104. Determinat¸i funct¸iile derivabile f : [0, 1] → R pentru care f (0) = 0, iar f ′ (x) = f (ax), ∀x ∈ [0, 1], cu a ∈ [0, 1] fixat. Gheorghe Iurea, Ia¸si x + y  f (x) + f (y) XII.105. Fie f : (0, ∞) → R o funct¸ie semiconvex˘a (f ≤ , 2 2 ∀x, y ∈ (0, ∞)). ¸i c˘a˜ pentru orice x ∈ (0, ∞), ¸sirul (fn (x))n≥1 definit prin fn (x) = • a) Demonstrat ‹ 1 n f x+ − f (x) , este monoton. n b) Deducet ¸i c˘a pentru orice x ∈ (0, ∞), ¸sirul (en (x))n≥1 definit prin en (x) =  ‹ 1 n 1+ , este cresc˘ator. nx Dan S ¸ tefan Marinescu ¸si Viorel Cornea, Hunedoara

166

Probleme pentru preg˘ atirea concursurilor A. Nivel gimnazial G166. Demonstrat¸i c˘a urm˘atoarele propozit¸ii sunt adev˘arate. a) ∀n ∈ N, n ≥ 2, ∃x1 , x2 , . . . , xn ∈ N∗ astfel ˆıncˆat x1 x2 + x2 x3 + . . . + xn x1 = x1 x2 . . . xn . b) ∀n ∈ N, n ≥ 5, @x1 , x2 , . . . , xn ∈ 2N∗ astfel ˆıncˆat x1 x2 + x2 x3 + . . . + xn x1 = x1 x2 . . . xn . c) ∃x1 , x2 , . . . , xn ∈ 2N + 1 astfel ˆıncˆat x1 x2 + x2 x3 + . . . + xn x1 = x1 x2 . . . xn ⇔ n ∈ 2N∗ + 1. Dan Popescu, Suceava G167. Fie 1 = d1 < d2 < . . . < dk = n tot¸i divizorii pozitivi ai num˘arului natural n. Dac˘a exist˘a i, j cu j > i > 13 ¸si d27 + d2i = d2j , ar˘atat¸i c˘a n este multiplu de 8. Titu Zvonaru, Com˘ ane¸sti G168. Pentru x, y, z ∈ R∗+ , demonstrat¸i c˘a are loc inegalitatea   x(y + z) y(x + z) z(x + y) x2 y2 z2 + + ≤2 + + . x + yz y + xz z + xy x + yz y + xz z + xy S ¸ tefan Gavril, Piatra Neamt¸ G169. Demonstrat¸i c˘a exist˘a o infinitate de numere irat¸ionale α cu proprietatea c˘a α3 ¸si α2 + α sunt, de asemenea, irat¸ionale. Gabi Ghidoveanu ¸si Dumitru Mihalache, Bˆ arlad G170. O mult¸ime A ⊂ R, de cardinal 2009, are proprietatea c˘a fiecare element al ei este mai mare decˆat o zecime din suma celor 2008 numere r˘amase. Ar˘atat¸i c˘a A cont¸ine cel put¸in 12 numere negative. Andrei Nedelcu, Ia¸si G171. Punctele planului care au, ˆın raport cu un reper y cartezian, ambele coordonate numere naturale, le parcurgem ˆın sensul indicat de s˘aget¸i ˆın figur˘a, pornind din origine. Not˘am cu an,k pozit¸ia punctului de coordonate (n, k) ˆın ¸sirul obt¸inut (de exemplu, a0,0 = 1, a0,2 = 4, a2,2 = 13 etc.). Exprimat¸i num˘ arul an,k ˆın funct¸ie de n ¸si de k. Lucian Georges L˘ adunc˘ a, Ia¸si

0

x

G172. O tabl˘a dreptunghiular˘a m × n, m, n ≥ 2, are p˘atr˘a¸telele unitate de la intersect¸iile liniilor de ordin impar cu coloanele de ordin impar colorate ˆın negru, restul p˘atr˘ a¸telelor fiind albe. A recolora o linie (coloan˘a) ˆınseamn˘a a schimba culorile tuturor p˘atr˘ a¸telelor acelei linii (coloane). Ar˘atat¸i c˘a tabla nu poate fi transformat˘a ˆıntr-una complet alb˘a prin recolorarea cˆatorva linii ¸si coloane. R˘ azvan Ceuc˘ a, elev, Ia¸si G173. Not˘am cu T (a, b, c) triunghiul care are laturile de lungimi a, b ¸si c. Demonstrat¸i c˘a triunghiurile T (b, 2c, 2mb ) ¸si T (c, 2b, 2mc ) pot fi confect¸ionate (pe rˆand) dintr-o aceea¸si bucat˘a de carton, f˘ar˘ a pierdere de material. Petru Asaftei, Ia¸si 167

b = 40◦ . S˘ G174. Se consider˘a triunghiul ABC isoscel cu m(A) a se arate c˘a ◦ Õ Õ nu exist˘a puncte P ∈ Int ABC pentru care m(P AB) = 30 , m(P BC) = 10◦ ¸si Õ m(P CA) = 35◦ . Gabriel Popa ¸si Paul Georgescu, Ia¸si

G175. Fie ABCD un patrulater ˆınscris ˆın cercul de raz˘a R. Demonstrat¸i c˘a AB · AD + CB · CD ≤ 4R2 . Gheorghe Costovici, Ia¸si

B. Nivel liceal L166. Fie ABCD un dreptunghi, iar C un cerc prin A, care intersecteaz˘a (AB), (AC) ¸si (AD) ˆın M, N , respectiv P . Ar˘atat¸i c˘a AM · AB + AP · AD = AN · AC. Gheorghe Iurea, Ia¸si L167. Fie ABC un triunghi cu AB > AC. Cercul ˆınscris ˆın triunghi este tangent laturilor BC ¸si AC ˆın D, respectiv E. Consider˘am T un punct pe latura [BC] ¸si not˘am cu J centrul cercului ˆınscris ˆın △ABT. Dac˘a DE trece prin mijlocul segmentului [CJ], demonstrat¸i c˘a triunghiul AT C este isoscel. Titu Zvonaru, Com˘ ane¸sti L168. Demonstrat¸i c˘a ˆın orice triunghi, cu notat¸iile uzuale, are loc inegalitatea a b c 11p2 − 15r2 − 60Rr 3 + + ≥ ≥ . b+c c+a a+b 6p2 − 6r2 − 24Rr 2 Marius Olteanu, Rm. Vˆ alcea L169. Care este probabilitatea ca razele cercurilor exˆınscrise unui triunghi ales aleator, s˘a fie laturile unui nou triunghi? Petru Minut¸, Ia¸si L170. Fie n ∈ N, n ≥ 2 ¸si a1 , a2 , . . . , an ∈ R+ cu a1 +a2 +. . .+an = S. Consider˘am k ∈ N, 1 ≤ k ≤ n − 1 ¸si α1 , α2 ∈ R+ cu α1 + α2 = 1. Demonstrat¸i inegalitatea X

(ai1 + ai2 + . . . + aik )α1 (S − ai1 − . . . − aik )α2 ≤

1≤i1 0 exists such that #

ai x + bi −

”√

—

ax + b ∈ {0, 1} , for ∀x > x0 .

i=n

Marian Tetiva, Bˆ arlad L175. Show that [n]

2 X

Cn2k Ωk = 2n Ωn

k=0

(2k − 1)!! , k ∈ N∗ ; by convention Ω0 = 1. where Ωk = (2k)!! Gheorghe Costovici, Ia¸si 171

Pagina rezolvitorilor COVASNA ´ AR ´ Emese). S ¸ coala cu clasele I-VIII ”Avram Iancu”. Clasa a V-a (prof. LAZ ˆ ˘ ˘ ˆ BARLA Oana: P(161,163), V(95,96), VI.96; PATRANJEL Andrada-Maria: P(161, ˘ 163), V(95,96), VI.96; TIMARU Carmen-Ioana: P(161,163), V(95,96), VI.96; UT ¸A ˘ Ioana: P(161, 163), V(95,96), VI.96; VLADEA Diana: P(161,163), V(95,96), VI.96. CRAIOVA ˘ Liceul Teoretic ”Tudor Arghezi”. Clasa a V-a (prof. DRACEA Dorina). VˆIRLAN Leonard: P.161, V(95-97). IAS ¸I S ¸ coala nr. 3 ”Al. Vlahut¸˘ a”. Clasa I (inst. MAXIM Gabriela). CUCURUZ ˘ Raluca: P(154,155, 157, 164-167); DASCALU Lorena: P(154,155,157,164-167); POPESCU Alexandru: P(154,157,164-167); ROBU Carmen: P(154,157,164-167); S¸ER˘ BANOIU Alexandru: P(154,157,164-167); TORAC George: P(154,157,164-167); ˘ NICA Daniel: P(154,155,157,164-167). Clasa a II-a (inst. CRACIUN Marilena). ˘ CREANGA Adrian: P(154,155,157,164-167); FILIP Vlad: P(154,155,157,164-167); ˘ ˘ ValenPOPESCU Claudia: P(154,155,157,164-167). Clasa a II-a (inv. MARIUT ¸A tina). ENEA Codrut¸ Alexandru: P(154, 155, 157, 164-167); GHEORGHIU BeatriceElena: P(154,155,157,164-167); HERGHELEGIU M˘ad˘alina: P(154,155,157,164-167); ˘ Alin: P(154,155,157,164-167); HUHU Paula: P(154,155,157,164-167); HREAPCA ˘ Andreea-Maria: POPOVICIU Teodor-Andrei: P(154,155,157,164-167); ROMILA P(154,155,157,164-167). Clasa a VI-a (prof. MARIN Mirela). MARCU Ana: V(105,106), VI(102,104,105); RUSU M˘ad˘alina-Andreea: V(105,106), VI(102,104,105); TIBA S¸tefana-Alexandra: V(105,106), VI(102,104,105). S ¸ coala nr. 13 ”Alexandru cel Bun”. Clasa I (inst. COJOCARIU Ana). ACATRINEI Andra: P(154,164-167); PERDUN Patricia-Maria: P(154,164-167); PRISECARU Alexandru-Julian: P(154,164-167), SAMSON Constantin-C˘at˘alin: P(154,164167); S¸TEFAN Tudor: P(154,164-167); ZAHARIA S¸tefan-Eusebiu: P(164,164-167). ˘ Elena). BACIU TuS ¸ coala nr. 14 ”Gh. Mˆ arzescu”. Clasa a IV-a (inst. NUT ¸A ˘ George-S¸tefan: P(164-167,169-171); NEDELEU dor: P(164-167,170); CHIRILUT ¸A ˆ Iulia: P(164-167,170); POSTUDOR Georgiana-M˘ad˘alina: P(164-167,170); T ¸ AMBALARIU Ioana-Vasilica: P(164-167,170). ˘ S ¸ coala nr. 22 ”B.P. Ha¸sedeu”. Clasa I (inv. BLAJUT ¸ Cristina). SAVA Cosmina ˘ Ioana: P(164-167,169). Clasa a IV-a (inv. S¸TEFAN Liviu). BLAJUT ¸ CristinMarian: P(167-168). ˆ S ¸ coala nr. 26 ”George Co¸sbuc”. Clasa a II-a (inst. VARLAN Elena). AMARIEI ˘ ALUC ˘ Romeo: P(154-157,164-167); GHEBAN Andreea: P(154-157,164-167); PAV ˘ Ana-Maria: P(154-157,164-167); PICHIU Cosmin: P(154-157,164-167); TATARU Alice: P(154,157,164-167); T ¸ IPLEA Iulian: P(154-157,164-167). Clasa a III-a (inv. BUCATARIU Rica). BARHAN S¸tefana-Adina: P(164-166,168,169,171,172); CHIRIAC Alexandra: P(164-166,168,169,171,172) CUPET ¸ Valeria: P(164-166,168, ˘ Andrei-David: P(164-166,168,169,171,172); FRUNZA ˘ Diana169,171,172); FRUNZA Mihaela: P(164-169,171,172); IVANOV Alexandra: P(164-166,168,169,171,172); 172

MˆINDRU Liana: P(164-169,171,172). Clasa a IV-a (inst. RACU Maria). APA˘ CHIT ¸ EI Aura-Georgiana: P(164-171); BURA Emma-Andreea: P(164-170); CRACIUN Ioana-Daniela: P(164-171); LES¸OVSCHI Alexandra-Ioana: P(164-169); LUPU Roxana-Elena: P(164-169). S ¸ coala SAM ”M. Kog˘ alniceanu”, T ¸ ig˘ ana¸si. Clasa a II-a (ˆınv. GALIA Paraschiva). CAZADOI Ioana-Cristina: P(154,155,157,164-167); DUCA Cristina Mihaela: P(154155,157,164-167); SANDU Rebeca: P(154-155,157,164-167). ˆ SFANTU GHEORGHE (Tulcea) ˘ Elena). HALCHIN S ¸ coala generak˘ a cu clasele I-VIII. Clasa a II-a (ˆınv. GAVRILA ˘ Ioana: P(154-158); SIDORENCU Adrian: P(154-158). Clasa a IV-a (ˆınv. GAVRILA ˘ Elena). BALAN Silviu: P(154-161); CLADIADE Bogdan-Robert: P(154-161); CUCU Delia: P(154-161); EFIMOV Cosmin Alexandru: P(154-161). Clasa a VI-a (prof. ˘ SAILEANU Sorin). SIDORENCU Andrei: V(95,96), VI.96, VII(96,98), VIII.96.

Elevi rezolvitori premiat¸i S ¸ coala nr. 26 ”G. Co¸sbuc”, Ia¸si 1. LES ¸ OVSCHI Alexandra-Ioana (cl. a IV-a): 2/2008(6pb), 1/2009(6pb), 2/2009 (6pb). 2. LUPU Roxana-Elena (cl. a IV-a): 2/2008(6pb), 1/2009(6pb), 2/2009(6pb).

Vizitat¸i noua pagina web a revistei:

http://www.recreatiimatematice.ro

173

IMPORTANT • ˆIn scopul unei leg˘aturi rapide cu redact¸ia revistei, pot fi utilizate urm˘atoarele adrese e-mail: t [email protected] ¸si [email protected] . Pe aceast˘a cale colaboratorii pot purta cu redact¸ia un dialog privitor la materialele trimise acesteia, procurarea numerelor revistei etc. Suger˘am colaboratorilor care trimit probleme originale pentru publicare s˘a le numeroteze ¸si s˘a-¸si ret¸in˘ a o copie xerox a lor pentru a putea purta cu u¸surint¸˘a o discut¸ie prin e-mail asupra accept˘arii/neaccept˘arii acestora de c˘atre redact¸ia revistei. • La problemele de tip L se primesc solut¸ii de la orice iubitor de matematici elementare (indiferent de preocupare profesional˘ a sau vˆ arst˘ a ). Fiecare dintre solut¸iile acestor probleme - ce sunt publicate ˆın revist˘a dup˘a un an - va fi urmat˘a de numele tuturor celor care au rezolvat-o. • Adres˘ am cu insistent¸˘ a rug˘ amintea ca materialele trimise revistei s˘ a nu fie (s˘ a nu fi fost) trimise ¸si altor publicat¸ii. • Rug˘am ca materialele tehnoredactate s˘a fie trimise pe adresa redact¸iei ˆınsot¸ite de fi¸sierele lor (de preferint¸˘a ˆın LATEX). • Pentru a facilita comunicarea redact¸iei cu colaboratorii ei, autorii materialelor sunt rugat¸i s˘a indice adresa e-mail.

174

Revista semestrială RECREAŢII MATEMATICE este editată de ASOCIAŢIA “RECREAŢII MATEMATICE”. Apare la datele de 1 martie şi 1 septembrie şi se adresează elevilor, profesorilor, studenţilor şi tuturor celor pasionaţi de matematica elementară. În atenţia tuturor colaboratorilor Materialele trimise redacţiei spre publicare (note şi articole, chestiuni de metodică, probleme propuse etc.) trebuie prezentate îngrijit, clar şi concis; ele trebuie să prezinte interes pentru un cerc cât mai larg de cititori. Se recomandă ca textele să nu depăşească patru pagini. Evident, ele trebuie să fie originale şi să nu fi apărut sau să fi fost trimise spre publicare altor reviste. Rugăm ca materialele tehnoredactate să fie însoţite de fişierele lor. Problemele destinate rubricilor: Probleme propuse şi Probleme pentru pregătirea concursurilor vor fi redactate pe foi separate cu enunţ şi demonstraţie/rezolvare (câte una pe fiecare foaie) şi vor fi însoţite de numele autorului, şcoala şi localitatea unde lucrează/învaţă. Redacţia va decide asupra oportunităţii publicării materialelor primite. În atenţia elevilor Numele elevilor ce vor trimite redacţiei soluţii corecte la problemele din rubricile de Probleme propuse şi Probleme pentru pregatirea concursurilor vor fi menţionate în Pagina rezolvitorilor. Se va ţine seama de regulile: 1. Pot trimite soluţii la minimum cinci probleme propuse în numărul prezent şi cel anterior al revistei; pe o foaie va fi redactată soluţia unei singure probleme. 2. Elevii din clasele VI-XII au dreptul să trimită soluţii la problemele propuse pentru clasa lor, pentru orice clasă mai mare, din două clase mai mici şi imediat anterioare. Elevii din clasa a V-a pot trimite soluţii la problemele propuse pentru clasele a IV-a, a V-a şi orice clasă mai mare, iar elevii claselor I-IV pot trimite soluţii la problemele propuse pentru oricare din clasele primare şi orice clasă mai mare. Orice elev poate trimite soluţii la problemele de concurs (tip G şi L). 3. Vor fi menţionate următoarele date personale: numele şi prenumele, clasa, şcoala şi localitatea, precum şi de numele profesorului cu care învaţă. 4. Plicul cu probleme rezolvate se va trimite prin poştă (sau va fi adus direct) la adresa Redacţiei: Prof. dr. Temistocle Bîrsan Str. Aurora, nr. 3, sc. D, ap. 6, 700 474, Iaşi Jud. IAŞI E-mail: [email protected]

CUPRINS Către cititori dupǎ zece ani de apariţie a revistei .......................................................................... 85 NECULAI GHEORGHIU (1930-2009) ............................................................................................ 87

ARTICOLE ŞI NOTE T. BÎRSAN, G. DOSPINESCU – Conjectura Beal pentru polinoame ..................................... 89 C.-L. BEJAN – Teorema lui Brouwer - un caz particular elementar........................................ 94 D. MĂRGHIDANU – Exponentul de triangularitate al unui triunghi...................................... 96 D. POPESCU – Asupra unui şir de integrale Riemann ............................................................. 98 A. VERNESCU – Dualitatea unor sume combinatoriale ........................................................ 101

NOTA ELEVULUI D. M. MOCANU – Asupra problemei C.O: 5004 din G.M.................................................... 105

CORESPONDENŢE A. REISNER – Matrices à coefficients dans un corps fini ..................................................... 107

CHESTIUNI METODICE M. MIHEŢ – O metodǎ de rezolvare a problemelor ..............................................................111 S. BOGA – Metoda identificǎrii.............................................................................................115

CUM CONCEPEM ... CUM REZOLVĂM D. MIHALACHE, M. TETIVA– Bisectoarele exterioare nu sunt ca bisectoarele interioare..117

MATEMATICA ÎN CLASELE PRIMARE D.M. BĂTINEŢU-GIURGIU – Metoda falsei ipoteze - variante de utilizare.........................121

ŞCOLI ŞI DASCĂLI Liceul Teoretic "Garabet Ibrăileanu"........................................................................................123 LIDIA COHAL (1930-2009) ...................................................................................................126

CONCURSURI ŞI EXAMENE Concursul de matematică "Al. Myller", ed. a VII-a, 2009 .................................................. 127 Concursul de matematică "Florica T. Câmpan", 2009 ......................................................... 130

PROBLEME ŞI SOLUŢII Soluţiile problemelor propuse în nr. 2/2008 .......................................................................... 135 Soluţiile problemelor pentru pregătirea concursurilor din nr. 2/2008 ................................ 152 Probleme propuse ...................................................................................................................... 161 Probleme pentru pregătirea concursurilor .............................................................................. 167 Training problems for mathematical contests ....................................................................... 169 Pagina rezolvitorilor ............................................................................................................... 172 ISSN 1582 – 1765

7 lei